Spring HESI

Lakukan tugas rumah & ujian kamu dengan baik sekarang menggunakan Quizwiz!

Quality of Life (QOL)

A patient's perceptions of how he or she deals with a disease or with everyday life when suffering from a particular condition. It is subjective because information cannot be measured objectively; however, it has been in the health-care literature for at least 20 years.

A community center is presenting a nurse-led program on the Patient Protection and Affordable Care Act. Which statement made by a participant indicates a need for further teaching? A). "My small company will now have to offer the 75 employees health insurance or pay a penalty." B). "As long as my son is a full-time student in College, I will be able to keep him on my health insurance until he is 26 years old." C). "I signed up for the state health insurance exchange before the designated deadline to make sure I had health insurance." D). "Since I have now been diagnosed with diabetes, my health insurance plan cannot charge me higher premiums."

"As long as my son is a full-time student in College, I will be able to keep him on my health insurance until he is 26 years old." Patient Protection and Affordable Care Act, dependents can remain on health plans until the age of 26, regardless of their status in school. They are not required to be full-time students to be eligible for the benefit. All other options are correct.

Which statement made by a nurse shows that the nurse is engaging in an activity to help cope with secondary traumatic stress and burnout?

"I am enjoying my quilting group that meets each week at my church."

After a class on Pender's health promotion model students make the following statements. Which statement does the faculty member need to clarify?

"Perceived self-efficacy is not related to the model"

As part of a faith community nursing program in her church, a nurse is developing a health promotion program on breast self-examination for the women's group. Which statement made by one of the participants is related to the individual's perception of susceptibility to an illness?

"Since my mother had breast cancer, I know that I am at increased risk for developing breast cancer"

A patient comes to the local health clinic and states: "I've noticed how many people are out walking in my neighborhood. Is walking good for you?" What is the best response to help the patient through the stages of change for exercise? 1. "Walking is OK. I really think running is better." 2. "Yes, walking is great exercise. Do you think you could go for a 5-minute walk next week?" 3. "Yes, I want you to begin walking. Walk for 30 minutes every day and start to eat more fruits and vegetables." 4. "They probably aren't walking fast enough or far enough. You need to spend at least 45 minutes if you are going to do any good."

"Yes, walking is great exercise. Do you think you could go for a 5 minutes walks next week?"

Misdemeanor

(n.) a crime or offense that is less serious than a felony; any minor misbehavior or misconduct fine or jail time less than a year

liability

(n.) a debt; something disadvantageous

Sources of Diagnostic Errors in Interpreting/Analyzing Data

- Inaccurate interpretation of cues - Failure to consider conflicting cues - Using an insufficient number of cues - Using unreliable or invalid cues - Failure to consider cultural influences or developmental stage

In evaluating the gross-motor development of a 5-month-old infant, which of the following would the nurse expect the infant to do? 1 Roll from abdomen to back 2 Move from prone to sitting unassisted 3 Sit upright without support 4 Turn completely over

1 Roll from abdomen to back

Which factors influence a person's approach to death? (Select all that apply) 1. Culture 2. Age 3. Spirituality 4. Personal Beliefs 5. Previous experiences with death 6. Gender 7. Level of education 8. Degree of social support

1. Culture 3. Spirituality 4. Personal Beliefs 5. Previous experiences with death 8. Degree of social support

A year after her husband's death, a widow visits the unit on which he died. She talks about the anniversary and how much she misses him. Which type of giref is she experiencing? 1. Normal 2. Complicated 3. Chronic 4. Disenfranchised

1. Normal

A grieving patient complains of confusion, inability to concentrate, and insomnia. What do these symptoms indicate? 1. These are normal symptoms of grief 2. There is a need for pharmacological support for insomnia 3. The patient is experiencing complicated grief 4. These are common complaints of the admitted patient.

1. These are normal symptoms of grief

Steps in ethical decision making

1. determine if issue is ethical 2. gather relevant data 3. determine ones own values on issue 4. verbalize issue and consider course of action 5.negotiate outcome 6. evaluate action

A health care provider's prescription reads to administer an intravenous (IV) dose of 400,000 units of penicillin G benzathine (Bicillin). The label on the 10-mL ampule sent from the pharmacy reads penicillin G benzathine (Bicillin), 300,000 units/mL. The nurse prepares how much medication to administer the correct dose? Record your answer using one decimal place.

1.3 mL

A health care provider prescribes 1 unit of packed red blood cells to infuse over 4 hours. The unit of blood contains 250 mL. The drop factor is 10 drops (gtt)/1 mL. The nurse prepares to set the flow rate at how many drops per minute? Record your answer to the nearest whole number.

10 drops

Which of the following is the best intervention to help a hospitalized patient maintain some autonomy? 1. Use therapeutic techniques when communicating with the patient 2. Allow the patient to determine timing and scheduling of interventions 3. Encourage family to only visit for short periods of time 4. Provide the patient with a private room close to the nurse's station.

2. Allowing the patient to determine timing and scheduling of interventions

With the exception of pregnant or lactating women, the young adult has usually completed physical growth by the age of: 18. 20. 25. 30.

20

A health care provider's prescription reads 1000 mL of normal saline (NS) to infuse over 12 hours. The drop factor is 15 drops (gtt)/1 mL. The nurse prepares to set the flow rate at how many drops per minute? Record your answer to the nearest whole number.

21 drops

What is the palliative care team's primary obligation for the patient with severe pain? 1. Postmortem care 2. Teaching about grief stages 3. Enhancing the patient's quality of life 4. Supporting the family after the death

3. Enhancing the patient's quality of life

A health care provider prescribes 1000 mL of normal saline 0.9% to infuse over 8 hours. The drop factor is 15 drops (gtt)/1 mL. The nurse sets the flow rate at how many drops per minute? Record your answer to the nearest whole number.

31 drops

On entering a room the nurse sees the patient crying softly. What is the most therapeutic response? 1. Using silence 2. Asking, "Why are you crying today?" 3. Using therapeutic touch 4. Stating, "I see that you're crying."

4. Stating, "I see that you're crying."

A health care provider prescribes 1000 mL D5W to infuse at a rate of 125 mL/hour. The nurse determines that it will take how many hours for 1 L to infuse?

8 hours

Nursing Code of Ethics

A collective statement about the group's expectations and standards of behavior. The American Nurses Association (ANA) established. Advocacy Responsibility Accountability Confidentiality Social Networking

living will

A document that indicates what medical intervention an individual wants if he or she becomes incapable of expressing those wishes.

The nurse is caring for a client with a diagnosis of cancer who is immunosuppressed. the nurse would consider implementing neutropenic precautions if the client's white blood cell count was which value? A. 2000 mm^3 B. 5800 mm^3 C. 8400 mm^3 D. 11,500 mm^3

A. 2000 mm^3

A client with a history of cardiac disease is due for a morning dose for furosemide. Which serum potassium level, if noted in the client's laboratory report, should be reported before administering the dose of furosemide? A. 3.2 mEq/L (3.2 mmol/L) B. 3.8 mEq/L (3.8 mmol/L) C. 4.2 mEq/L (4.2 mmol/L) D. 4.8 mEq/L (4.8 mmol/L)

A. 3.2 mEq/L (3.2 mmol/L)

The nurse is instructing a client how to perform a testicular self-examination (TSE). The nurse should explain that which is the *best* time to perform this exam? A. After a shower or bath B. while standing to void C. After having a bowel movement D. While lying in bed before arising

A. After a shower or bath

The nurse receives a telephone call from the postanesthesia care unit stating that a client is being transferred to the surgical unit. The nurse plans to take which action *first* on arrival of the client A. Assess the patency of the airway B. Check tubes or drains for patency C. Check the dressing to assess for bleeding D. Assess the vital signs to compare with preoperative measurements

A. Assess the patency of the airway

Which of the following might be a cause of stress for young adults? Select all that apply a. Being single b. Career c. Sexuality d. Activity

A. B. C

A 45-year-old woman who is obese tells a nurse that she wants to lose weight. After conducting a thorough assessment, the nurse concludes that which of the following may be contributing factors to the woman's obesity? Select all that apply. A. The woman works in an executive position that is very demanding. B. The woman works out at the corporate gym at 5 am two mornings per week. C. The woman says that she has little time to prepare meals at home and eats out at least four nights a week. D. The woman says that she tries to eat "low-cholesterol" foods to help lose weight. E. The woman says that she vacations annually to reduce stress.

A. C. D.

Intimate partner violence (IPV) is linked to which of the following factors? Select all that apply. A. Alcohol abuse B. Marriage C. Pregnancy D. Unemployment E. Drug use

A. C. D. E.

A client who has had abdominal surgery complains of feeling as though "something gave way" in the incisional site. The nurse removes the dressing and notes the presence of a loop of bowel protruding through the incision. Which interventions should the nurse take? *Select all that apply.* A. Contact the surgeon B. Instruct the client to remain quiet C. Prepare the client for wound closure D. Document the findings and actions taken E. Place a sterile saline dressing and ice packs over the wound F. Place the client in a supine position without a pillow under the head.

A. Contact the surgeon B. Instruct the client to remain quiet C. Prepare the client for wound closure D. Document the findings and actions taken

The nurse is providing discharge instructions to a Chinese American client regarding prescribed dietary modifications. During the teaching session, the client continuously turns away from the nurse. The nurse should implement which *best* action? A. Continue with the instructions, verifying client understanding B. Walk around the client so that the nurse constantly faces the client C. Give the client a dietary booklet and return later to continue with the instructions D. Tell the client about the importance of the instructions for the maintenance of health care

A. Continue with the instructions, verifying client understanding

A client who has undergone preadmission testing has had blood drawn for serum laboratory studies, including a complete blood count, coagulation studies, and electrolytes and creatinine levels. Which laboratory result should be reported to the surgeon's office by the nurse, knowing that it could cause surgery to be postponed? A. Hemoglobin, 8.0 g/dL (80 mmol/L) B. Sodium, 145 mEq/L (145 mmol/L) C. Serum creatinine, 0.8 mg/dL (70.6 umol/L) D. platelets, 210,000 cells/mm^3 (210 x 10^3/uL/210 x 10^9/L)

A. Hemoglobin, 8.0 g/dL (80 mmol/L)

A burn client is receiving treatments of topical mafenide acetate to the site of injury. The nurse monitors the client, knowing that which finding indicates that a systemic effect has occurred? A. Hyperventilation B. Elevated blood pressure C. Local rash at burn site D. Local pain at burn site

A. Hyperventilation Rationale: mafenide acetate is a Carbonic anhydrase inhibitor and can suppress renal excretion of acid, thereby causing acidosis. Clients receiving this treatment should be monitored for signs of an acid base imbalance (hyperventilation). if this occurs, the medication will probably be discontinued for one to two days. Options three and four describe local rather than systemic effects. An elevated blood pressure may be expected from the pain that occurs with a burn injury.

Which clients have a high risk of obesity and diabetes mellitus? *Select all that apply.* A. Latino American man B. Native American man C. Asian American woman D. Hispanic American man E. African American woman

A. Latino American man B. Native American man D. Hispanic American man E. African American woman

The nurse is monitoring a client admitted to the hospital with a diagnosis of appendicitis who is scheduled for surgery in 2 hours. The client begins to complain of increased abdominal pain and begins to vomit. On assessment, the nurse notes that the abdomen is distended and bowel sounds are diminished. Which is the *most appropriate* nursing intervention? A. Notify the health care provider (HCP) B. Administer the prescribed pain medication C. Call and ask the operating room team to perform surgery as soon as possible D. Reposition the client and apply a heating pad on the warm setting to the client's abdomen

A. Notify the health care provider (HCP) Rationale: On the basis of the signs and symptoms presented in the question, the nurse should suspect peritonitis and notify the HCP. Administering pain medication is not an appropriate intervention. Heat should never be applied to the abdomen of a client with suspected appendicitis because of the risk of rupture. Scheduling surgical time is not within the scope of nursing practice, although the HCP probably would perform the surgery earlier than the prescheduled time.

A client with parenteral nutrition (PN) infusing has disconnected the tubing from the central line catheter. The nurse assesses for the client and suspects an air embolism. The nurse should *immediately* place the client in which position? A. On the left side, with the head lower than the feet B. On the left side, with the head higher than the feet C. On the right side, with the head lower than the feet D. On the right side, with the head higher than the feet

A. On the left side, with the head lower than the feet

The nurse is reviewing a surgeon's prescription sheet for a preoperative client that states that the client must be nothing by mouth (NPO) after midnight. The nurse should call the surgeon to clarify that which medication should be given to the client and not withheld? A. Prednisone B. Ferrous sulfate C. Cyclobenzaprine D. Conjugated estrogen

A. Prednisone

The nurse is caring for a client with meningitis and implements which transmission-based precautions for this client? A. Private room or cohort client B. Personal respiratory protection device C. Private room with negative airflow pressure D. Mask worn by staff when the client needs to leave the room

A. Private room or cohort client

A client with carcinoma of the lung develops syndrome of inappropriate antidiuretic hormone (SIADH) as a complication of the cancer. The nurse anticipates that the health care provider will request which prescriptions? *Select all that apply.* A. Radiation B. Chemotherapy C. Increased fluid intake D. Decreased oral sodium intake E. Serum sodium level determination F. Medication that is antagonistic to antidiuretic hormone

A. Radiation B. Chemotherapy E. Serum sodium level determination F. Medication that is antagonistic to antidiuretic hormone

The nurse notes documentation that a client is exhibiting Cheyne-Stokes respirations. On assessment of the client, the nurse should expect to note which finding? A. Rhythmic respirations with periods of apnea B. Regular rapid and deep, sustained respirations C. Totally irregular respiration in rhythm and depth D. Irregular respirations with pauses at the end of inspiration and expiration

A. Rhythmic respirations with periods of apnea

Which client is at risk for the development of a sodium level at 130 mEq/L (130 mmol/L)? A. The client who is taking diuretics B. The client with hyperaldosteronism C. The client with Cushing's syndrome D. The client who is taking corticosteroids

A. The client who is taking diuretics

The nurse is assessing the colostomy of a client who has had an abdominal perineal resection for a bowel tumor. Which assessment finding indicates that the colostomy is beginning to function? A. The passage of flatus B. Absent bowel sounds C. The client's ability to tolerate food D. Bloody drainage from the colostomy

A. The passage of flatus

The nurse is caring for a client who takes ibuprofen for pain. the nurse is gathering information on the client's medication history, and determines it is necessary to contact the health care provider (HCP) if the client is also taking which medications? *Select all that apply.) A. Warfarin B. Glimepiride C. Amlodipine D. Simvastatin E. Hydrochlorothiazide

A. Warfarin B. Glimepiride C. Amlodipine

The nurse reviews the blood gas results of a client with atelectasis. The nurse analyzes the results and determines that the client is experiencing respiratory acidosis. Which result validates the nurse's findings? A. pH 7.25, PaCO2 50 mmHg B. pH 7.35, PaCO2 40 mmHg C. pH 7.50, PaCO2 52 mmHg D. pH 7.52, PaCO2 28 mmHg

A. pH 7.25, PaCO2 50 mmHg

Match the following definitions with the key terms related to intersectionality. Term Definition A. Groups have unequal access to resources, services, and positions. B. A group has been overlooked in research and the design of interventions. C. One's place in society is based on membership in a social group that determines access to resources. Definitions 1. Under inclusion 2. Social inequality 3. Social location

A2 B1 C3

7. A nurse is caring for a patient with stress and is in the evaluation stage of the critical thinking model. Which actions will the nurse take? a. Select nursing interventions and promote patient's adaptation to stress. b. Establish short- and long-term goals with the patient experiencing stress. c. Identify stress management interventions and achieve expected outcomes. d. Reassess patient's stress-related symptoms and compare with expected outcomes.

ANS: D During the evaluation stage, the nurse compares current stress-related symptoms against established measurable outcomes to evaluate the effectiveness of the intervention. Selecting appropriate interventions and establishing goals are part of the planning process.

4. Which statement indicates that the nurse has a good understanding of teaching/learning? a. "Teaching and learning can be separated." b. "Learning is an interactive process that promotes teaching." c. "Learning consists of a conscious, deliberate set of actions designed to help the teacher." d. "Teaching is most effective when it responds to the learner's needs."

ANS: D Teaching is most effective when it responds to the learner's needs. It is impossible to separate teaching from learning. Teaching is an interactive process that promotes learning. Teaching consists of a conscious, deliberate set of actions that help individuals gain new knowledge, change attitudes, adopt new behaviors, or perform new skills.

Nursing Diagnosis Format (example)

AT RISK OF ________ (ex INJURY), RELATED TO ___________ (ex DIZZINESS), AS EVIDENCED BY ____________ (ex HISTORY OF PREVIOUS FALLS). - use of BOTH subjective and objective data help to form nursing diagnosis.

A client is receiving Heparin Sodium 25,000 Units in 5% Dextrose Injection 500 ml IV at 1,000 unit/hour per protocol for acute coronary syndrome (ACS). The client's partial thromboplastin time (PTT) is 76 seconds. Based on the ACS protocol, the infusion should be decreased by 100 units/hour for a PTT between 71 to 80 seconds. The nurse should program the pump to deliver how many ml/hour? (Enter numeric value only.)

Adjust the infusion to 900 units/hour (1,000 units/hour minus 100 units/hour per protocol). Using the formula, D/H x Q, 900 units/1 hour divided by 25,000 units x 500 ml = 90/5= 18 ml/hour.

A child who is scheduled for a kidney transplant receives a prescription for basiliximab (Simulect) 20 mg IV 2 hours prior to surgery. The medication is available in a 20 mg vial that is reconstituted by adding 5 ml sterile water for injection, and administered in a 50 ml bag of normal saline over 30 minutes. The nurse should program the infusion pump to deliver how many ml/hour? (Enter the numeric value only.)

After reconstituting the medication vial, the nurse adds the 5 ml of medication to the 50 ml of sterile water to result in a 55 ml volume to infuse in 30 minutes. Using the formula, Volume/Time = 55 ml / 0.5 hours = 110 ml/hour

Which of the following are major public health problems commonly affecting older adults? (Select all that apply.) 1. Substance abuse 2. Dementia 3. Financial limitations 4. Communicable diseases 5. Chronic physical illnesses

Answer: 1, 2, 3, 5. The older-adult population frequently has restricted or fixed income levels and a higher percentage of chronic illnesses. Managing these illnesses further depletes the financial resources for the older-adult population, further increasing their risks for health problems.

A patient is admitted to a medical unit. The patient is fearful of hospitals. The nurse carefully assesses the patient to determine the exact fears and then establishes interventions designed to reduce these fears. In this setting, how is the nurse practicing patient advocacy? 1. Seeking out the nursing supervisor to talk with your patient 2. Documenting patient fears in the medical record in timely manner 3. Working to change the hospital environment 4. Assessing the patient's point of view and preparing to articulate it

Assessing the patient's point of view and preparing to articulate it

Which activity performed by the nurse is related to maintaining competency in nursing practice? A). Asking another nurse about how to change the settings on a medication pump B). Regularly attending unit staff meetings C). Participating as a member of the professional nursing council D). Attending a review course in preparation for the certification examination

Attending a review course in preparation for a certification examination - Maintaining ongoing competency is a nurse's responsibility. Earning certification in a specialty area is one mechanism that demonstrates competency. Specialty certification has been shown to be positively related to patient safety.

The nurse provides a list of instructions to a client being discharged to home with a peripherally inserted central catheter (PICC). The nurse determines that the client *needs further instructions* if the client made which statement? A. "I need to wear a MedicAlert tag or bracelet" B. "I need to restrict my activity while this catheter is in place" C. "I need to keep the insertion site protected when in the shower or bath D. "I need to check the markings on the catheter each time the dressing is changed

B. "I need to restrict my activity while this catheter is in place"

A 55-year-old male client confides in the nurse that he is concerned about his sexual function. What is the nurse's *best* response? A. "How often do you have sexual relations?" B. "Please share with me more about your concerns." C. "You are still young and have nothing to be concerned about." D. "You should not have a decline in testosterone until you are in your 80s."

B. "Please share with me more about your concerns."

The emergency department (ED) nurse receives a telephone call and is informed that a tornado has hit a local residential area and that numerous casualties have occurred. The victims will be brought to the ED. The nurse should take which *initial* action? A. Prepare the triage rooms B. Activate the emergency response plan C. Obtain additional supplies from the central supply department D. Obtain additional nursing staff to assist in treating the casualties

B. Activate the emergency response plan

The nurse is assessing the perineal wound in a client who has returned from the operating room following an abdominal perineal resection and notes serosanguineous drainage form the wound. Which nursing intervention is *most appropriate?* A. Clamp the surgical drain B. Change the dressing as prescribed C. Notify the health care provider (HCP) D. Remove and replace the perineal packing

B. Change the dressing as prescribed

The nurse has a prescription to hang a 1000-mL intravenous (IV) bag of 5% dextrose in water with 20 mEq of potassium chloride. The nurse also needs to hang an IV infusion of piperacillin/tazobactam. The client has one IV site. The nurse should plan to take which action *first?* A. Start a second IV site B. Check compatibility of the medication and IV fluids C. Mix the prepackaged piperacillin/tazobactam per agency policy D. Prime the tubing with the IV solution, and back prime the medication

B. Check compatibility of the medication and IV fluids

A client is being weaned from parenteral nutrition (PN) and is expected to begin taking solid food today. The ongoing solution rate has been 100 mL/hour. The nurse anticipates that which prescription regarding the PN solution will accompany the diet prescription? A. Discontinue the PN B. Decrease PN rate to 50 mL/hour C. Start 0.9% normal saline at 25 mL/hour D. Continue current infusion rate prescriptions for PN

B. Decrease PN rate to 50 mL/hour

The nurse is caring for a client following a mastectomy. Which nursing intervention would assist in preventing lymphedema of the affected arm? A, Placing cool compresses on the affected arm B. Elevating the affected arm on a pillow above heart level C. Avoiding arm exercises in the immediate postoperative period D. Maintaining an intravenous site below the antecubital area on the affected side

B. Elevating the affected arm on a pillow above heart level

The nurse is caring for a client who suffered an inhalation injury from a wood stove. The carbon monoxide blood report reveals a level of 12%. Based on this level, the nurse would anticipate noting which sign in the client? A. Coma B. Flushing C. Dizziness D. Tachycardia

B. Flushing

When caring for a client with an internal radiation implant, the nurse should observe which principles? *Select all that apply.* A. Limiting the time with the client to 1 hour per shift B. Keeping pregnant women out of the client's room C. Placing the client in a private room with a private bath D. Wearing a lead shield when providing direct client care E. Removing the dosimeter film badge when entering the client's room

B. Keeping pregnant women out of the client's room C. Placing the client in a private room with a private bath D. Wearing a lead shield when providing direct client care Rationale: The time that the nurse spends in the room of a client with an internal radiation implant is 30 minutes per 8-hour shift. The client must be placed in a private room with a private baht. Lead shielding can be used to reduce the transmission of radiation. The dosimeter film badge must be worn when in the client's room. Children younger than 16 years of age and pregnant women are not allowed in the client's room.

The nurse is caring for a client with a nasogastric tube that is attached to low suction. The nurse monitors the client for manifestations of which disorder that the client is at risk for? A. Metabolic acidosis B. Metabolic alkalosis C. Respiratory acidosis D. Respiratory alkalosis

B. Metabolic alkalosis

The nurse manager is discussing the facility protocol in the event of a tornado with the staff. Which instructions should the nurse manager include in the discussion? *Select all that apply.* A. Open doors to client rooms B. Move beds away from windows C. Close window shades and curtains D. Place blankets over clients who are confined to bed E. Relocate ambulatory clients from the hallways back into their rooms

B. Move beds away from windows C. Close window shades and curtains D. Place blankets over clients who are confined to bed

The nurse is preparing to hang fat emulsion (lipids) and notes that fat globules are visible at the top of the solution. The nurse should take which action? A. Roll the bottle of solution gently B. Obtain a different bottle of solution C. Shake the bottle of solution vigorously D. Run the bottle of solution under warm water

B. Obtain a different bottle of solution

The nurse is preparing a continuous intravenous (IV) infusion at the medication cart. As the nurse goes to insert the spike end of the IV tubing into the IV bag, the tubing drops and the spike end hits the top of the medication cart. The nurse should take which action? A. Obtain a new IV bag B. Obtain new IV tubing C. Wipe the spike end of the tubing with povidone iodine D. Scrub the spike end of the tubing with an alcohol swab

B. Obtain new IV tubing

A client is recovering from abdominal surgery and has a large abdominal wound. The nurse should encourage the client to eat which food item that is naturally high in vitamin C to promote wound healing? A. Milk B. Oranges C. Bananas D. Chicken

B. Oranges

The nurse is caring for a client who is on a mechanical ventilator. Blood gas results indicate a pH of 7.50 and a PaCO2 of 30 mmHg. The nurse has determined that the client is experiencing respiratory alkalosis. Which laboratory value would *most likely* be noted in this condition? A. Sodium level of 145 mEq/L (145 mmol/L) B. Potassium level of 3.0 mEq/L (3.0 mmol/L) C. Magnesium level of 1.3 mEq/L (0.65 mmol/L) D. Phosphorus level of 3.0 mEq/dL (0.97 mmol/L)

B. Potassium level of 3.0 mEq/L (3.0 mmol/L)

The nurse provides instructions to a client with a low potassium level about the foods that are high in potassium and tells the client to consume which foods? *Select all that apply.* A. Peas B. Raisins C. Potatoes D. Cantaloupe E. Cauliflower F. Strawberries

B. Raisins C. Potatoes D. Cantaloupe F. Strawberries

The nurse is preparing to care for a client with a potassium deficit. The nurse reviews the client's record and determines that the client is a risk for developing the potassium deficit because of which situation? A. Sustained tissue damage B. Requires nasogastric suction C. Has a history of Addison's disease D. Uric acid level of 9.4 mg/dL (559 umol/L)

B. Requires nasogastric suction

The nurse is caring for a client with diabetic ketoacidosis and documents that the client is experiencing Kussmaul's respirations. Which patterns did the nurse observe? *Select all that apply.* A. Respirations that are shallow B. Respirations that are increased in rate C. Respirations that are abnormally slow D. Respirations that are abnormally deep E. Respirations that cease for several seconds

B. Respirations that are increased in rate D. Respirations that are abnormally deep

The nurse reviews the arterial blood gas results of a client and notes the following: pH 7.45, PaCO2 of 30 mmHg (30 mmHg), and HCO3- of 20 mEq/L (20 mmol/L). The nurse analyzes these results as indicating which condition? A. Metabolic acidosis, compensated B. Respiratory alkalosis, compensated C. Metabolic alkalosis, uncompensated D. Respiratory acidosis, uncompensated

B. Respiratory alkalosis, compensated

The nurse assesses a client's surgical incision for signs of infection. Which finding by the nurse would be interpreted as a normal finding at the surgical site? A. Red, hard skin B. Serous drainage C. Purulent drainage D. Warm, tender skin

B. Serous drainage

A nurse is planning care for a patient going to surgery. Who is responsible for informing the patient about the surgery along with possible risks, complications, and benefits? A. Family member B. Surgeon C. Nurse D. Nurse manager

B. Surgeon The person performing the procedure is responsible for informing the patient about the procedure and its risks, benefits, and possible complications.

A client has been discharged to home on parenteral nutrition (PN). With each visit, the home care nurse should assess which parameter *most* closely in monitoring this therapy? A. Pulse and weight B. Temperature and weight C. Pulse and blood pressure D. Temperature and blood pressure

B. Temperature and weight

The nurse is testing the extra ocular movements in a client to assess for muscle weakness in the eyes. The nurse should implement which assessment technique which assessment technique to assess for muscle weakness in the eye? A. Test the corneal reflexes B. Test the 6 cardinal positions of gaze C. Test visual acuity, using a Snellen eye chart D. Test sensory function by asking the client to close the eyes and then lightly touching the forehead, cheeks, and chin

B. Test the 6 cardinal positions of gaze

A client with acute myelocytic leukemia is being treated with busulfan (Myleran, Busulfex). Which laboratory value would the nurse specifically monitor during treatment with this medication? A. Clotting time B. Uric acid level C. Potassium level D. Blood glucose level

B. Uric acid level Rationale: Busulfan (Myleran, Busulfex) can cause an increase in the uric acid level. Hyperuricemia can produce uric acid nephropathy, renal stones, and acute kidney injury. Options 1, 3, and 4 are not specifically related to this medication.

The nurse is administering fluids intravenously as prescribed to a client who sustained superficial partial-thickness burn injuries to the back and legs. In elevating the adequacy of fluid resuscitation, the nurse understands that which assessment would provide the *most* reliable indicator for determining the adequacy? A. Vital signs B. Urine output C. Mental status D. Peripheral pulses

B. Urine output

The nurse is conducting a dietary assessment on a client who is on a vegan diet. The nurse provides dietary teaching and should focus on foods high in which vitamin and may be lacking in a vegan diet? A. Vitamin A B. Vitamin B12 C. Vitamin C D. Vitamin E

B. Vitamin B12

The nurses on a medical unit have seen an increase in the number of pressure ulcers that develop in their patients. They decide to initiate a quality improvement project using the Plan-Do-Study-Act (PDSA) model. Which of the following is an example of "Do" from that model? A). Implementing the new skin care protocol on all medicine units. B). Reviewing the data collected on patients cared for using the protocol. C). Reviewing the quality improvement reports on the six patients who developed ulcers over the last 3 months. D). Based on findings from patients who developed ulcers, implementing an evidence-based skin care protocol.

Based on findings from patients who developed ulcers, implementing an evidence-based skin care protocol

Certification

Beyond NCLEX-RN; exam in nursing specialty; minimum practice requirements are set depending in the certification; include years required working in specialty area. National organization such as ANA

When a nurse tries to understand a patient's and family caregiver's perspective of why a patient is falling at home, the nurse applies the intellectual standard of ________ to understand all viewpoints.

Broad

As part of chemotherapy education, the nurse teaches a female client about the risk for bleeding and self-care during the period of greatest bone marrow suppression (the nadir). The nurse understands that *further teaching is needed* if the client makes which statement? A. "I should avoid blowing my nose." B. "I may need a platelet transfusion if my platelet count is too low." C. "I'm going to take aspirin for my headache as soon as I get home." D. "I will count the number of pads and tampons I use when menstruating."

C. "I'm going to take aspirin for my headache as soon as I get home."

Silver sulfadiazine is prescribed for a client with a partial thickness burn and the nurse provides teaching about the medication. Which statement made by the client indicates a *need for further teaching* about the treatments? A. "The medication is an antibacterial." B. "The medication will help heal the burn." C. "The medication is likely to cause stinging every time it is applied." D. "The medication should be applied directly to the wound."

C. "The medication is likely to cause stinging every time it is applied." Rationale: Silver sulfadiazine is an antibacterial that has a broad spectrum of activity against gram-negative bacteria gram positive bacteria, and yeast. It is applied directly to the wound to assistant healing. It does not cause stinging when applied.

Tamoxifen citrate is prescribed for a client with metastatic breast carcinoma. The client asks the nurse if her family member with bladder cancer can also take this medication. The nurse *most appropriately* responds by making which statement? A. "This medication can be used only to treat breast cancer." B. "Yes, your family member can take this medication for bladder cancer as well." C. "This medication can be taken to prevent and treat clients with breast cancer." D. "This medication can be taken by anyone with cancer as long as their health care provider approves it."

C. "This medication can be taken to prevent and treat clients with breast cancer." Rationale: Tamoxifen is an antineoplastic medication that competes with estradiol for binding to estrogen in tissues containing high concentrations of receptors. Tamoxifen is used to treat metastatic breast carcinoma in women and men. Tamoxifen is also effective in delaying the recurrence of cancer following mastectomy. Tamoxifen reduces DNA synthesis and estrogen response.

While performing a cardiac assessment on a client with an incompetent heart valve, the nurse auscultates a murmur. The nurse documents the finding and describes the sound as which? A. Lub-dub sounds B. Scratchy, leathery heart noise C. A blowing or swooshing noise D. Abrupt, high-pitched snapping noise

C. A blowing or swooshing noise

The registered nurse is planning the client assignments for the day. Which is the *most appropriate* assignment for an unlicensed assistive personnel (UAP)? A. A client requiring a colostomy irrigation B. A client receiving continuous tube feedings C. A client who requires urine specimen collections D. A client with difficulty swallowing food and fluids

C. A client who requires urine specimen collections

A client diagnosed with conductive hearing loss asks the nurse to explain the cause of the hearing problem. The nurse plans to explain to the client that this condition is caused by which problem? A. A defect in the cochlea B. A defect in cranial nerve VIII C. A physical obstruction to the transmission of sound waves D. A defect in the sensory fibers that lead to the cerebral cortex

C. A physical obstruction to the transmission of sound waves

The nurse enters a client's room and finds that the wastebasket is on fire. The nurse immediately assists the client out of the room. What is the *next* nursing action? A. Call for help B. Extinguish the fire C. Activate the fire alarm D. Confine the fire by closing the room door

C. Activate the fire alarm

A mother calls a neighbor who is a nurse and tells the nurse that her 3-year-old child has just ingested liquid furniture polish. The nurse would direct the mother to take which *immediate* action? A. Induce vomiting B. Call an ambulance C. Call the Poison Control Center D. Bring the child to the emergency department

C. Call the Poison Control Center

The nurse who works on the night shift enters the medication room and finds a co-worker with a tourniquet wrapped around the upper arm. The co-worker is about to insert a needle, attached to a syringe containing a clear liquid, into the antecubital area. Which is the *most appropriate* action by the nurse? A. Call security B. Call the police C. Call the nursing supervisor D. Lock the co-worker in the medication room until help is obtained

C. Call the nursing supervisor

The nurse is creating a plan of care for a client scheduled for surgery. The nurse should include which activity in the nursing care plan for the client on the day of Surgery? A. Avoid oral hygiene and rinsing with mouthwash B. Verify that the client has not eaten for the last 24 hrs C. Have the client void immediately before going into surgery D. Report immediately any slight increase in blood pressure or pulse

C. Have the client void immediately before going into surgery

The nurse is reading a health care provider's (HCP's) progress notes in the client's record and reads that the HCP has documented "insensible fluid loss of approximately 800 mL daily." The nurse makes notation that insensible fluid loss occurs through which type of excretion? A. Urinary output B. Wound drainage C. Integumentary output D. The gastrointestinal tract

C. Integumentary output

The nurse working in the emergency department (ED) is assessing a client who recently returned from Liberia and presented complaining of a fever at home, fatigue, muscle pain, and abdominal pain. Which action should the nurse take *next?* A. Check the client's temperature B. Contact the health care provider C. Isolate the client in a private room D. Check a complete set of vital signs

C. Isolate the client in a private room

Which purposes of placental functioning should the nurse include in a prenatal class? *Select all that apply.* A. It cushions and protects the baby B. It maintains the temperature of the baby C. It is the way the baby gets food and oxygen D. It prevents all antibodies and viruses from passing to the baby E. It provides an exchange of nutrients and waste products between the mother and developing fetus

C. It is the way the baby gets food and oxygen E. It provides an exchange of nutrients and waste products between the mother and developing fetus

The nurse is performing an assessment on a client who is at 38 weeks' gestation and notes that the fetal heart rate (FHR) is 174 beats/minute. On the basis of this finding, what is the *priority* nursing action? A. Document the finding B. Check the mother's heart rate C. Notify the health care provider (HCP) D. Tell the client that the fetal heart rate is normal

C. Notify the health care provider (HCP)

The nurse is monitoring a client for signs and symptoms related to superior vena cava syndrome. Which is an *early* sign of this oncological emergency? A. Cyanosis B. Arm edema C. Periorbital edema D. Mental status changes

C. Periorbital edema

The nurse manager is planning the clinical assignments for the day. Which staff members can be assigned to care for a client with herpes zoster? *Select all that apply.* A. The nurse who never had roseola B. The nurse who never had mumps C. The nurse who never had chickenpox D. The nurse who never had German measles E. The nurse who never received the varicella zoster vaccine

C. The nurse who never had chickenpox* E. The nurse who never received the varicella zoster vaccine

A child who weighs 55 pounds receives a prescription for atovaquone with proguanil (Malarone Pediatric) 125 mg/50 mg PO daily. A drug reference states that children 11 to 20 kg should receive 1 pediatric tablet daily; 21 to 30 kg should receive 2 pediatric tablets daily; 31 to 40 kg should receive 3 pediatric tablets daily; and greater than 40 kg should receive 1 adult tablet daily with food. The drug is available as atovaquone 62.5 mg/proguanil 25 mg pediatric tablets. How many tablets should the nurse administer?

Calculate each drug component dose, using the formula, D/H 125 mg / 62.5 mg (atovaquone) = 2 combined with 50 mg / 25 mg (proguanil) = 2. The child should receive 2 tablets.

A client receives a prescription for an intravenous infusion 0.45% sodium chloride 500 ml to be infused over 6 hours. The nurse should program the infusion pump to deliver how many ml/hour? (Enter numeric value only. If rounding is required, round to the nearest whole number.)

Calculate using Volume/Time: 500 ml/6 hours = 83.3 ml/hour

Sexually transmitted infections (STIs) continue to be a major health problem in young adults. Men ages 20 to 24 years have the highest rate of which STI? Chlamydia Syphilis Gonorrhea Herpes zoster

Chlamydia

Sources of Diagnostic Errors

Collecting data, data clustering, labeling data, interpreting/analyzing data

The nurse notes that a client is receiving an oxytocin (Pitocin) infusion via a pump that is programmed to deliver 30 ml/hour. The available solution is Ringer's Lactated 1,000 ml with Pitocin 20 units. How many milliunits/minute is the client receiving? (Enter numeric value only.)

Convert units to milliunits, 20 x 1,000 = 20,000 units. Using D/H x Q = 30 ml/hour X/20,000 units x 1,000 ml = 30 ml/hour (60 minutes) X/20 =30/60, reduce X/2 = 1/2, and 2X = 20, so X = 10 milliunits/minute OR 20/1,000 = 0.02 1000 : 0.02 :: 30 : X = 0.6 :: 1,000X X = 600 and 600/60 = 10 milliunits/minute.

Which of the following are examples of collaborative problems? (Select all that apply.) A) Nausea B) Hemorrhage C) Wound infection D) Fear

Correct Answer(s): B, C Hemorrhage and wound infection are collaborative problems, actual or potential physiological complications. Nurses typically monitor for these to detect changes in a patient's status. Nausea and fear are both NANDA-I approved nursing diagnoses.

The nursing diagnosis readiness for enhanced communication is an example of a(n): A) Risk nursing diagnosis. B) Actual nursing diagnosis. C) Health promotion nursing diagnosis D) Wellness nursing diagnosis.

Correct Answer(s): C A patient's readiness for enhanced communication is an example of a health-promotion diagnosis because it implies the patient's motivation and desire to strengthen his health.

A nurse is talking with a patient who is visiting a neighborhood health clinic. The patient came to the clinic for repeated symptoms of a sinus infection. During their discussion the nurse checks the patient's medical record and realizes that he is due for a tetanus shot. Administering the shot is an example of what type of preventive intervention? A) Tertiary B) Direct care C) Primary D) Secondary

Correct Answer(s): C An immunization is an example of a primary prevention aimed at health promotion.

Which of the following might be a cause of stress for the middle-aged adult? a. Financial security b. Planned retirement c. Arrival of grandchildren d. Caring for children and aging parents

D

An 87-year-old woman is brought to the emergency department for treatment of a fractured arm. On physical assessment, the nurse notes old and new ecchymotic areas on the client's chest and legs and asks how the bruises were sustained. The client, although reluctant, tells the nurse in confidence that her son frequently hits her if supper is not prepared on time when he arrives home from work. Which is the *most appropriate* nursing response? A. "Oh, really? I will discuss this station with your son." B. "Let's talk about the ways you can manage your time to prevent this from happening." C. "Do you have any friends who can help you out until you resolve these important issues with your son?" D. "As a nurse, I am legally bound to report abuse. I will stay with you while you give the report and help find a safe place for you to stay."

D. "As a nurse, I am legally bound to report abuse. I will stay with you while you give the report and help find a safe place for you to stay."

The nurse is preparing a plan of care for a client, and is asking the client about religious preferences. The nurse considers the client's religious preferences as being characteristic of a Jehovah's Witness if which client statement is made? A. "I cannot have surgery" B. "I cannot have any medicine" C. "I believe the soul lives on after death" D. "I cannot have any food containing or prepared with blood

D. "I cannot have any food containing or prepared with blood

A staff nurse is precasting a new graduate nurse and the new graduate is assigned to care for a client with chronic pain. Which statement, if made by the new graduate nurse, indicates the *need for further teaching* regarding pain management? A. "I will be sure to ask my client what his pain level is on a scale of 0 to 10." B. "I know that I should follow up after giving medication to make sure it is effective." C. "I know that pain in the older client might manifest as sleep disturbances or depression." D. "I will be sure to cue in to any indicators that the client may be exaggerating their pain."

D. "I will be sure to cue in to any indicators that the client may be exaggerating their pain."

The nurse is assigned to care for four clients. In planning client rounds, which client should the nurse assess *first*? A. A postoperative client preparing for discharge with a new medication B. A client requiring daily dressing changes of a recent surgical incision C. A client scheduled for a chest x-ray after insertion of a nasogastric tube D. A client with asthma who requested a breathing treatment during the previous shift

D. A client with asthma who requested a breathing treatment during the previous shift

A nursing graduate is attending an agency orientation regarding the nursing model of practice implemented in the health care facility. The nurse is told that the nursing model is a team nursing approach. The nurse determines that which scenario is characteristic of the team-based model of nursing practice? A. Each staff member is assigned a specific task for a group of clients B. A staff member is assigned to determine the client's needs at home and begin discharge planning C. A single registered nurse (RN) is responsible for providing care to a group of 6 clients with the aid of an unlicensed assistive personnel (UAP) D. An RN leads 2 licensed practical nurses (LPNs) and 3 UAPs in providing care to a group of 12 clients

D. An RN leads 2 licensed practical nurses (LPNs) and 3 UAPs in providing care to a group of 12 clients

A client with a 3-day history of nausea and vomiting presents to the emergency department. The client is hyperventilating and has a respiratory rate of 10 breaths/minute. The electrocardiogram (ECG) monitor displays tachycardia, with a heart rate of 120 beats/minute. Arterial blood gases are drawn and the nurse reviews the results, expecting to note which finding? A. A decreased pH and an increased PaCO2 B. An increased pH and a decreased PaCO2 C. A decreased pH and a decreased HCO3- D. An increased pH and an increased HCO3-

D. An increased pH and an increased HCO3-

A client has just undergone insertion of a central venous catheter at the bedside under ultrasound. The nurse would be sure to check which results before initiating the flow rate of the client's intravenous (IV) solution at 100 mL/hour? A. Serum osmolality B. Serum electrolyte levels C. Intake and output record D. Chest radiology results

D. Chest radiology results

Contact precautions are initiated for a client with a health care-associated (nosocomial) infection caused by methicillin-resistant Staphylococcus aureus. The nurse prepares to provide colostomy care and should obtain which protective items to perform this procedure? A. Gloves and gown B. Gloves and goggles C. Gloves, gown, and shoe protectors D. Gloves, gown, goggles, and a mask or face shield

D. Gloves, gown, goggles, and a mask or face shield

A client admitted to the hospital with a suspected diagnosis of acute pancreatitis is being assessed by the nurse. Which assessment findings would be consistent with acute pancreatitis? *Select all that apply.* A. Diarrhea B. Black, tarry stools C. Hyperactive bowel sounds D. Gray-blue color at the flank E. Abdominal guarding and tenderness F. Left upper quadrant pain with radiation to the back

D. Gray-blue color at the flank E. Abdominal guarding and tenderness F. Left upper quadrant pain with radiation to the back

A client is receiving a continuous intravenous infusion of heparin sodium to treat deep vein thrombosis. The client's activated partial thromboplastin time (aPTT) is 65 seconds. The nurse anticipates that which action is needed? A. Discontinuing the heparin infusion B. Increasing the rate of the heparin infusion C. Decreasing the rate of the heparin infusion D. Leaving the rate of the heparin infusion as is

D. Leaving the rate of the heparin infusion as is

The nurse is caring for a client with cirrhosis of the liver. To minimize the effects of the disorder, the nurse teaches the client about foods that are high in thiamine. The nurse determines that the client has the *best* understanding of the dietary measures to follow if the client states an intention to increase the intake of which food? A. Milk B. Chicken C. Broccoli D. Legumes

D. Legumes

The nurse is teaching a client who has iron deficiency anemia about food she should include in the diet. The nurse determines that the client understands the dietary modifications if which items are selected from the menu? A. Nuts and milk B. Coffee and tea C. Cooked rolled oats and fish D. Oranges and dark green leafy vegetables

D. Oranges and dark green leafy vegetables

A client with small cell lung cancer is being treated with etoposide. The nurse monitors the client during administration, knowing that which adverse effect is specifically associated with this medication? A. Alopecia B. Chest pain C. Pulmonary fibrosis D. Orthostatic hypotension

D. Orthostatic hypotension Rationale: An adverse effect specific to etoposide is orthostatic hypotension. Etoposide should be administered slowly over 30 to 60 minutes to avoid hypotension. The client's blood pressure is monitored during the infusion. Hair loss occurs with nearly all the antineoplastic medications. Chest pain and pulmonary fibrosis are unrelated to this medication.

The nurse is reviewing a plan of care for a client with an internal radiation implant. Which intervention if noted in the plan *indicates the need for revision* of the plan? A. Wearing gloves when emptying the client's bedpan B. Keeping all linens in the room until the implant is removed C. Wearing a lead apron when providing direct care to the client D. Placing the client in a semiprivate room at the end of the hallway

D. Placing the client in a semiprivate room at the end of the hallway

A client who is found unresponsive has arterial blood gas drawn and the results indicate the following: pH is 7.12, PaCO2 is 90mmHg, and HCO3- is 22 mEq/L. The nurse interprets the results as indicating which condition? A. Metabolic acidosis with compensation B. Respiratory acidosis with compensation C. Metabolic acidosis without compensation D. Respiratory acidosis without compensation

D. Respiratory acidosis without compensation

The nurse is conducting preoperative teaching with a client about the use of an incentive spirometer. The nurse should include which piece of information in discussions with the client? A. Inhale as rapidly as possible B. Keep a loose seal between the lips and the mouthpiece C. After maximum inspiration, hold the breath for 15 seconds and exhale D. The best results are achieved when sitting up or with the head of the bed elevated 45 to 90 degrees

D. The best results are achieved when sitting up or with the head of the bed elevated 45 to 90 degrees

Which client is at risk for the development of a potassium level of 5.5 mEq/L (5.5 mmol/L)? A. The client with colitis B. The client with Cushing's syndrome C. The client who has been overusing laxatives D. The client who has sustained a traumatic burn

D. The client who has sustained a traumatic burn

The clinic nurse assess the skin of a client with psoriasis after the client has used a new topical treatment for 2 months. The nurse identifies which characteristics as improvement in the manifestations of psoriasis? *Select all that apply.* A. Presence of striae B. Palpable radial pulses C. Absence of any ecchymosis on the extremities D. Thinner and decrease in number of reddish papules E. Scarce amount of silvery-white scaly patches on the arms

D. Thinner and decrease in number of reddish papules E. Scarce amount of silvery-white scaly patches on the arms

The nurse monitors the client receiving parenteral nutrition (PN) for complications of the therapy and should assess the client for which manifestations of hyperglycemia? A. Fever, weak pulse, and thirst B. Nausea, vomiting, and oliguria C. Sweating, chills, and abdominal pain D. Weakness, thirst, and increased urine output

D. Weakness, thirst, and increased urine output

priviledged information

Data related to the treatment and progress of the patient that can be released only when written authorization of the patient or guardian is obtained.

A 50-year-old male patient is seen in the clinic. He tells a nurse that he has recently lost his job and his wife of 26 years has asked for a divorce. He has a flat affect. Family history reveals that his father committed suicide at the age of 53. The nurse assesses for the following: Cardiovascular disease Depression Sexually transmitted infection Iron deficiency anemia

Depression

A client is receiving dantrolene sodium (Dantrium) PO for malignant hyperthermia. The maximum safe dose is 8 mg/kg/day in 4 divided doses. The client currently weighs 48.5 kg. What is the maximum safe dose the nurse should administer? (Enter numeric value only. If rounding is required, round to the nearest whole number.)

Determine the maximum number of mg this client should receive in a day: 48.5 kg x 8 mg/kg/day = 388 mg/day. Next determine how many mg the nurse should deliver each dose: 388 mg/4 doses = 97 mg.

A client with a gastrostomy tube (GT) receives a prescription for Osmolite® 1/2 strength enteral formula at 80 ml/hour. To prepare a 4 hour solution, the nurse should dilute the full-strength formula with how many ml of water? (Enter numeric value only.)

Determine the total volume needed at 80 ml/hour x 4 hours = 320 ml. Use the formula, Desired strength/strength on Hand x Volume = 50/100 x 320 = 160 ml of Osmolite® enteral formula, which must be diluted to half strength. Or use ratio and proportion, Desired strength (1/2 = 1 part : 2 parts) :: Volume of full strength : Total Desire volume 1 : 2 :: X : 320 ml 2X = 320, and X = 160 ml of Osmolite® enteral formula, full strength 320 ml total volume - 160 ml of full-strength formula = 160 ml of water to create 1/2 strength or 50% concentration.

By using known criteria in conducting an assessment such as reviewing with a patient the typical characteristics of pain, a nurse is demonstrating which critical thinking attitude? 1. Curiosity 2. Adequacy 3. Discipline 4. Thinking independently

Discipline

Beneficence

Doing good or causing good to be done; kindly action. One of the four principals of the ethical theory of deontology.

Patient Safety Goals

Establishing a policy for medication safety & reconciliation. This is part of JCAHO: the accrediting agency for healthcare organizations. They establish _____

A single young adult interacts with a group of close friends from college and work. They celebrate birthdays and holidays together. In addition, they help one another through many stressors. These individuals are viewed as: Family. Siblings. Substitute parents. Alternative family structure.

Family

Medicaid

Federal program that provides medical benefits for low-income persons.

A child's immunizations may cause discomfort during administration, but the benefits of protection from disease, both for the individual and society, outweigh the temporary discomforts. Which principle is involved in this situation? 1. Fidelity 2. Beneficence 3. Nonmalficence 4. Respect for autonomy

Fidelity

A client with pericardial effusion has phrenic nerve compression resulting in recurrent hiccups. The healthcare provider prescribes metoclopramide (Reglan) liquid 10 mg PO q 6 hours. Reglan is available as 5 mg/5 ml. A measuring device marked in teaspoons is being used. How many teaspoons should the nurse administer?

First, using the formula, Desired dose/dose on Hand x Quantity of volume on hand (D/H x Q), 10 mg / 5 mg x 5ml = 10 ml. Next using the known conversion of 5 ml = 1 tsp: 5 ml : 1 tsp :: 10 ml : X 5 / 10 : 1 / X 5X = 10 X = 2

HIIPA

HEALTH INSURANCE PORTABILITY AND ACCOUNTABILITY ACT OF 1996 mandates the confidentiality of patient records

A nurse is preparing to teach an older adult who has chronic arthritis how to practice meditation. Which of the following strategies are appropriate? (Select all that apply.)

Have patient identify a quiet room in the home that has minimal interruptions. Suggest use of a quiet fan running in the room. Show the patient how to sit comfortably with the limitation of his arthritis and focus on a prayer.

A 50-year-old woman has elevated serum cholesterol levels that increase her risk for cardiovascular disease. One method to control this risk factor is to identify current diet trends and describe dietary changes to reduce the risk. This nursing activity is a form of: Referral. Counseling. Health education. Stress-management techniques.

Health education

When taking care of patients, a nurse routinely asks if they take any vitamins or herbal medications, encourages family members to bring in music that the patient likes to help the patient relax, and frequently prays with her patients if that is important to them. The nurse is practicing which model?

Holistic

A 44-year-old male patient has just been told that his wife and child were killed in an auto accident while coming to visit him in the hospital. Which of the following statements are defining characteristics that support a nursing diagnosis of Spiritual Distress related to loss of family members? (Select all that apply.)

I have nothing to live for now." "Why would my God do this to me?"

A patient who is recovering from a bilateral amputation of the legs below the knee shows transcendence when she states:

I see God's grace and become relaxed when I watch the sun set at night."

a patient in the ER has developed wheezing and shortness of breath. the nurse gives the ordered medicated nebulizer treatment now and in 4 hours. which standard of practice is performed? - Planning - Evaluation - Assessment - Implementation

Implementation

nurses in an acute care hospital are attending a unit based education program to learn how to use a new pressure relieving device for patients at risk for pressure ulcers this is what type of education? - Continuing education - Graduate education - In-service education - Professional Registered Nurse Education

In-service education

a critical care nurse is using a computerized decision support system to correctly position her ventilated patients to reduce pneumonia caused by accumulated respiratory secretions. this is an example of what QSEN competency - Patient-centered care - Safety - Teamwork and collaboration - Informatics

Informatics

A nurse is completing an assessment on a male patient, age 24. Following the assessment, the nurse notes that his physical and laboratory findings are within normal limits. Because of these findings, nursing interventions are directed toward activities related to: Instructing him to return in 2 years. Instructing him in secondary prevention. Instructing him in health promotion activities. Implementing primary prevention with vaccines.

Instructing him in health promotion activities

Tort Law

Involving an act that brings harm to a person or damage to property

JCAHO

Joint Commission on Accreditation of Healthcare Organizations

Which of the following are symptoms of secondary traumatic stress and burnout that commonly affect nurses?

Lack of interest in exercise; Difficulty falling asleep; Lack of desire to go to work; Anxiety while working

The nurses assesses the following risk factors for coronary artery disease (CAD) in a female patient. Which factors are classified as genetic and physiological?

Mother died from CAD at age 48; History of hypertension; Elevated cholesterol level

When determining the amount of information that a patient needs to make decisions about the prescribed course of therapy, many factors affect his or her compliance with the regimen, including educational level and socioeconomic factors. Which additional factor affects compliance? Gender Lifestyle Motivation Family history

Motivation

Medicare

National Health Insurance program for the elderly (65) and disabled.

A nurse is caring for a patient with a seriously advanced infection who asks to have a spiritual care provider come who can offer Blessingway, a practice that attempts to remove ill health. This patient is likely a member of which religion or culture?

Navajo

A student nurse is developing a plan of care for a 74-year-old female patient who has spiritual distress over losing a spouse. As the nurse develops appropriate interventions, which characteristics of older adults should be considered? (Select all that apply.)

Older adults achieve spiritual resilience through frequent expressions of gratitude. Correct Have the patient determine if her husband left a legacy behind. Correct Offer the patient her choice of rituals or participation in exercise. Correct

Two patient deaths have occurred on a medical unit in the last month. The staff notices that everyone feels pressured and team members are getting into more arguments. As a nurse on the unit, what will best help you manage this stress? 1. keep a journal 2. participate in a unit meeting to discuss feelings about the patient deaths. 3. Ask the nurse manager to assign you to less difficult patients. 4. Review the policy and procedure manual on proper care of patients after death.

Participate in a unit meeting to discuss feelings about the patient deaths.

Sharing eating utensils with a person who has a contagious illness increases the risk of illness. This type of health risk arises from: Lifestyle. Community. Family history. Personal hygiene habits.

Personal hygiene habits

A nurse is conducting a home visit with an older-adult couple. While in the home the nurse weights each individual and reviews the 3 day food diary with them. She also checks their blood pressure and encourages them to increase their fluids and activity levels to help with their voiced concern about constipation. The nurse is addressing which level of need according to Maslow?

Physiological

A patient registered at the local fitness center and purchased a pair of exercise shoes. The patient is in what stage of behavioral change?

Preparation

A nurse is using data collected from the unit to monitor the incidence of falls after the unit implemented a new fall protocol. The nurse is working in which area? A). Quality improvement B). Health care patient system C). Nursing informatics D). Computerized nursing network

Quality improvement - Quality data are the outcome of both QI initiatives. QI is an approach to the continuous study and improvement of the processes of providing health care services to meet the needs of patients and others and inform health care policy. The QI program of an institution focuses on improvement of health care-related processes such as fall prevention.

Confidentiality

Rights and privileges of patients for protection of privacy. Fine for violations. Cannot share med info without patient consent to anyone not involved in direct care of that patient.

A student nurse is telling a faculty member that her patient talked about gaining spiritual comfort from being focused on her inner self, including her values and principles. The instructor explains that this is an example of:

Self-transcendence

a nurse meets with the registered dietician and physical therapist to develop a plan of care that focuses on improving nutrition and mobility for a patient. this is an example of which quality and safety in the education of nurses competency - Patient-centered care - Safety - Teamwork and collaboration - Informatics

Teamwork and collaboration

Patient Privacy

The patient's right to keep information confidential; outlined in HIPPA.

A male client receives a prescription for ondansetron hydrochloride (Zofran) 4 mg IV to prevent postoperative nausea after an inguinal hernia repair. The medication is available in 2 mg/ml. How many ml should the nurse administer? (Enter numeric value only.) Use ratio and proportion, 4 mg : X ml = 2 mg : 1 ml 2X = 4 X = 2

Use ratio and proportion, 4 mg : X ml = 2 mg : 1 ml 2X = 4 X = 2

The nurse is preparing to administer Hepatitis B Vaccine, Recombinant (Energix-B) 5 mcg IM to a school-aged child. The vaccine is labeled, 10 mcg/ml. How many ml should the nurse administer? (Enter numeric value only. If rounding is required, round to the nearest tenth).

Use ratio and proportion, 5 mcg : X ml :: 10 mcg : 1ml 10X = 5 X = 0.5 ml

The healthcare provider prescribes acetazolamide (Diamox) 600 mg/m2/day divided into 3 doses. The nurse calculates the child's body surface area (BSA) as 0.7 m2. How many mg should the child receive per dose? (Enter the numeric value only.)

Using the child's BSA, 0.7 m2, calculate the mg/dose, 600 mg x 0.7 m2 = 420 mg/day/3 doses = 140 mg/dose

The healthcare provider prescribes a continuous intravenous infusion of dextrose 5% and 0.45% sodium chloride with KCl 20 mEq/1000 ml to be delivered over 8 hours. The nurse should program the infusion pump to deliver how many ml/hour? (Enter numeric value only. If rounding is required, round to the nearest whole number.)

Using the formula volume/time: 1000 ml/8 hours = 125 ml/hour

The healthcare provider prescribes a continuous infusion of 5% dextrose in 0.45% sodium chloride at 85 ml/hour. The IV administration set delivers 10 gtt/ml. The nurse should regulate the drop rate to deliver how many gtts/minute? (Enter numeric value only. If rounding is required, round to the nearest whole number.)

Volume/Time (minutes) x drop factor (gtt/ml) = 85 ml/ 60 minutes x 10 gtt/ml = 14.16 = 14 gtt/minute

accountability

You administer a once-a-day cardiac medication at the wrong time, but nobody sees it. However you contact the primary care provider and your head nurse and follow agency procedure

confidentiality

You see an open medical record on the computer and close it so no one else can read the record without proper access.

responsibility

You tell your patine that you will return in 30 minutes to give him his next pain medication

Nursing Diagnosis

a clinical judgment about individual, family, or community responses to actual and potential health problems or life processes that the nurse is licensed and competent to treat. **what makes it unique is having patients involved in the process (if/when possible)**

Health Promotion Diagnosis

a clinical judgment of a person's, family's or community's motivation, desire, and readiness to increase well-being and actualize human health potential as expressed in their readiness to enhance specific health behaviors such as nutrition and exercise. - can be used in any health state, do not require current levels of wellness. ex: readiness for enhanced family coping, readiness for enhanced nutrition

Related Factor

a condition, historical factor, or etiology that gives a context for the defining characteristics and shows a type of relationship with the nursing diagnosis; needed to individualize and formulate a nursing diagnosis.

A new nurse asks the preceptor why a change-of-shift report is important since care is documented in the chart. What is the preceptor's best response? a. "A hand-off report provides an opportunity to share essential information to ensure patient safety and continuity of care." b. "A change-of-shift report provides the oncoming nurse with data to help set priorities and establish reimbursement costs." c. "A hand-off report provides an opportunity for the oncoming nurse to ask questions and determine research priorities." d. "A change-of-shift report provides important information to caregivers and develops relationships within the health care team."

a. "A hand-off report provides an opportunity to share essential information to ensure patient safety and continuity of care."

A nurse is charting on a patient's record. Which action is most accurate legally? a. Charts legibly b. States the patient is belligerent c. Uses correction fluid to correct error d. Writes entry for another nurse

a. Charts legibly

A nurse has taught the staff about informatics. Which statement indicates that the staff needs more education? a. If a nurse has computer competency, the nurse is competent in informatics. b. To be proficient in informatics, a nurse should be able to discover, retrieve, and use information in practice. c. A nurse needs to know how to acquire, critique, and apply scientific evidence from literature databases. d. Nursing informatics integrates nursing science, computer science, and information science to manage and communicate information in nursing practice.

a. If a nurse has computer competency, the nurse is competent in informatics.

A nurse prepared an audiotaped exchange with another nurse of information about a patient. Which action did the nurse complete? The nurse completed a a. Report. b. Record. c. Consultation. d. Referral.

a. Report.

Professional nursing promotes

accountability responsibility advocacy confidentiality

Types of Nursing Diagnoses (3)

actual diagnoses, risk diagnoses, and health promotion diagnoses

Contemporatry nursing requires that the nurse has knowledge and skills for a variety of professional roles and responsibilities. Which of the following are examples. - Caregiver - Autonomy and accountability - Patient Advocate - Health Promotion - Lobbyist

all but lobbyist

Collaborative Problem

an actual or potential physiological complication that nurses monitor to detect the onset of changes in a patient's status.

When thinking critically and making decisions about patient care you..

apply knowledge, clinical experiences and professional standards.

A nurse has seen many cancer cancer patients struggle with pain management because they are afraid of becoming addicted to the medicine. Pain control is priority for cancer care. By helping patients focus on their values and beliefs about paid control, a nurse can best make clinical decisions. This is an example of: 1. Creativity 2. Fairness 3. Clinical reasoning 4. Applying ethical criteria

applying ethical criteria

Which situation will require the nurse to obtain a telephone order? a. As the nurse and primary care provider leave a patient's room, the primary care provider gives the nurse an order. b. At 0100, a patient's blood pressure drops from 120/80 to 90/50 and the incision dressing is saturated with blood. c. At 0800, the nurse and primary care provider make rounds and the primary care provider tells the nurse a diet order. d. A nurse reads an order correctly as written by the primary care provider in the patient's medical record.

b. At 0100, a patient's blood pressure drops from 120/80 to 90/50 and the incision dressing is saturated with blood.

A nurse prepares to insert a Foley catheter. The procedure manual calls for the patient to lie the dorsal recumbent options. The patient complains of having back pain when lying on her back. Despite this, the nurse positions the patient supine with knees flexed as the manual recommends and begins to insert the catheter. This is an example of: 1. Accuracy 2. Reflection 3. Rick taking 4. Basic critical thinking

basic critical thinking

Core measures

benchmark evidence-based standards of best practices used to gauge how well a hospital gives care to its patients who are admitted to seek treatment for a specific disease (e.g., heart failure) or who need a specific treatment (e.g., an immunization) The Joint Commission (TJC) and Centers for Medicare and Medicaid Services (CMS)

A nurse has taught the patient how to use crutches. The patient went up and down the stairs using crutches with no difficulties. Which information will the nurse use for the "I" in PIE charting? a. Patient went up and down stairs b. Deficient knowledge regarding crutches c. Demonstrated use of crutches d. Used crutches with no difficulties

c. Demonstrated use of crutches

A nurse is a member of an interdisciplinary team that uses critical pathways. According to the critical pathway, on day 2 of the hospital stay, the patient should be sitting in the chair. It is day 3, and the patient cannot sit in the chair. What should the nurse do? a. Focus charting using the DAR format. b. Add this data to the problem list. c. Document the variance in the patient's record. d. Report a positive variance in the next interdisciplinary team meeting.

c. Document the variance in the patient's record.

A nurse has provided care to a patient. Which entry should the nurse document in the patient's record? a. "Patient seems to be in pain and states, 'I feel uncomfortable.'" b. Status unchanged, doing well c. Left abdominal incision 1 inch in length without redness, drainage, or edema d. Patient is hard to care for and refuses all treatments and medications. Family present.

c. Left abdominal incision 1 inch in length without redness, drainage, or edema

A nurse developed the following discharge summary sheet. Which critical information should be added? TOPIC Medication Diet Activity level Follow-up care Wound care Phone numbers When to call the doctor Time of discharge a. Kardex form b. Admission nursing history c. Mode of transportation d. SOAP notes

c. Mode of transportation

Ethics is the study of

conduct and character. It is concerned with determined what is good or valuable for individuals and society at large

Which entry will require follow-up by the nurse manager? 0800 Patient states, "Fell out of bed." Patient found lying by bed on the floor. Legs equal in length bilaterally with no distortion, pedal pulses strong, leg strength equal and strong, no bruising or bleeding. Neuro checks within normal limits. States, "Did not pass out." Assisted back to bed. Call bell within reach. Bed monitor on. -------------------Jane More, RN 0810 Notified primary care provider of patient's status. New orders received. -------------------Jane More, RN 0815 Portable x-ray of L hip taken in room. States, "I feel fine." -------------------Jane More, RN 0830 Incident report completed and placed on chart. -------------------Jane More, RN a. 0800 b. 0810 c. 0815 d. 0830

d. 0830

A nurse wants to reduce data entry errors on the computer system. Which behavior should the nurse implement? a. Use the same password all the time. b. Share password with only one other staff member. c. Print out and review computer nursing notes at home. d. Chart on the computer immediately after care is provided.

d. Chart on the computer immediately after care is provided.

Georgia Board of Nursing

deals with licensures, nursing practice information, and more in the state of Georgia

Clinical Criterion

either an objective or subjective sign, symptom, or risk factor that when analyzed with other criteria, leads to a diagnostic conclusion.

human genetics

ethically to what extent genetics should be taken into account for health care.

Defining characteristics

found within data clusters; patterns of data that contain the clinical criteria that are observable and verifiable.

Continuing Education

involves formal, organized educational programs offered by universities, hospitals, state nurses associations, professional nursing organizations, and educational and health care institutions. It has an evaluation component.

inservice education

is an informal learning activity that delivers information required by a person in order to do their job properly at a specific place by the employer. It does not have an evaluation component.

What does critical thinking involve?

open-mindedness, continual inquiry, and perseverance, combined with a willingness to look at each unique patient situation and determine which identified assumptions are true and relevant.

A nurse just started working at a well-baby clinic. One of her recent experiences was to help a mother learn the steps of breastfeeding. During the first clinic visit the mother had difficulty positioning the baby during feeding. After the visit the nurse considers what affected the inability of the mother to breastfeed, including the mother's obesity and inexperience. The nurse's review of the situation is called. 1. Reflection 2. Perseverance 3. intuition 4. Problem situation

reflection

A nurse is preparing medications for a patient. The nurse checks the name of the medication on the label with the name of the medication on the doctor's order. At the bedside the nurse checks the patient's name against the medication order as well. The nurse is following which critical thinking attitude: 1. responsible 2. complete 3. accurate 4. broad

responsible

Accountability

the ability to answer for one's actions. Likewise, an institution exercise by monitoring individual and institutional compliance with national standards.

State Board of Education

the legal governing body that exercises general control and supervision of the schools in a state

Diagnostic Label

the name of the nursing diagnosis as approved by NANDA International; describes the essence of a patient's response to health conditions in as few words as possible.

Civil Law

the system of law concerned with private relations between members of a community rather than criminal, military, or religious affairs. protect a person's rights

Licensure

to be given a license to practice nursing in a state or province after successfully meeting requirements In the US all boards of of nursing require RN candidates to pass the NCLEX-RN. This provides a standardized minimum knowledge base for nurses.

Actual Diagnosis

type of nursing diagnosis; describes human responses to health conditions or life processes that exist in an individual, family, or community. Defining characteristics support the diagnostic judgement. ex. wandering, impaired social interaction, stress urinary incontinence.

Responsibility

willingness to respect one's professional obligations and to follow through. for example, following an agency's policy and procedures. A nurse agrees accept consequences for his/her actions and the actions of those they delegate tasks to. Nurse takes action to remain competent.

Etiology

related factor of a nursing diagnosis; always within the domain of nursing practice and a condition that responds to nursing interventions.

Risk Diagnosis

type of nursing diagnosis; describes human responses to health conditions or life processes that may develop in a vulnerable individual, family, or community. - do NOT have related factors or defining characteristics because they have not occurred yet; instead they have risk factors. ex.: risk for lonliness, risk for acute confusion

*Allison, age 15 years, calls her best friend Laura and is crying. She has a date with John, someone she has been hoping to date for months, but now she has a pimple on her forehead. Laura firmly believes that John and everyone else will notice the blemish right away. This is an example of the:* A. Imaginary audience. B. False-belief syndrome. C. Personal fable. D. Personal absorption syndrome.

*Answer: A* Rationale: Adolescents are quite egocentric and have the belief that everyone is focused on them and sees all of their flaws.

*When Ryan was 3 months old, he had a toy train; when his view of the train was blocked, he did not search for it. Now that he is 9 months old, he looks for it, reflecting the presence of:* A. Object permanence. B. Sensorimotor play. C. Schemata. D. Magical thinking.

*Answer: A* Rationale: He is now in Piaget's later stage of sensorimotor thought and has learned that objects exist even though he cannot see or touch them.

*You are caring for a recently retired man who appears withdrawn and says he is "bored with life." Applying the work of Havinghurst, you would help this individual find meaning in life by:* A. Encouraging him to explore new roles. Correct B. Encouraging relocation to a new city. C. Explaining the need to simplify life. D. Encouraging him to adopt a new pet.

*Answer: A* Rationale: The activity theory states that continuing an active, involved lifestyle results in greater satisfaction and well-being.

*Which of the following are examples of the conventional reasoning form of cognitive development? (Select all that apply.)* A. A 35-year-old woman is speaking with you about her recent diagnosis of a chronic illness. She is concerned about her treatment options in relation to her ability to continue to care for her family. As she considers the options and alternatives, she incorporates information, her values, and emotions to decide which plan will be the best fit for her. B. A young father is considering whether or not to return to school for a graduate degree. He considers the impact the time commitment may have on the needs of his wife and infant son. C. A teenage girl is encouraged by her peers to engage in shoplifting. She decides not to join her peers in this activity because she is afraid of getting caught in the act. D. A single mother of two children is unhappy with her employer. She has been unable to secure alternate employment but decides to quit her current job.

*Answer: A, B* Rationale: Adults recognize that there are various solutions to problems and that different situations demand different solutions. Decision making includes consideration of the potential impact of a situation on others. Avoiding punishment is a pre-conventional stage of reasoning.

*Which of the following activities are examples of the use of activity theory in older adults? (Select all that apply.)* A. Teaching an older adult how to use e-mail to communicate with a grandchild who lives in another state B. Introducing golf as a new hobby C. Leading a group walk of older adults each morning D. Engaging an older adult in a community project with a short-term goal E. Directing a community play at the local theater

*Answer: A, B, D* Rationale: Activity theory aids cognitive performance by using new approaches. A daily walk would not be a new activity, and the repetitiveness of it would not meet the definition of Activity Theory. Activities that have short-term goals can be engaging for older adults.

*According to Piaget's cognitive theory, a 12-year-old child is most likely to engage in which of the following activities?* A. Using building blocks to determine how houses are constructed B. Writing a story about a clown who wants to leave the circus C. Drawing pictures of a family using stick figures D. Writing an essay about patriotism

*Answer: B* Rationale: As adolescents mature, their thinking moves to abstract and theoretical subjects. They have the capacity to reason with respect to possibilities.

*An 18-month-old child is noted by the parents to be "angry" about any change in routine. This child's temperament is most likely to be described as:* A. Slow to warm up. B. Difficult. C. Hyperactive. D. Easy.

*Answer: B* Rationale: Children described as "difficult" adapt slowly to new routines and express their emotions forcefully; they like consistent structure.

*The nurse teaches parents how to have their children learn impulse control and cooperative behaviors. This would be during which of Erikson's stages of development?* A. Trust versus mistrust B. Initiative versus guilt C. Industry versus inferiority D. Autonomy versus sense of shame and doubt

*Answer: B* Rationale: Toddlers are learning that parents and society have expectations about behaviors and that they must learn to control their behavior.

*Nine-year-old Brian has a difficult time making friends at school and being chosen to play on the team. He also has trouble completing his homework and, as a result, receives little positive feedback from his parents or teacher. According to Erikson's theory, failure at this stage of development results in:* A. A sense of guilt. B. A poor sense of self. C. Feelings of inferiority. D. Mistrust.

*Answer: C* Rationale: School-age children need to feel real accomplishment and be accepted by peers to develop a sense of industry.

*What is the proper order by age progression for the stages of Freud's psychosexual development? 1. Phallic 2. Latent 3. Oral 4. Genital 5. Anal* A. 3, 5, 1, 4, 2 B. 3, 5, 2, 1, 4 C. 5, 3, 2, 1, 4 D. 3, 5, 1, 2, 4

*Answer: D*

*Elizabeth, who is having unprotected sex with her boyfriend, comments to her friends, "Did you hear about Kathy? You know, she fools around so much; I heard she was pregnant. That would never happen to me!" This is an example of adolescent:* A. Imaginary audience. B. False-belief syndrome. C. Personal fable. D. Sense of invulnerability.

*Answer: D* Rationale: Adolescents can be risk takers and believe that they are immune to the negative consequences of behaviors; they are just beginning to be future oriented in their thought process and see everything as black or white.

*Dave reports being happy and satisfied with his life. What do we know about him?* A. He is in one of the later developmental periods, concerned with reviewing his life. B. He is atypical, since most people in any of the developmental stages report significant dissatisfaction with their lives. C. He is in one of the earlier developmental periods, concerned with establishing a career and satisfying long-term relationships. D. It is difficult to determine Dave's developmental stage since most people report overall satisfaction with their lives in all stages

*Answer: D* Rationale: Each of the life stages can be achieved successfully and result in satisfaction, including old age.

*When preparing a 4-year-old child for a procedure, which method is developmentally most appropriate for the nurse to use?* A. Allowing the child to watch another child undergoing the same procedure B. Showing the child pictures of what he or she will experience C. Talking to the child in simple terms about what will happen D. Preparing the child through play with a doll and toy medical equipment

*Answer: D* Rationale: Preschoolers are in the preoperational stage of cognitive development and learn more easily when play is used to teach.

*The nurse is aware that preschoolers often display a developmental characteristic that makes them treat dolls or stuffed animals as if they have thoughts and feelings. This is an example of:* A. Logical reasoning. B. Egocentrism. C. Concrete thinking. D. Animism.

*Answer: D* Rationale: This is the belief that inanimate objects have lifelike qualities; it is a component of magical thinking evident in preoperational thought.

*You are working in a clinic that provides services for homeless people. The current local regulations prohibit providing a service that you believe is needed by your patients. You adhere to the regulations but at the same time are involved in influencing authorities to change the regulation. This action represents _______ stage of moral development.*

*Answer: Social contract (post conventional) orientation* Rationale: At this stage the individual recognizes that at times the law must be changed to meet the needs of society and that all people have basic rights, regardless of their social group.

Which of the following nursing activities is provided in a secondary health care environment? (Select all that apply.) A). Conducting blood pressure screenings for older adults at the Senior Center. B). Teaching a clinic patient with chronic obstructive pulmonary disease purse-lipped breathing techniques. C). Changing the postoperative dressing for a patient on a medical-surgical unit. D). Doing endotracheal suctioning for a patient on a ventilator in the medical intensive care unit.

- Changing the postoperative dressing for a patient on a medical-surgical unit. - Doing endotracheal suctioning for a patient on a ventilator in the medical intensive care unit. **In secondary care the diagnosis and treatment of illnesses are traditionally the most common services. Secondary services are usually provided in an acute care facility. Critical care units and inpatient medical-surgical units provide secondary and tertiary care.**

Sources of Error in Clustering Data

- Insufficient cluster of cues - Premature or early closure - Incorrect clustering

Sources of Diagnostic Errors in Collecting Data

- Lack of knowledge or skill - Inaccurate data - Missing data - Disorganization

Which of the following are characteristics of managed care systems? (Select all that apply.) A). Provider receives a predetermined payment for each patient in the program B). Payment is based on a set fee for each service provided. C). System includes a voluntary prescription drug program for an additional cost. D). System tries to reduce costs while keeping patients healthy. E). Focus of care is on prevention and early intervention.

- Provider receives a predetermined payment for each patient in the program. - System tries to reduce costs while keeping patients healthy. - Focus of care is on prevention and early intervention. **Managed care programs have administrative control over primary health care services for a defined patient population. The provider or health care system receives a predetermined capitated payment for each patient enrolled in the program. In this case the managed care organization assumes financial risk in addition to providing patient care. The focus of care of the organization shifts from individual illness care to prevention, early intervention, and outpatient care. If people stay healthy, the cost of medical care declines. Systems of managed care focus on containing or reducing costs, increasing patient satisfaction, and improving the health or functional status of the individual.**

Which of the following are examples of the nurse participating in primary care activities? (Select all that apply.) A). Providing prenatal teaching on nutrition to a pregnant woman during the first trimester B). Assessing the nutritional status of older adults who come to the community center for lunch. C). Working with patients in a cardiac rehabilitation program D). Providing home wound care to a patient E). Teaching a class to parents at the local grade school about the importance of immunization

- Providing prenatal teaching on nutrition to a pregnant woman during the first trimester - Assessing the nutritional status of older adults who come to the community center for lunch. - Teaching a class to parents at the local grade school about the importance of immunizations. **Primary care activities are focused on health promotion. Health promotion programs contribute to quality health care by helping patients acquire healthier lifestyles. Health promotion activities help keep people healthy through exercise, good nutrition, rest, and adopting positive health attitudes and practices.**

The nursing staff is developing a quality program. Which of the following are nursing-sensitive indicators from the National Database of Nursing Quality Indicators (NDNQI) that the nurses can use to measure patient safety and quality for the unit? (Select all that apply.) A). Use of physical restraints B). Pain assessment, intervention, and reassessment C). Patient satisfaction with food preparation D). Registered nurse (RN) education and certification E). Number of outpatient surgical cases per year

- Use of physical restraints - Pain assessment, intervention, and reassessment - Registered nurse (RN) education and certification **Physical restraint use, pain management, and RN education and certification are among the reportable nursing sensitive indicators for NDNQI. The other options are not nursing-sensitive indicators.**

Errors in Labeling Data

- Wrong diagnostic label selected - Evidence that another diagnosis is more likely - Condition a collaborative problem - Failure to validate nursing diagnosis with patient - Failure to seek guidance

A health care provider's prescription reads morphine sulfate, 8 mg stat. The medication ampule reads morphine sulfate, 10 mg/mL. The nurse prepares how many milliliters to administer the correct dose?

0.8 mL

A nurse is presenting a program to workers in a factory covering safety topics, including the wearing of hearing protectors when workers are in the factory. Which level of prevention is the nurse practicing? 1. Primary prevention 2. Secondary prevention 3. Tertiary prevention 4. Quaternary prevention

1 Primary prevention is aimed at health promotion and includes health education programs, immunizations, and physical and nutritional fitness activities. It can be provided to an individual and includes activities that focus on maintaining or improving the general health of individuals, families, and communities. It also includes specific protection such as hearing protection in occupational settings.

In which of the following examples is a nurse applying critical thinking skills in practice? (SATA) 1. The nurse thinks back about a personal experience before administering a medication subcutaneously 2. The nurse uses a pain-rating scale to measure a patient's pain. 3. The nurse explains a procedure step by step for giving an enema to a patient care technician 4. The nurse gathers data on a patient with a mobility limitation to identify a nursing diagnosis. 5. A nurse offers support to a colleague who has witnessed a stressful event.

1 2 4

Which of the following describes a nurse's application of a specific knowledge base during critical thinking? (SATA) 1. initiative in reading current evidence from the literature 2. application of nursing theory 3. reviewing policy and procedure manual 4. considering holistic view of patient needs 5. previous time caring for a specific group of patients

1 2 4

When nurses are communicating with adolescents, they should: 1 Be alert to clues to their emotional state. 2 Ask closed-ended questions to get straight answers. 3 Avoid looking for meaning behind adolescents' words or actions. 4 Avoid discussing sensitive issues such sex and drugs.

1 Be alert to clues to their emotional state.

An 8-year-old child is being admitted to the hospital from the emergency department with an injury from falling off her bicycle. Which of the following will most help her adjust to the hospital? 1 Explain hospital routines such as meal times to her. 2 Use terms such as "honey" and "dear" to show a caring attitude. 3 Explain when her parents can visit and why siblings cannot come to see her. 4 Since she is young, orient her parents to her room and hospital facility.

1 Explain hospital routines such as meal times to her.

In an interview with a pregnant patient, the nurse discussed the three risk factors that have been cited as having a possible effect on prenatal development. They are: 1 Nutrition, stress, and mother's age. 2 Prematurity, stress, and mother's age. 3 Nutrition, mother's age, and fetal infections. 4 Fetal infections, prematurity, and placenta previa.

1 Nutrition, stress, and mother's age.

At a well-child examination, the mother comments that her toddler eats little at mealtime, will only sit briefly at the table, and wants snacks all the time. Which of the following should the nurse recommend? 1 Provide nutritious snacks. 2 Offer rewards for eating at mealtimes. 3 Avoid snacks so she is hungry at mealtime. 4 Explain to her firmly why eating at mealtime is important.

1 Provide nutritious snacks.

A 63-year-old patient is retiring from his job at an accounting firm where he was in a management role for the past 20 years. He has been with the same company for 42 years and was a dedicated employee. His wife is a homemaker. She raised their five children, babysits for her grandchildren as needed, and belongs to numerous church committees. What are your major concerns for this patient? (Select all that apply.) 1 The loss of his work role 2 The risk of social isolation 3 A determination if the wife will need to start working 4 How the wife expects household tasks to be divided in the home in retirement 5 The age the patient chose to retire

1 The loss of his work role 4 How the wife expects household tasks to be divided in the home in retirement

What are the physical changes that occur as death approaches? 1. Unresponsiveness 2. Erythema 3. Mottling 4. Restlessness 5. Increased Urine Output 6. Weakness 7. Incontinence

1. 3. 4. 6. 7.

When planning care for the dying patient, which interventions promote the patient's dignity? 1. Providing respect 2. Viewing patients as a whole 3. Providing symptom management 4. Showing interest 5. Being present 6. Using a preferred name

1. 2. 4. 5. 6.

A nurse on a busy medicine unit is assigned to four patients. It is 10AM. Two patients have medications due and one of those has a specimen of urine to be collected. One patient is having complications from surgery and is being prepared to return to the operating room. The fourth patient requires instructions about activity restrictions before going home this afternoon. Which of the following should the nurse use in making clinical decisions appropriate for the patient group? (SATA) 1. consider availability of assistive personnel to obtain the specimen 2. combine activités to resolve more than one patient problem 3. analyze the diagnoses/problems and decide which are most urgent based on patients' needs. 4. plan a family conference for tomorrow to make decisions about resources that patient will need to go home. 5. identify the nursing diagnoses for the patient going home 1. Consider availability of assistive personnel to obtain the specimen 2. combine activities to resolve more than one patient problem 3. analyze the diagnoses/problems and decide which are most urgent

1. Consider availability of assistive personnel to obtain the specimen 2. combine activities to resolve more than one patient problem 3. analyze the diagnoses/problems and decide which are most urgent based on patients' needs

To best assist a patient in the grieving process, which of the following is most helpful to determine? 1. Previous experiences with grief and loss 2. Religious affiliation and denomination 3. Ethnic background and cultural practices 4. Current financial status

1. Previous experiences with grief and loss

A health care provider's prescription reads levothyroxine (Synthroid), 150 mcg orally daily. The medication label reads Synthroid, 0.1 mg/tablet. The nurse administers how many tablet(s) to the client?

1.5 tablets

A health care provider prescribes 3000 mL of D5W to be administered over a 24-hour period. The nurse determines that how many milliliters per hour will be administered to the client?

125 mL

A health care provider's prescription reads potassium chloride 30 mEq to be added to 1000 mL normal saline (NS) and to be administered over a 10-hour period. The label on the medication bottle reads 40 mEq/20 mL. The nurse prepares how many milliliters of potassium chloride to administer the correct dose of medication?

15 mL

A health care provider prescribes heparin sodium, 1300 units/hour by continuous intravenous (IV) infusion. The pharmacy prepares the medication and delivers an IV bag labeled heparin sodium 20,000 units/250 mL D5W. An infusion pump must be used to administer the medication. The nurse sets the infusion pump at how many milliliters per hour to deliver 1300 units/hour? Record your answer to the nearest whole number.

16 mL

A 180-pound adult male is admitted to the Emergency Center after receiving thermal burns to 60 percent of his body. Using the Parkland formula of 4 ml/kg/24 hours, the client should receive how many ml of fluid during the first 24 hours? (Enter the numeric value only. If rounding is required, round to the nearest whole number.)

19632 Using the Parkland formula, the client's fluid requirements for the first 24 hours after injury: 4 ml lactated Ringer's solution x Body weight (in kilograms) x Percent burn. Convert 180 pounds to kg = 180 / 2.2 = 81.8 kg 4 ml x 81.8 kg x 60 = 19632 ml over the first 24 hours.

Match the concepts for a critical thinker with the application of the term. a. Anticipate how a patient might respond to a treatment. b. Organize assessment on the basis of patient priorities. c. Be objective in asking questions of a patient. d. Be tolerant of the patient's views and beliefs. ___1. Truth seeking ___2. Open- mindedness ___3. Analyticity ___4. Systematicity

1c 2d 3a 4b

A nurse has conducted an assessment of a new patient who has come to the medical clinic. The patient is 82 years old and has had osteoarthritis for 10 years and diabetes mellitus for 20 years. He is alert but becomes easily distracted during the nursing history. He recently moved to a new apartment, and his pet beagle died just 2 months ago. He is most likely experiencing: 1 Dementia. 2 Depression. 3 Delirium. 4 Disengagement.

2 Depression.

A 71-year-old patient enters the emergency department after falling down stairs in the home. The nurse is conducting a fall history with the patient and his wife. They live in a one-level ranch home. He has had diabetes for over 15 years and experiences some numbness in his feet. He wears bifocal glasses. His blood pressure is stable around 130/70. The patient does not exercise regularly and complains of weakness in his legs when climbing stairs. He is alert, oriented, and able to answer questions clearly. What are the fall risk factors for this patient? (Select all that apply.) 1 Presence of a chronic disease 2 Impaired vision 3 Residence design 4 Blood pressure 5 Leg weakness 6 Exercise history

2 Impaired vision 5 Leg weakness 6 Exercise history

Parents are concerned about their toddler's negativism and ask the nurse for guidance. Which is the most appropriate recommendation? 1 Provide more attention. 2 Reduce opportunities for a "no" answer. 3 Be consistent with punishment. 4 Provide opportunities for the toddler to make decisions.

2 Reduce opportunities for a "no" answer.

You are caring for a 4-year-old child who is hospitalized for an infection. He tells you that he is sick because he was "bad." Which is the most correct interpretation of his comment? 1 Indicative of extreme stress 2 Representative of his cognitive development 3 Suggestive of excessive discipline at home 4 Indicative of his developing sense of inferiority

2 Representative of his cognitive development

Taste buds atrophy and lose sensitivity, and appetite may decrease. As a result, the older adult is less able to discern: 1 Spicy and bland foods. 2 Salty, sour, and bitter tastes. 3 Hot and cold food temperatures. 4 Moist and dry food preparations.

2 Salty, sour, and bitter tastes.

A nurse is caring for a patient preparing for discharge from the hospital the next day. The patient does not read and has a hearing loss. His family caregiver will be visiting before discharge. What can you do to facilitate the patient's understanding of his discharge instructions? (Select all that apply.) 1 Speak loudly so the patient can hear you. 2 Sit facing the patient so he is able to watch your lip movements and facial expressions. 3 Present one idea or concept at a time. 4 Send a written copy of the instructions home with him and tell him to have the family review them. 5 Include the family caregiver in the teaching session.

2 Sit facing the patient so he is able to watch your lip movements and facial expressions. 3 Present one idea or concept at a time. 5 Include the family caregiver in the teaching session.

During a home health visit a nurse talks with a patient and his family caregiver about the patient's medications. The patient has hypertension and renal disease. Which of the following findings places him at risk for an adverse drug event? (Select all that apply.) 1 Taking two medications for hypertension 2 Taking a total of eight different medications during the day. 3 Having one physician who reviews all medications 4 Patient's health history 5 Involvement of the caregiver in assisting with medication administration

2 Taking a total of eight different medications during the day. 4 Patient's health history

A patient's family member is considering having her mother placed in a nursing center. You have talked with the family before and know that this is a difficult decision. Which of the following criteria would you recommend in choosing a nursing center? (Select all that apply.) 1 The center should be clean, and rooms should look like a hospital room. 2 There should be adequate staffing on all shifts. 3 Social activities should be available for all residents. 4 Three meals should be served daily with a set menu and serving schedule. 5 Family involvement in care planning and assisting with physical care is necessary.

2 There should be adequate staffing on all shifts. 3 Social activities should be available for all residents. 5 Family involvement in care planning and assisting with physical care is necessary.

A health care provider's prescription reads phenytoin (Dilantin) 0.2 g orally twice daily. The medication label states that each capsule is 100 mg. The nurse prepares how many capsule(s) to administer one dose?

2 capsule(s)

A health care provider's prescription reads clindamycin phosphate (Cleocin Phosphate) 0.3 g in 50 mL normal saline (NS) to be administered intravenously over 30 minutes. The medication label reads clindamycin phosphate (Cleocin Phosphate) 900 mg in 6 mL. The nurse prepares how many milliliters of the medication to administer the correct dose? Round your answer to the nearest whole number.

2 mL

A nurse is conducting a patient-centered interview. Place the statements from the interview in the correct order. 1. "You say you've lost weight. Tell me how much weight you have lost in the last month." 2. "My name is Todd. I'll be the nurse taking care of you today. I'm going to ask you a series of questions to gather your health history." 3. "I have no further questions. Thank you for your patience." 4. "Tell me what brought you to the hospital." 5. "So, to summarize, you've lost about 6 pounds in the last month, and your appetite has been poor—correct?"

2) "My name is Todd. I'll be the nurse taking care of you today. I'm going to ask you a series of questions to gather your health history." 4) "Tell me what brought you to the hospital." 1) "You say you've lost weight. Tell me how much weight you have lost in the last month." 5) "So, to summarize, you've lost about 6 pounds in the last month, and your appetite has been poor—correct?" 3) "I have no further questions. Thank you for your patience."

A patient is receiving palliative care for symptom management related to anxiety and pain. A family member asks if the patient is dying and now in "hospice." What does the nurse tell the family about palliative care? 1. Palliative care and hospice are the same thing 2. Palliative care is for any patient, any time, any disease in any setting 3. Palliative care strategies are primarily designed to treat the patient's illness 4. Palliative care relieves the symptoms of illness and treatment. 5. Palliative care selects home health care services

2. 4.

A family member of a dying patient talks casually with the nurse and expresses relief that she will not have to visit at the hospital anymore. Which theoretical description of grief best applies to this family member? 1. Denial 2. Anticipatory Grief 3. Yearning and Searching 4. Dysfunctional Grief

2. Anticipatory Grief

When providing postmortem care, which action is a priority for the nurse? 1. Locating the patient's clothing 2. Providing culturally and religiously sensitive care in body preparation 3. Transporting the body to the morgue as soon as possible 4. Providing postmortem care to protect the family of the deceased from having to view the body

2. Providing culturally and religiously sensitive care in body preparation

A client who has a gastrostomy feeding tube is receiving 3/4 strength Ensure 240 ml every 6 hours. Full strength Ensure is available in a 240 ml can. The nurse should use how many ml of Ensure to prepare the feeding? (Enter numeric value only. If needed, round to the nearest whole number.)

240 ml Ensure : 1 :: X ml Ensure : 0.75 240/X : 1 / 0.75 X = 240 x 0.75 = 180 ml Ensure + 60 ml water = 240 ml

Cefuroxime sodium, 1 g in 50 mL normal saline (NS), is to be administered over 30 minutes. The drop factor is 15 drops (gtt)/mL. The nurse sets the flow rate at how many drops per minute?

25 drops

An aspect of clinical decision making is knowing the patient. Which of the following is the most critical aspect of developing the ability to know the patient? 1. Working in multiple health care settings. 2. Learning good communication skills 3. spending time establishing relationships with patients 4. relying on evidence in practice

3

A patient had surgery for a total knee replacement a week ago and is currently participating in daily physical rehabilitation sessions at the surgeon's office. In what level of prevention is the patient participating? 1. Primary prevention 2. Secondary prevention 3. Tertiary prevention 4. Quaternary prevention

3 Tertiary prevention involves minimizing the effects of long-term disease or disability by interventions directed at preventing complications and deterioration following surgery. Tertiary prevention activities are directed at rehabilitation rather than diagnosis and treatment. Care at this level aims to help patients achieve as high a level of functioning as possible, despite the limitations caused by illness or impairment. This level of care is called preventive care because it involves preventing further disability or reduced functioning.

Based on the trans-theoretical model of change, what is the most appropriate response to a patient who states: "Me, stop smoking? I've been smoking since I was 16!" 1. "That's fine. Some people who smoke live a long life." 2. "OK. I want you to decrease the number of cigarettes you smoke by one each day, and I'll see you in 1 month." 3. "I understand. Can you think of the greatest reason why stopping smoking would be challenging for you?" 4. "I'd like you to attend a smoking cessation class this week and use nicotine replacement patches as directed."

3 The patient's response indicates that he is in the precontemplation stage and not intending to make a change in behavior in the next 6 months. In this stage the patient is not interested in information about the behavior and may be defensive when confronted with the information. Asking an open-ended question about smoking may stimulate the patient to identify a reason to begin a behavior change. Nurses are challenged to motivate and facilitate health behavior change in working with individuals.

A nurse changed a patient's surgical wound dressing the day before and now prepares for another dressing change. The nurse had difficulty removing the gauze from the wound bed yesterday, causing the patient discomfort. Today he gives the patient an analgesic 30 minutes before the dressing change. Then he adds some sterile saline to loosen the gauze for a few minutes before removing it. The patient reports that the procedure was much more comfortable. Which of the following describes the nurse's approach to the dressing change? (SATA) 1. clinical inference 2. basic critical thinking 3. complex critical thinking 4. experience 5. reflection

3 4

The nurse is providing information on prevention of sudden infant death syndrome (SIDS) to the mother of a young infant. Which of the following statements indicates that the mother has a good understanding? (Select all that apply.) 1 "I won't use a pacifier to help my baby sleep." 2 "I'll be sure my baby does not spend any time on her abdomen." 3 "I'll place my baby on her back for sleep." 4 "I'll be sure to keep my baby's room cold."

3 "I'll place my baby on her back for sleep." 4 "I'll be sure to keep my baby's room cold."

A parent has brought her 6-month-old infant in for a well-child check. Which of her statements indicates a need for further teaching? 1 "I can start giving her whole milk at about 12 months." 2 "I can continue to breastfeed for another 6 months." 3 "I've started giving her plenty of fruit juice as a way to increase her vitamin intake." 4 "I can start giving her solid food now."

3 "I've started giving her plenty of fruit juice as a way to increase her vitamin intake."

You see a 76-year-old woman in the outpatient clinic. Her chief complaint is vision. She states she has really noticed glare in the lights at home. Her vision is blurred; and she is unable to play cards with her friends, read, or do her needlework. You suspect that she may have: 1 Presbyopia. 2 Disengagement. 3 Cataract(s). 4 Depression.

3 Cataract(s).

Kyphosis, a change in the musculoskeletal system, leads to: 1 Decreased bone density in the vertebrae and hips. 2 Increased risk for pathological stress fractures in the hips. 3 Changes in the configuration of the spine that affect the lungs and thorax. 4 Calcification of the bony tissues of the long bones such as in the legs and arm.

3 Changes in the configuration of the spine that affect the lungs and thorax.

You are working in an adolescent health center when a 15-year-old patient shares with you that she thinks she is pregnant and is worried that she may now have a sexually transmitted infection (STI). Her pregnancy test is negative. What is your next priority of care? 1 Contact her parents to alert them of her need for birth control. 2 Refer her to a primary health care provider to obtain a prescription for birth control. 3 Counsel her on safe sex practices. 4 Ask her to have her partner come to the clinic for STI testing.

3 Counsel her on safe sex practices.

A major life event such as the death of a loved one, a move to a nursing home, or a cancer diagnosis could precipitate: 1 Dementia. 2 Delirium. 3 Depression. 4 Stroke.

3 Depression.

You are working with an older adult after an acute hospitalization. Your goal is to help this person be more in touch with time, place, and person. What might you try? 1 Reminiscence 2 Validation therapy 3 Reality orientation 4 Body image interventions

3 Reality orientation

A nurse is using the Plan-Do-Study-Act (PDSA) strategy to do a quality improvement project to decrease patient falls on a nursing unit. What is the correct sequence for PDSA? 1. Bedside change of shift report is piloted on two medical-surgical units 2. Patient satisfaction levels after implementation of the bedside report are compared to patient satisfaction levels before the change 3. The nursing council develops a strategy for bedside change of shift report 4. After modifications are made in the shift report elements, bedside shift report is implemented on all nursing units A). 1, 3, 2, 4 B). 2, 1, 3, 4 C). 1, 2, 3, 4 D). 3, 1, 2, 4

3, 1, 2, 4

Which comment to a patient by a new nurse regarding palliative care needs to be corrected? 1. "Even though you're continuing treatment, palliative care is something we might want to talk about." 2. "Palliative care is appropriate for people with any diagnosis." 3. "Only people who are dying receive palliative care." 4. "Children are able to receive palliative care."

3. Only people who are dying receive palliative care

A young mother is dying of breast cancer with bone mestastasis and tells the nurse, "My body hurts so much. I can hardly move. Why is God making me suffer when I have done nothing bad in my life? I feel like giving up. How can I care for my children when I can't even care for myself?" What is the most appropriate nursing diagnosis for this patient? 1. Spiritual distress related to questioning God 2. Hopelessness related to terminal diagnosis 3. Pain related to disease process 4. Anticipatory grief related to impending death

3. Pain related to disease process

Gentamicin sulfate, 80 mg in 100 mL normal saline (NS), is to be administered over 30 minutes. The drop factor is 10 drops (gtt)/mL. The nurse sets the flow rate at how many drops per minute? Record your answer to the nearest whole number.

33 drops

The nurse is completing a time tape for a 1000-mL intravenous (IV) bag that is scheduled to infuse over 8 hours. The nurse has just placed the 1100 marking at the 500-mL level. The nurse would place the mark for 1200 at which numerical level (mL) on the time tape? *Fill in the blank.*

375 mL Rationale: If the IV is scheduled to run over 8 hours, the hourly rate is 125 mL/hour. Using 500 mL as the reference point, the next hourly marking would be at 375 mL, which is 125 mL less than 500.

A nurse enters a 72 year old patient's home and begins to observe her behaviors and examine her physical condition. The nurse learns that the patient lives alone and notices bruising on the patient's leg. When watching the patient walk, the nurse notes that she has an unsteady gait and leans to one side. The patient admits to having fallen in the past. The nurse identifies the patient as having the nursing diagnosis of Risk for Falls. This scenario is an example of: 1. inference 2. Basic critical thinking 3. Evaluation 4 Diagnostic reasoning

4

Place the steps of the scientific method in their correct order with number 1 being the first step of the process. 1. formulate a question or hypothesis 2. evaluate results of the study 3. collect data 4 identify the problem 5. test the question or hypothesis

4 3 1 5 2

Sexuality is maintained throughout our lives. Which answer below best explains sexuality in an older adult? 1 When the sexual partner passes away, the survivor no longer feels sexual. 2 A decrease in an older adult's libido occurs. 3 Any outward expression of sexuality suggests that the older adult is having a developmental problem. 4 All older adults, whether healthy or frail, need to express sexual feelings.

4 All older adults, whether healthy or frail, need to express sexual feelings.

Which of the following statements is most descriptive of the psychosocial development of school-age children? 1 Boys and girls play equally with each other. 2 Peer influence is not yet an important factor to the child. 3 They like to play games with rigid rules. 4 Children frequently have "best friends."

4 Children frequently have "best friends."

The type of injury a child is most vulnerable to at a specific age is most closely related to which of the following? 1 Provision of adult supervision. 2 Educational level of the parent 3 Physical health of the child 4 Developmental level of the child

4 Developmental level of the child

Older adults experience a change in sexual activity. Which best explains this change? 1 The need to touch and be touched is decreased. 2 The sexual preferences of older adults are not as diverse. 3 Physical changes usually do not affect sexual functioning. 4 Frequency and opportunities for sexual activity may decline.

4 Frequency and opportunities for sexual activity may decline.

Which approach would be best for the nurse to use with a hospitalized toddler? 1 Always give several choices. 2 Set few limits to allow for open expression. 3 Use noninvasive methods when possible. 4 Gain cooperation before attempting treatment.

4 Gain cooperation before attempting treatment.

The school nurse is counseling an obese 10-year-old child. What factor would be important to consider when planning an intervention to support the child's health? 1 Concentrate on the child only rather than the family since it is the child's responsibility. 2 Consider the use of medications to suppress the appetite. 3 First plan for weight loss through dieting and then add activity as tolerated. 4 Plan food intake to allow for growth

4 Plan food intake to allow for growth

While working in the high-school clinic, one of the students tells you that she is worried about her friend who has started to refuse to participate in group activities, no longer cares about how she looks at school, and is not going to all of her classes. Your assessment of these symptoms may indicate that: 1 She has just broken up with her boyfriend and time will heal all. 2 You will need to observe her over time to see if symptoms persist. 3 School may be too difficult for her right now. 4 She may be at increased risk for suicide.

4 She may be at increased risk for suicide.

You are caring for an 80-year-old man who recently lost his wife. He shares with you that he has been drinking more than he ever did in the past and feels hopeless without his wife. He reports that he rarely sees his children and feels isolated and alone. This patient is at risk for: 1 Dementia. 2 Liver failure. 3 Dehydration. 4 Suicide.

4 Suicide.

A student nurse is caring for a 78-year-old patient with multiple sclerosis. The patient has had an indwelling Foley catheter in for 3 days. Eight hours ago the patient's temperature was 37.1° C (98.8° F). The student reports her recent assessment to the registered nurse (RN): the patient's temperature is 37.2° C (99° F); the Foley catheter is still in place, draining dark urine; and the patient is uncertain what time of day it is. From what the RN knows about presentation of symptoms in older adults, what should he recommend first? 1 Tell the student that temporary confusion is normal and simply requires reorientation 2 Tell the student to increase the patient's fluid intake since the urine is concentrated 3 Tell the student that her assessment findings are normal for an older adult 4 Tell the student that he will notify the physician of the findings

4 Tell the student that he will notify the physician of the findings

The patient for whom you are caring needs a liver transplant to survive. This patient has been out of work for several months and doesn't have heath insurance or enough cash. Even though several ethical principles are at work in this case, list the principles from highest to lowest priority. 1. Accountability: You are the nurse are accountable for the well-being of this patient 2. Respect for autonomy: The patient's autonomy will be violated if he does not receive the liver transplant 3. Ethics of care: The caring thing that a nurse could provide this patient is resources for a liver transplant 4. Justice: The greatest question in this situation is how to determine the just distribution of the resources

4, 2, 3, 1

A nurse has the responsibility of managing a deceased patient's postmortem care. What is the proper order for postmortem care? 1. Bathe the body of the deceased 2. Collect any needed specimens 3. Remove all tubes and indwelling lines 4. Position the body for family viewing 5. Speak to the family members about their possible participation 6. Ensure that the request for organ/tissue donation was completed 7. Notify support person (e.g. spiritual care provider, bereavement specialist) for the family 8. Accurately tag the body, including the identity of the deceased and safety issues regarding infection control 9. Elevate the head of the bed

6. 9. 2. 5. 7. 3. 1. 4. 8.

At the end of the shift, the nurse is recording the fluid balance for a client receiving a continuous gastrostomy tube (GT) feeding. Based on the client's records, how many ml should the nurse record for the total fluid balance (intake - output) for the shift that started at 0700 and ended at 1900? (Enter numeric value only.) (Click on each image asset for additional information.)

695 The tube feeding is administered at 75 ml/hour for 11 hours (0700 - 1800) and then turned off for one hour because the residual is greater than 200 ml. 11 hours x 75 ml/hr = 825 ml. Three doses of 20 ml of medication are administered, with 10 ml of water used to flush the GT before and after each dose. 20 ml x 3 doses = 60 ml; 10 ml (flush) x 2 x 3 doses = 60 ml; so 825 ml + 60 ml + 60 ml = 945 ml. 250 ml of residual is removed and not replaced. 945 ml - 250 ml = 695 ml total fluid balance (Intake - output).

A loading dose of acetylcysteine (Mucomyst) 8 grams, which is available as a 20% solution (200 mg acetylcysteine per ml) is prescribed by nasogastric tube for a client with acetaminophen toxicity. How many ml of diluent should be added to the medication to obtain a 1:4 concentration? (Enter the numerical value only. If rounding is required, round to the whole number.)

8 grams = 8,000 mg prescribed dose. Using the formula, D/H x Q, 8,000 mg / 200 mg x 1 ml = 40 ml of the 20% solution. Dilute the 40 ml to a 1:4 concentration for administration using ratio and proportion, 1 : 4 solution :: 40 ml : X X= 160 ml total volume to administer. Subtract total volume of 160 ml - 40 ml of 20% concentration = 120 ml diluent is added to obtain a 1:4 concentration.

A health care provider prescribes regular insulin, 8 units/hour by continuous intravenous (IV) infusion. The pharmacy prepares the medication and then delivers an IV bag labeled 100 units of regular insulin in 100 mL normal saline (NS). An infusion pump must be used to administer the medication. The nurse sets the infusion pump at how many milliliters per hour to deliver 8 units/hour?

8 mL

Common Law

A legal system based on custom and court rulings Judicial rulings

advocacy

A patient at the end of life wants to go home to die, but the family wants every care possible. The nurse contacts the primary care provider abut the patient's request

nurse anesthetist

A registered nurse who has received additional training and education in the administration of anesthetic medications.

Which activity shows a nurse engaged in primary prevention?

A school health nurse provides a program to the first year students on healthy eating.

intentional tort

A tort committed by one who intends to do the act that creates the harm. assault, battery, slander

A nurse working on a medicine nursing unit is assigned to a 78-year-old patient who just entered the hospital with symptoms of H1N1 flu. The nurse finds the patient to be short of breath with an increased respiratory rate of 30 breaths/min. He lost his wife just a month ago. The nurse's knowledge about this patient results in which of the following assessment approaches at this time? (Select all that apply.) A) A problem-focused approach B) A structured comprehensive approach C) Using multiple visits to gather a complete database D) Focusing on the functional health pattern of role-relationship

A) A problem-focused approach C) Using multiple visits to gather a complete database

What technique(s) best encourage(s) a patient to tell his or her full story? (Select all that apply.) A) Active listening B) Back channeling C) Validating D) Use of open-ended questions E) Use of closed-ended questions

A) Active listening B) Back channeling D) Use of open-ended questions

A patient who visits the allergy clinic tells the nurse practitioner that he is not getting relief from shortness of breath when he uses his inhaler. The nurse decides to ask the patient to explain how he uses the inhaler, when he should take a dose of medication, and what he does when he gets no relief. On the basis of Gordon's functional health patterns, which pattern does the nurse assess? A) Health perception-health management pattern B) Value-belief pattern C) Cognitive-perceptual pattern D) Coping-stress tolerance pattern

A) Health perception-health management pattern

Which of the following are examples of data validation? (Select all that apply.) A) The nurse assesses the patient's heart rate and compares the value with the last value entered in the medical record. B) The nurse asks the patient if he is having pain and then asks the patient to rate the severity. C) The nurse observes a patient reading a teaching booklet and asks the patient if he has questions about its content. D) The nurse obtains a blood pressure value that is abnormal and asks the charge nurse to repeat the measurement. E) The nurse asks the patient to describe a symptom by saying, "Go on."

A) The nurse assesses the patient's heart rate and compares the value with the last value entered in the medical record. D) The nurse obtains a blood pressure value that is abnormal and asks the charge nurse to repeat the measurement.

A nurse gathers the following assessment data. Which of the following cues form(s) a pattern suggesting a problem? (Select all that apply.) A) The skin around the wound is tender to touch. B) Fluid intake for 8 hours is 800 mL. C) Patient has a heart rate of 78 and regular. D) Patient has drainage from surgical wound. E) Body temperature is 101° F (38.3° C). F) Patient asks, "I'm worried that I won't return to work when I planned."

A) The skin around the wound is tender to touch. D) Patient has drainage from surgical wound. E) Body temperature is 101° F (38.3° C).

Which statement made by a new graduate nurse about the teachback technique requires intervention and further instruction by the nurse's preceptor? a. "After teaching a patient how to use an inhaler, I need to use the Teach Back technique to test my patient's understanding." b. "The Teach Back technique is an ongoing process of asking patients for feedback." c. "Using Teach Back will help me identify explanations and communication strategies that my patients will most commonly understand." d. "Using pictures, drawings, and models can enhance the effectiveness of the Teach Back technique."

A. Teach Back is not a test of patient knowledge or ability to use devices but a confirmation of how well the nurse explained concepts to patients

The nurse is preparing to teach a prenatal class about fetal circulation. Which statements should be included in the teaching plan? *Select all that apply.* A. "The ductus arteriosus allows blood to bypass the fetal lungs." B. "One vein carries oxygenated blood from the placenta to the fetus." C. "The normal fetal heart tone range is 140 to 160 beats per minute in early pregnancy." D. "Two arteries carry deoxygenated blood and waste products away from the fetus to the placenta." E. "Two veins carry blood that is high in carbon dioxide and other waste products way from the fetus to the placenta."

A. "The ductus arteriosus allows blood to bypass the fetal lungs B. "One vein carries oxygenated blood from the placenta to the fetus." D. "Two arteries carry deoxygenated blood and waste products away from the fetus to the placenta."

The nurse is teaching a client about coughing and deep-breathing techniques to prevent postoperative complications. Which statement is *most appropriate* for the nurse to make to the client at this time as it relates to these techniques?. A. "Use of an incentive spirometer will help prevent pneumonia." B. "Close monitoring of your oxygen saturation will detect hypoxemia." C. "Administration of intravenous fluids will prevent or treat fluid imbalance." D. "Early ambulation and administration of blood thinners will prevent pulmonary embolism."

A. "Use of an incentive spirometer will help prevent pneumonia."

A client involved in a motor vehicle crash presents to the emergency department with severe internal bleeding. The client is severely hypotensive and unresponsive. The nurse anticipates that which intravenous (IV) solution will *most likely* be prescribed for this client? A. 5% dextrose in lactated Ringer's solution B. 0.33% sodium chloride (1/3 normal saline) C. 0.45% sodium chloride (1/2 normal saline) D. 0.225% sodium chloride (1/4 normal saline)

A. 5% dextrose in lactated Ringer's solution

The nurse is assigned to care for a group of clients. On review of the clients' medical records, the nurse determines that which client is *most likely* at risk for a fluid volume deficit? A. A client with an ileostomy B. A client with heart failure C. A client on long-term corticosteroid therapy D. A client receiving frequent wound irrigations

A. A client with an ileostomy

The nurse, caring for a group of adult clients on an acute care medical-surgical nursing unit, determines that which clients would be the *most likely* candidates for parenteral nutrition (PN)? *Select all that apply.* A. A client with extensive burns B. A client with cancer who is septic C. A client who has had an open cholecystectomy D. A client with severe exacerbation of Crohn's disease E. A client with persistent nausea and vomiting from chemotherapy

A. A client with extensive burns B. A client with cancer who is septic D. A client with severe exacerbation of Crohn's disease E. A client with persistent nausea and vomiting from chemotherapy

The nurse is teaching a client about the risk factors associated with colorectal cancer. The nurse determines that *further teaching is necessary* related to colorectal cancer if the client identifies which item as an associated risk factor? A. Age younger than 50 years B. History of colorectal polyps C. Family history of colorectal cancer D. Chronic inflammatory bowel disease

A. Age younger than 50 years

The nursing instructor asks a nursing student to explain the characteristics of the amniotic fluid. The student responds correctly by explaining which as characteristics of amniotic fluid? *Select all that apply.* A. Allows for fetal movement B. Surrounds, cushions, and protects the fetus C. Maintains the body temperature of the fetus D. Can be used to measure fetal kidney function E. Prevents large particles such as bacteria from passing to the fetus F. Provides an exchange of nutrients and waste products between the mother and the fetus

A. Allows for fetal movement B. Surrounds, cushions, and protects the fetus C. Maintains the body temperature of the fetus D. Can be used to measure fetal kidney function

The clinic nurse prepares to perform a focused assessment on a client who is complaining of symptoms of a cold, a cough, and lung congestion. Which should the nurse include for this type of assessment? *Select all that apply.* A. Auscultating lung sounds B. Obtaining the client's temperature C. Assessing the strength of peripheral pulses D. Obtaining information about the client's respirations E. Performing a musculoskeletal and neurological examination F. Asking the client about a family history of any illness or disease

A. Auscultating lung sounds B. Obtaining the client's temperature D. Obtaining information about the client's respirations

Chronic illness (e.g., diabetes mellitus, hypertension, rheumatoid arthritis) may affect a person's roles and responsibilities during middle adulthood. When assessing the health-related knowledge base of both the middle-age patient with a chronic illness and his family, your assessment includes which of the following? Select all that apply. A. The medical course of the illness B. The prognosis for the patient C. Socioeconomic status D. Coping mechanisms of the patient and family E. The need for community and social services

A. B. D. E.

The nurse is applying a topical corticosteroid to a client with eczema. The nurse should apply the medication to which body area? *Select all that apply.* A. Back B. Axilla C. Eyelids D. Soles of the feet E. Palms of the hands

A. Back D. Soles of the feet E. Palms of the hands Rationale: topical corticosteroids can be absorbed into the systemic circulation. Absorption is higher from regions where the skin is especially permeable (scalp, axilla, face, eyelids, neck, perineum, genitalia), and lower from regions where permeability is poor (back, palms, soles). The nurse should avoid areas of higher absorption to prevent systemic absorption.

The nurse is caring for a client who is postoperative following a pelvic exenteration and the health care provider changes the client's diet from NPO (nothing by mouth) status to clear liquids. The nurse should check which *priority* item before administering the diet? A. Bowel sounds B. Ability to ambulate C. Incision appearance D. Urine specific gravity

A. Bowel sounds

A postoperative client has been placed on a. clear liquid diet. The nurse should provide the client with which items that are allowed to be consumed on this diet? *Select all that apply.* A. Broth B. Coffee C. Gelatin D. Pudding E. Vegetable juice F. Pureed vegetables

A. Broth B. Coffee C. Gelatin

Formation of positive health habits may prevent the development of chronic illness later in life. Which of the following are examples of positive health habits? Select all that apply. A. Routine screening and diagnostic tests B. Unprotected sexual activity C. Regular exercise D. Excess alcohol consumption E. Consistent seat belt use

A. C. E.

The nurse is changing the central line dressing of a client receiving parenteral nutrition (PN) and notes that the catheter insertion site appears reddened. The nurse should *next* assess which item? A. Client's temperature B. Expiration date on the bag C. Time of last dressing change D. Tightness of tubing connections

A. Client's temperature

The nurse calls the health care provider (HCP) regarding a new medication prescription because the dosage prescribed is higher than the recommended dosage. The nurse is unable to locate the HCP, and the medication is due to be administered. Which action should the nurse take? A. Contact the nursing supervisor B. Administer the dose prescribed C. Hold the medication until the HCP can be contacted D. Administer the recommended dose until the HCP can be located

A. Contact the nursing supervisor

The nurse instructs a client with chronic kidney disease who is receiving hemodialysis about dietary modifications. The nurse determines that the client understands these dietary modifications. if the client selects which items from the dietary menu? A. Cream of wheat, blueberries, coffee B. Sausage and eggs, banana, orange juice C. Bacon, cantaloupe melon, tomato juice D. Cured pork, grits, strawberries, orange juice

A. Cream of wheat, blueberries, coffee

The nurse is creating a plan of care for the client with multiple myeloma and includes which *priority* intervention in the plan? A. Encouraging fluids B. Providing frequent oral care C. Coughing and deep breathing D. Monitoring the red blood cell count

A. Encouraging fluids Rationale: Hypercalcemia caused by bone destruction is a priority concern in the client with multiple myeloma. The nurse should administer fluids in adequate amounts to maintain a urine output of 1.5 to 2 L/day; this requires about 3 L of fluid intake per day. The fluid is needed not only to dilute the calcium overload but also to prevent protein from precipitating in the renal tubules. Options 2, 3, and 4 may be components of the plan of care but are not the priority in this client.

A critically ill Hispanic client tells the nurse through an interpreter that she is Roman Catholic and firmly believes in the rituals and traditions of the Catholic faith. Based on the client's statements, which actions by the nurse demonstrate cultural sensitivity and spiritual support? *Select all that apply.* A. Ensures that a close kin stays with the client B. Makes a referral for a Catholic priest to visit the client C. Removes the crucifix from the wall in the client's room D. Administers the sacrament of the sick to the client if death is imminent E. Offers to provide a means for praying the rosary if the client wishes F. Reminds the dietary department that meals served on Fridays during Lent do not contain meat

A. Ensures that a close kin stays with the client B. Makes a referral for a Catholic priest to visit the client E. Offers to provide a means for praying the rosary if the client wishes

The nurse is caring for a client with lung cancer and bone metastasis. What signs and symptoms would the nurse recognize as indications of a possible oncological emergency? *Select all that apply.* A. Facial edema in the morning B. Weight loss of 20 lb (9 kg) in 1 month C. Serum calcium level of 12 mg.dL (3.0 mmol/L) D. Serum sodium level of 136 mg/dL (136 mmol/L) E. Serum potassium level of 3.4 mg/dL (3.4 mmol/L) F. Numbness and tingling of the lower extremities

A. Facial edema in the morning C. Serum calcium level of 12 mg.dL (3.0 mmol/L) F. Numbness and tingling of the lower extremities

The nurse is reviewing the laboratory results of a client diagnosed with multiple myeloma. Which would the nurse expect to note specifically in this disorder? A. Increased calcium level B. Increased white blood cells C. Decreased blood urea nitrogen level D. Decreased number of plasma cells in the bone marrow

A. Increased calcium level Rationale: Findings indicative of multiple myeloma are an increased number of plasma cells in the bone marrow, anemia, hypercalcemia caused by the release of calcium from the deteriorating bone tissue, and an elevated blood urea nitrogen level. An increased white blood cell count may or may not be present and is not related specifically to multiple myeloma.

The nurse is monitoring the status of a postoperative client in the immediate postoperative period. The nurse would become *most* concerned with which sign that could indicate an evolving complication? A. Increasing restlessness B. A pulse of 86 beats/min C. Blood pressure of 110/70 mmHg D. Hypoactive bowel sounds in all 4 quadrants

A. Increasing restlessness

The clinic performing an admission assessment on a client votes the client is taking Azelaic acid period the nurse determines that which client complaint may be associated with the use of this medication? A. Itching B. Euphoria C. Drowsiness D. Frequent urination

A. Itching Rationale: Azelaic acid is a topical medication used to treat mild to moderate acne. Adverse effects include burning, itching, stinging, redness of the skin, and hypo pigmentation of the skin in clients with a dark complexion. the effects noted in the other options are not specifically associated with this medication.

The nurse reviews a client's laboratory report and notes that the client's serum phosphorus (phosphate) level is 1.8 mg/dL (0.45 mmol/L). Which condition *most likely* caused this serum phosphorus level? A. Malnutrition B. Renal insufficiency C. Hypoparathyroidism D. Tumor lysis syndrome

A. Malnutrition

The nurse notes that a client's arterial blood gas (ABG) results reveal a pH of 7.50 and a PaCO2 of 30 mmHg. The nurse monitors the client for which clinical manifestations associated with these ABG results? *Select all that apply.* A. Nausea B. Confusion C. Bradypnea D. Tachycardia E. Hyperkalemia F. Lightheadedness

A. Nausea B. Confusion D. Tachycardia F. Lightheadedness

Potassium chloride intravenously is prescribed for a client with hypokalemia. Which actions should the nurse take to plan for preparation and administration fo the potassium? *Select all that apply.* A. Obtain an intravenous (IV) infusion pump B. Monitor urine output during administration C. Prepare the medication for bolus administration D. Monitor the IV site for signs of infiltration or phlebitis E. Ensure that the medication is diluted in the appropriate volume of fluid F. Ensure that the bag is labeled so that it reads the volume of potassium in the solution

A. Obtain an intravenous (IV) infusion pump B. Monitor urine output during administration D. Monitor the IV site for signs of infiltration or phlebitis E. Ensure that the medication is diluted in the appropriate volume of fluid F. Ensure that the bag is labeled so that it reads the volume of potassium in the solution

The nurse is reviewing the history and physical examination of a client who will be receiving asparaginase (Elspar), an antineoplastic agent. The nurse contacts the health care provider before administering the medication if which disorder is documented in the client's history? A. Pancreatitis B. Diabetes mellitus C. Myocardial infarction D. Chronic obstructive pulmonary disease

A. Pancreatitis Rationale: Asparaginase (Elspar) is contraindicated if hypersensitivity exists, in pancreatitis, or if the client has a history of pancreatitis. The medication impairs pancreatic function and pancreatic function tests should be performed before therapy begins and when a week or more has elapsed between dose administrations. The client needs to be monitored for signs of pancreatitis, which include nausea, vomiting, and abdominal pain. The conditions noted in options 2, 3, and 4 are not contraindicated with this medication.

The nurse is conducting a history and monitoring laboratory values on a client with multiple myeloma. What assessment findings should the nurse expect to note? *Select all that apply.* A. Pathological fracture B. Urinalysis positive for nitrites C. Hemoglobin level of 15.5 g/dL (155 mmol/L) D. Calcium level of 8.6 mg/dL (2.15 mmol/L) E. Serum creatinine level of 2.0 mg/dL (176.6 mcmol/L)

A. Pathological fracture B. Urinalysis positive for nitrites E. Serum creatinine level of 2.0 mg/dL (176.6 mcmol/L)

The nurse is reviewing laboratory results and notes that a client's serum sodium level is 150 mEq/L (150 mmol/L). The nurse reports the serum sodium level to the health care provider (HCP) and the HCP prescribes dietary instructions based the sodium level. Which acceptable food items does the nurse instruct the client to consume? *Select all that apply.* A. Peas B. Nuts C. Cheese D. Cauliflower E. Processed oat cereals

A. Peas B. Nuts D. Cauliflower

Several laboratory tests are prescribed for a client, and the nurse reviews the results of the tests. Which laboratory test results should the nurse report? *Select all that apply.* A. Platelets 35,000 mm^3 (35 x 10^9/L) B. Sodium 150 mEq/L (150 mmol/L) C. Potassium 5.0 mEq/L (5.0 mmol/L) D. Segmented neutrophils 40% (0.40) E. Serum creatinine, 1 mg/dL (88.3 umol/L) F. White blood cells, 3,000 mm^3 (3.0 x 10^9/L)

A. Platelets 35,000 mm^3 (35 x 10^9/L) B. Sodium 150 mEq/L (150 mmol/L) D. Segmented neutrophils 40% (0.40) F. White blood cells, 3,000 mm^3 (3.0 x 10^9/L)

The nurse has just assisted a client back to bed after a fall. The nurse and health care provider have assessed the client and have determined that the client is not injured. After completing the incident report, the nurse should implement which action *next*? A. Reassess the client B. Conduct a staff meeting to describe the fall C. Document int the nurse's notes that an incident report was completed D. Contact the nursing supervisor to update information regarding the fall

A. Reassess the client

The ambulatory care nurse is discussing preoperative procedures with a Japanese/American client who is scheduled for surgery the following week. During the discussion, the client continually smiles and nods the head. How should the nurse interpret this nonverbal behavior? A. Reflecting a cultural value B. An acceptance of the treatment C. Client agreement to the required procedures D. Client understanding of the preoperative procedures

A. Reflecting a cultural value

A client rings the call light and complains of pain at the site of an intravenous (IV) infusion. The nurse assesses the site and determines that phlebitis has developed. The nurse should take which actions in the care of this client? *Select all that apply.* A. Remove the IV catheter at that site B. Apply warm moist packs to the site C. Notify the health care provider (HCP) D. Start a new IV line in a proximal portion of the same vein E. Document the occurrence, actions taken, and the client's response

A. Remove the IV catheter at that site B. Apply warm moist packs to the site C. Notify the health care provider (HCP) E. Document the occurrence, actions taken, and the client's response

The nurse is caring for a client with several broken ribs. The client is *most likely* to experience what type of acid-base imbalance? A. Respiratory acidosis from inadequate ventilation B. Respiratory alkalosis from anxiety and hyperventilation C. Metabolic acidosis from calcium loss due to broken bones D. Metabolic alkalosis from taking analgesics containing base products

A. Respiratory acidosis from inadequate ventilation

The nurse is preparing to care for a burn client scheduled for an escharotomy procedure being performed for a third-degree circumferential arm burn. The nurse understands that which finding is the anticipated therapeutic outcome of the escharotomy? A. Return of distal pulses B. Brisk bleeding from the site C. Decreasing edema formation D. Formation of granulation tissue

A. Return of distal pulses

A client had a 100-mL bag of 5% dextrose in 0.9% sodium chloride hung at 1500. The nurse making rounds at 1545 finds that the client is complaining of a pounding headache and is dyspneic, experiencing chills, and apprehensive, with an increased pulse rate. The intravenous (IV) bag has 400 mL remaining. The nurse should take which action *first?* A. Slow the IV infusion B. Sit the client up in bed C. Remove the IV catheter D. Call the health care provider (HCP)

A. Slow the IV infusion

The nurse is monitoring the intravenous (IV) infusion of an antineoplastic medication. During the infusion, the client complains of pain at the insertion site. On inspection of the site, the nurse notes redness and swelling and that the infusion of the medication has slowed in rate. The nurse suspects extravasation and should take which actions? *Select all that apply.* A. Stop the infusion B. Notify the health care provider (HCP) C. Prepare to apply ice or heat to the site D. Restart the IV at a distal part of the same vein E. Prepare to administer a prescribed antidote into the site F. Increase the flow rate of the solution to flush the skin and subcutaneous tissue

A. Stop the infusion B. Notify the health care provider (HCP) C. Prepare to apply ice or heat to the site E. Prepare to administer a prescribed antidote into the site Rationale: Redness and swelling and a slowed infusion indicate signs of extravasation. If the nurse suspects extravasation during the intravenous administration of an antineoplastic medication, the infusion is stopped and the HCP is notified. Ice or heat may be prescribed for application to the site and an antidote may be prescribed to be administered into the site. Increasing the flow rate can increase damage to the tissues. Restarting an IV in the same vein can increase damage to the site and vein.

The charge nurse is planning the assignment for the day. Which factors should the nurse remain mindful of when planning the assignment? *Select all that apply.* A. The acuity level of the clients B. Specific requests from the staff C. The clustering of the rooms on the unit D. The number of anticipated client discharged E. Client needs and workers' needs and abilities

A. The acuity level of the clients E. Client needs and workers' needs and abilities

A pregnant client tells the clinic nurse that she wants to know the sex of her baby as soon as it can be determined. The nurse informs the client that she should be able to find out the sex at 12 weeks' gestation because of which factor? A. The appearance of the fetal external genitalia B. The beginning of differentiation in the fetal groin C. The fetal testes are descended into the scrotal sac D. The internal differences in males and females become apparent

A. The appearance of the fetal external genitalia

Which identifies accurate nursing documentation notations? *Select all that apply.* A. The client slept through the night B. Abdominal wound dressing is dry and intact without drainage C. The client seemed angry when awakened for vital sign measurement D. The client appears to become anxious when it is time for respiratory treatments E. The client's left lower medial leg wound is 3 cm in length without redness, drainage, or edema

A. The client slept through the night B. Abdominal wound dressing is dry and intact without drainage E. The client's left lower medial leg wound is 3 cm in length without redness, drainage, or edema

The home health care nurse is caring for a client with cancer who is complaining of acute pain. The *most appropriate* determination of the client's pain should include which assessment? A. The client's pain rating B. Nonverbal cues from the client C. The nurse's impression of the client's pain D. Pain relief after appropriate nursing intervention

A. The client's pain rating

Salicylic acid is prescribed for a client with a diagnosis of psoriasis. the nurse monitors the client, knowing that which finding indicates the presence of systemic toxicity from this medication? A. Tinnitus B. Diarrhea C. Constipation D. Decreased respirations

A. Tinnitus Rationale: Salicylic acid is absorbed readily through the skin, and systemic toxicity can result period symptoms include tinnitus, dizziness, hyperpnea comma and psychological disturbances period constipation and diarrhea are not associated with salicylism.

The nurse is assessing a client with a suspected diagnosis of hypocalcemia. Which clinical manifestation would the nurse expect to note in the client? A. Twitching B. Hypoactive bowel sounds C. Negative Trousseau's sign D. Hypoactive deep tendon reflexes

A. Twitching

The nurse reviews a client's electrolyte laboratory report and notes that the potassium level is 2.5 mEq.L (2.5 mol/L). Which patterns should the nurse watch for on the electrocardiogram (ECG) as a result of the laboratory value? *Select all that apply.* A. U waves B. Absent P waves C. Inverted T waves D. Depressed ST segment E. Widened QRS complex

A. U waves C. Inverted T waves D. Depressed ST segment

The nurse has just reassessed the condition of a postoperative client who was admitted 1 hour ago to the surgical unit. The nurse plans to monitor which parameter most carefully during the next hour? A. Urinary output of 20 mL/hour B. Temperature of 37.6° C (99.6° F) C. Blood pressure of 100/70 mm Hg D. Serous drainage on the surgical dressing

A. Urinary output of 20 mL/hour

The nurse caring for a client who has been receiving intravenous (IV) diuretics suspects that the client is experiencing a fluid volume deficit. Which assessment finding would the nurse note in a client with this condition? A. Weight loss and poor skin turgor B. Lung congestion and increased heart rate C. Decreased hematocrit and increased urine output D. Increased respirations and increased blood pressure

A. Weight loss and poor skin turgor

A nurse enters the examination room of the emergency clinic and meets a 29-year-old patient who missed her last two follow-up appointments. The nurse notes from the medical record that the patient has high blood pressure that the doctor has been trying to help her manage. The patient just spoke with her doctor who left the room frustrated because the patient has not been taking her medication as prescribed. The patient confronts the nurse, saying, "I'm tired of being treated this way; no one cares. I need to find another doctor!" Using the C-LARA mnemonic, match the nurse's response to the correct letter of the mnemonic. Term Definition a. The nurse acknowledges that it is absolutely reasonable for patients to expect that their health care providers care about their situations and that it is disappointing when they have experiences that make them feel like they do not. b. The nurse uses a relaxation technique before responding to the patient's concerns. Calm yourself. Take a deep breath. Check your pulse. c. The nurse says, "I want to help you. I can do that better if you tell me what's making it difficult for you to come to your appointments and take your medicine each day." d. The nurse maintains eye contact and allows the patient to discuss her perspective while remaining attentively quiet. e. The nurse explains, "One thing I want you to understand is that your blood pressure medicine will only work if you take the same amount each day. Your follow-up appointments are important so we can get this blood pressure under control. Let me get a social worker who can help you figure out these transportation issues or see if he can find a doctor closer to your house." Definitions 1. C ____ 2. L ____ 3. A ____ 4. R ____ 5. A ____

A3 B1 C4 D2 E5

Match each letter of the RESPECT mnemonic with a statement that describes the concept the letter represents. Term Definition a. Ask about and try to understand barriers to care and adherence, and then offer resources to help the patient overcome them, involving family members if appropriate, and reassuring the patient that you are and will be available to help. b. Patients may have different reasons for not disclosing important information. Earn a patient's confidence through actions and attitude that demonstrate respect, compassion, and your interest in partnership. c. Work closely together with the patient by being flexible with regard to issues of control, negotiating roles when necessary, and stressing that you will be working together to mutually address medical problems. d. Provide explanations for the process and your action, checking often for understanding and using verbal clarification techniques such as Teach Back. e. Approach each encounter thinking about cultural competence and how you can demonstrate respect for the patient and his or her culture and beliefs. f. Approach the encounter with empathy, remembering that the patient has come to you for help. Seek out and understand the patient's rationale for his or her behaviors or illness, verbally acknowledging and legitimizing his or her feelings. g. Connect on a social level, seeking the patient's point of view; consciously attempt to suspend judgment; and avoid making assumptions. Definitions 1. R 2. E 3. S 4. P 5. E 6. C 7. T

A3 B7 C4 D5 E6 F2 G1

How can a nurse work on developing cultural awareness? (Select all that apply.) A. Reflect on his or her past learning about health, illness, race, gender, and sexual orientation B. Develop greater self-knowledge about personal biases C. Recognize consciously the multiple factors that influence his or her own world view D. Engage in an in-depth self-examination of his or her own background E. Learn as many facts as possible about an ethnic group

ABCD Remember that developing cultural awareness is a life-long process. It is a foundation of becoming culturally competent. Learning as many facts as possible about an ethnic group does not involve developing awareness about personal views, attitudes, and perceptions about ethnicit

Which of the following are considered social determinants of health? (Select all that apply.) a. Lack of primary health care providers in a zip code b. Poor-quality public school education that prevents a person from developing adequate reading skills c. Lack of affordable health insurance d. Employment opportunities that do not provide paid vacation or sick leave e. The number of times a person exercises during a week f. Neighborhood safety that prevents a person from walking around the block or socializing with neighbors outside of his or her home

ABCDF The social determinants of health are the circumstances in which people are born and grow up; the neighborhood in which they live, work, and age; and the systems put in place to deal with illness. These circumstances are in turn shaped by a wider set of forces: economics, social policies, and politics.

Which of the following changes can help create a more inclusive environment for lesbian, gay, bisexual, and transgender (LGBT) patients? (Select all that apply.) a. Explicitly including sexual orientation and gender identity into nondiscrimination policies b. Displaying art that reflects LGBT community c. Modifying health care forms to provide opportunities for gender identity and sexual orientation disclosure e. Not asking patients about their gender identity and sexual orientation to avoid making them uncomfortable f. Ensuring access to unisex or single-stall bathrooms

ABCE All of these help to create a more inclusive environment for LGBT patients. It is important to ask patients about their gender identity and sexual orientation to avoid making them uncomfortable.

A patient is admitted through the emergency department (ED) after a serious car accident. The nurse assesses the patient and quickly learns that he speaks little English. Spanish is his primary language. The nurse speaks some Spanish. Which interventions would be appropriate at this time? (Select all that apply.) The nurse requests a professional interpreter. Since this is an emergent situation, the nurse will interpret and identify the patient's priority needs. The nurse determines the interpreter's qualifications and makes sure that the interpreter can speak the patient's dialect. The nurse uses short sentences to explain the treatments provided in the ED. The nurse directs questions to the patient by looking at the patient instead of at the interpreter.

ACDE In any situation the nurse should use an interpreter and not the family to convey information to the patient. As the nurse you need to question the interpreter about his or her ability to speak the patient's dialect. It is your responsibility to introduce the interpreter to the patient. You are communicating with the patient and should direct your questions and responses to the patient and not the interpreter. Short sentences make it easier for the patient to understand complex information

Which of the following are examples of problems with the health care system that contribute to health disparities? (Select all that apply.) a. A health care provider assumes that the patient missed two appointments because the patient does not care about his or her health and does not inquire about the reasons for missed visits. b. The discharge nurse at a hospital uses Teach Back with a patient to ensure that she has communicated the discharge instructions clearly. c. A community hospital lacks an adequate staff of social workers who are able to ensure patients' access to resources they need to take care of their health. d A hospital discharges a patient without ensuring that the patient has a primary care provider and has made a follow-up appointment.e e. A nurse uses a family member as an interpreter to explain the patient's medications. f The hospital conducts quality improvement without stratifying data by race, ethnicity, language, socioeconomic status, sexual orientation, and other axes of social group identities.

ACDEF A large body of research shows that health care systems and health care providers contribute significantly to the problem of health disparities. Inadequate resources (lack of social workers), poor patient-provider communication (discharge instructions), a lack of culturally competent care, system fragmentation (not ensuring that the patient has a primary care provider or a follow-up appointment), and inadequate language access (use of interpreters) are critical factors that contribute to inequities in patient outcomes. RESPECT is the mnemonic for Rapport, Empathy, Support, Partnership, Explorations, Cultural competence, Trust

13. A patient in a motor vehicle accident states, "I did not run the red light," despite very clear evidence on the street surveillance tape. Which defense mechanism is the patient using? a. Denial b. Conversion c. Dissociation d. Compensation

ANS: A Denial consists of avoiding emotional conflicts by refusing to consciously acknowledge anything that causes intolerable emotional pain. Dissociation involves creating subjective numbness and less awareness of surroundings. Conversion involves repressing anxiety and manifesting it into nonorganic symptoms. Compensation occurs when an individual makes up for a deficit by strongly emphasizing another feature

12. A trauma survivor is requesting sleep medication because of "bad dreams." The nurse is concerned that the patient may be experiencing post-traumatic stress disorder (PTSD). Which question is a priority for the nurse to ask the patient? a. "Are you reliving your trauma?" b. "Are you having chest pain?" c. "Can you describe your phobias?" d. "Can you tell me when you wake up?"

ANS: A People who have PTSD often have flashbacks, recurrent and intrusive recollections of the event. The other answers involve assessment of problems not specific to PTSD.

14. A nurse is teaching the staff about the general adaptation syndrome. In which order will the nurse list the stages, beginning with the first stage? 1. Resistance 2. Exhaustion 3. Alarm a. 3, 1, 2 b. 3, 2, 1 c. 1, 3, 2 d. 1, 2, 3

ANS: A The general adaptation syndrome (GAS), a three-stage reaction to stress, describes how the body responds physiologically to stressors through stages of alarm, resistance, and exhaustion.

20. A nurse is helping parents who have a child with attention-deficit/hyperactivity disorder. Which strategy will the nurse share with the parents to reduce stress regarding homework assignments? a. Time-management skills b. Speech articulation skills c. Routine preventative health visits d. Assertiveness training for the family

ANS: A Time-management skills are most related to homework assignment completion. Time-management techniques include developing lists of prioritized tasks. Routine health visits are important but do not directly affect ability to complete homework. Speech and other developmental aspects need to be developed if the child is to be successful, but skill development will not directly reduce homework-related stress. Assertiveness includes skills for helping individuals communicate effectively regarding their needs and desires, but it does not help with homework assignments.

13. Which factors should the nurse assess to determine a patient's ability to learn? a. Developmental capabilities and physical capabilities b. Sociocultural background and motivation c. Psychosocial adaptation to illness and active participation d. Stage of grieving and overall physical health

ANS: A Developmental and physical capabilities reflect one's ability to learn. Sociocultural background and motivation are factors in readiness to learn. Psychosocial adaptation to illness and active participation are factors in readiness to learn. Readiness to learn is related to the stage of grieving. Overall physical health does reflect ability to learn; however, because it is paired here with stage of grieving (which is a readiness to learn factor), this is a wrong answer.

A nurse is preparing to teach a kinesthetic learner about exercise. Which technique should the nurse use? a. Let the patient touch and use the exercise equipment. b. Provide the patient with pictures of the exercise equipment. c. Let the patient listen to a podcast about the exercise equipment. d. Provide the patient with a case study about the exercise equipment.

ANS: A Kinesthetic learners learn best while they are moving and participating in hands-on activities. Demonstrations and role playing work well with these learners. Patients who are visual learners learn best when you use pictures and diagrams to explain information. Patients who prefer auditory learning are distracted by pictures and prefer listening to information (e.g., podcasts). Patients who learn best by reasoning logically and intuitively learn better if presented with a case study that requires careful analysis and discussion with others to arrive at conclusions.

8. While preparing a teaching plan, the nurse described what the learner will be able to accomplish after the teaching session. Which action did the nurse complete? a. Developed learning objectives b. Provided positive reinforcement c. Implemented interpersonal communication d. Presented facts and knowledge

ANS: A Learning objectives describe what the learner will be able to do after successful instruction. Positive reinforcement follows feedback and involves the use of praise and acknowledgment of new attitudes, behaviors, or knowledge. Interpersonal communication is necessary for the teaching/learning process, but describing what the learner will be able to do after successful instruction constitutes learning objectives. Facts and knowledge will be presented in the teaching session.

3. A nurse provides teaching about coping with long-term impaired functions. Which situation serves as the best example? a. Teaching a family member to give medications through the patient's permanent gastric tube b. Teaching a woman who recently had a hysterectomy about her pathology reports c. Teaching expectant parents about physical and psychological changes in childbearing women d. Teaching a teenager with a broken leg how to use crutches

ANS: A Not all patients fully recover from illness or injury. Many have to learn to cope with permanent health alterations. New knowledge and skills are often necessary for patients and/or family members to continue activities of daily living. Teaching family members to help the patient with health care management (e.g., giving medications through gastric tubes, doing passive range-of-motion exercises) is an example of coping with long-term impaired functions. Injured and ill patients need information and skills to help them regain or maintain their levels of health. Some examples of this include teaching a woman who recently had a hysterectomy about her pathology reports and expected length of recovery and teaching a teenager with a broken leg how to use crutches. In childbearing classes, you teach expectant parents about physical and psychological changes in the woman and about fetal development; this is part of health maintenance.

20. Which learning objective/outcome has the highest priority for a patient with life-threatening, severe food allergies that require an EpiPen (epinephrine)? a. The patient will administer epinephrine. b. The patient will identify the main ingredients in several foods. c. The patient will list the side effects of epinephrine. d. The patient will learn about food labels.

ANS: A Once you assist in meeting patient needs related to basic survival (how to give epinephrine), you can discuss other topics, such as nutritional needs and side effects of medications. For example, a patient recently diagnosed with coronary artery disease has deficient knowledge related to the illness and its implications. The patient benefits most by first learning about the correct way to take nitroglycerin and how long to wait before calling for help when chest pain occurs. Thus, in this situation, the patient benefits most by first learning about the correct way to take epinephrine. "The patient will learn about food labels" is not objective and measurable and is not correctly written.

A patient had a stroke and must use a cane for support. A nurse is preparing to teach the patient about the cane. Which learning objective/outcome is most appropriate? a. The patient will walk to the bathroom and back to bed using a cane. b. The patient will understand the importance of using a cane. c. The patient will learn how to use a cane. d. The patient will know the correct use of a cane.

ANS: A Outcomes often describe a behavior that identifies the patient's ability to do something on completion of teaching such as will empty colostomy bag, or will administer an injection. Understand, learn, and know are not behaviors that can be observed or evaluated.

11. When the nurse describes a patient's perceived ability to successfully complete a task, which term should the nurse use? a. Self-efficacy b. Motivation c. Attentional set d. Active participation

ANS: A Self-efficacy, a concept included in social learning theory, refers to a person's perceived ability to successfully complete a task. Motivation is a force that acts on or within a person (e.g., an idea, an emotion, a physical need) to cause the person to behave in a particular way. An attentional set is the mental state that allows the learner to focus on and comprehend a learning activity. Learning occurs when the patient is actively involved in the educational session.

A patient has been taught how to cough and deep breathe. Which evaluation method is most appropriate? a. Return demonstration b. Computer instruction c. Verbalization of steps d. Cloze test

ANS: A To demonstrate mastery of the skill, have the patient perform a return demonstration under the same conditions that will be experienced at home or in the place where the skill is to be performed. Computer instruction is use of a programmed instruction format in which computers store response patterns for learners and select further lessons on the basis of these patterns (programs can be individualized). Computer instruction is a teaching tool, rather than an evaluation tool. Verbalization of steps can be an evaluation tool, but it is not as effective as a return demonstration when evaluating a psychomotor skill. The Cloze test, a test of reading comprehension, asks patients to fill in the blanks in a written paragraph

1. A nurse is assessing a patient with prolonged stress. Which conditions will the nurse monitor for in this patient? (Select all that apply.) a. Cancer b. Diabetes c. Infections d. Allostasis e. Low blood pressure

ANS: A, B, C Stress causes prolonged changes in the immune system, which can result in impaired immune function, and this increases the person's susceptibility to changes in health, such as increased risk for infection, high blood pressure, diabetes, and cancers. Allostasis is a return to a state of balance; allostatic load occurs with prolonged stress.

Which statements by the nurse indicate a good understanding of patient education/teaching? (Select all that apply.) a. "Patient education is a standard for professional nursing practice." b. "Patient teaching falls within the scope of nursing practice." c. "Patient education is an essential component of safe, patient-centered care." d. "Patient education is not effective with children." e. "Patient teaching can increase health care costs." f. "Patient teaching should be documented in the chart."

ANS: A, B, C, F Patient education has long been a standard for professional nursing practice. All state Nurse Practice Acts acknowledge that patient teaching falls within the scope of nursing practice. Patient education is an essential component of providing safe, patient-centered care. It is important to document evidence of successful patient education in patients' medical records. Patient education is effective for children. Different techniques must be used with children. Creating a well-designed, comprehensive teaching plan that fits a patient's unique learning needs reduces health care costs, improves the quality of care, and ultimately changes behaviors to improve patient outcomes.

9. A senior college student visits the college health clinic about a freshman student living on the same dormitory floor. The senior student reports that the freshman is crying and is not adjusting to college life. The clinic nurse recognizes this as a combination of situational and maturational stress factors. Which is the best response by the nurse? a. "Let's call 911 because this freshman student is suicidal." b. "Give the freshman student this list of university and community resources." c. "I recommend that you help the freshman student start packing bags to go home." d. "You must make an appointment for the freshman student to obtain medications."

ANS: B A nurse can help reduce situational stress factors for individuals. Inform the patient about potential resources. Providing the student with a list of resources is one way to begin this process, as part of secondary prevention strategies. This is not a medical or psychiatric emergency, so calling 911 is not necessary. Not everyone who has sadness needs medications; some need counseling only. Not enough information is given to know whether the student would be best suited to leave college.

5. After a natural disaster occurred, an emergency worker referred a family for crisis intervention services. One family member refused to attend the services, stating, "No way, I'm not crazy." What is the nurse's best response? a. "Many times disasters can create mental health problems, so you really should participate with your family." b. "Seeking this kind of help does not mean that you have a mental illness; it is a short-term problem-solving technique." c. "Don't worry now. The psychiatrists are well trained to help." d. "This will help your family communicate better."

ANS: B Crisis intervention is a type of brief therapy that is more directive than traditional psychotherapy or counseling. It focuses on problem solving and involves only the problem created by the crisis. The other options do not properly reassure the patient and build trust. Giving advice in the form of "you really should participate" is inappropriate. "Don't worry now" is false reassurance. While crisis intervention may help families communicate better, the goal is to return to precrisis level of functioning; family therapy will focus on helping families communicate better

2. A woman who was sexually assaulted a month ago presents to the emergency department with reports of recurrent nightmares, fear of going to sleep, repeated vivid memories of the sexual assault, and inability to feel much emotion. Which medical problem will the nurse expect to see documented in the chart? a. General adaptation syndrome b. Post-traumatic stress disorder c. Acute stress disorder d. Alarm reaction

ANS: B Post-traumatic stress disorder is characterized by vivid recollections of the traumatic event and emotional detachment and often is accompanied by nightmares. General adaptation syndrome is the expected reaction to a major stressor. Acute stress disorder is a similar diagnosis that differs from PTSD in duration of symptoms. Alarm reaction involves physiological events such as increased activation of the sympathetic nervous system that would have occurred at the time of the sexual assault.

6. A preadolescent patient is experiencing maturational stress. Which area will the nurse focus on when planning care? a. Identity issues b. Self-esteem issues c. Physical appearance d. Major changing life events

ANS: B Preadolescents experience stress related to self-esteem issues, changing family structure as a result of divorce or death of a parent, or hospitalizations. Adolescent stressors include identity issues with peer groups and separation from their families. Children identify stressors related to physical appearance, families, friends, and school. Adult stressors centralize around major changes in life circumstances.

10. Despite working in a highly stressful nursing unit and accepting additional shifts, a new nurse has a strategy to prevent burnout. Which strategy will be best for the nurse to use? a. Delegate complex nursing tasks to nursing assistive personnel. b. Strengthen friendships outside the workplace. c. Write for 10 minutes in a journal every day. d. Use progressive muscle relaxation.

ANS: B Strengthening friendships outside of the workplace, arranging for temporary social isolation for personal "recharging" of emotional energy, and spending off-duty hours in interesting activities all help reduce burnout. Journaling and muscle relaxation are good stress-relieving techniques but are not directed at the cause of the workplace stress. Delegating complex nursing tasks to nursing assistive personnel is an inappropriate.

A patient with heart failure is learning to reduce salt in the diet. When would be the best time for the nurse to address this topic? a. At bedtime, when the patient is relaxed b. At lunchtime while the nurse is preparing the food tray c. At bath time, when the nurse is cleaning the patient d. At medication time, when the nurse is administering patient medication

ANS: B Appropriate times to talk about food/diet changes during routine nursing care are at breakfast, lunch, and dinner times or when the patient is completing the menu. Many nurses find that they are able to teach more effectively while delivering nursing care. For example, while hanging blood, you explain to the patient why the blood is necessary and the symptoms of a transfusion reaction that need to be reported immediately. In this situation, because the teaching is about food, coordinating it with routine nursing care that involves food can be effective. At bedtime would be a good time to discuss routines that enhance sleep. At bath time would be a good time to describe skin care and how to prevent pressure ulcers. At medication time would be a good time to explain the purposes and side effects of the medication.

17. Which nursing action is most appropriate for assessing a patient's learning needs? a. Assess the patient's total health care needs. b. Assess the patient's health literacy. c. Assess all sources of patient data. d. Assess the goals of patient care.

ANS: B Because health literacy influences how you deliver teaching strategies, it is critical for you to assess a patient's health literacy before providing instruction. The nursing process requires assessment of all sources of data to determine a patient's total health care needs. Evaluation of the teaching process involves determining outcomes of the teaching/learning process and the achievement of learning objectives, not patient care. Assessing the goal of meeting patient care is the evaluation component of the nursing process.

9. A student nurse learns that a normal adult heartbeat is 60 to 100 beats/minute. In which domain did learning take place? a. Kinesthetic b. Cognitive c. Affective d. Psychomotor

ANS: B Cognitive learning includes all intellectual behaviors and requires thinking. In the hierarchy of cognitive behaviors, the simplest behavior is acquiring knowledge. The student nurse acquired knowledge, which is cognitive. Kinesthetic is a type of learner who learns best with a hands-on approach. Affective learning deals with expression of feelings and acceptance of attitudes, opinions, or values. Psychomotor learning involves acquiring skills that require integration of mental and muscular activities, such as the ability to walk or to use an eating utensil.

6. A nurse is teaching a patient about the Speak Up Initiatives. Which information should the nurse include? a. The nurse is the center of the health care team. b. If you still do not understand, ask again. c. Ask a nurse to be your advocate or supporter. d. Inappropriate medical tests are the most common mistakes.

ANS: B If you still do not understand, ask again is part of the S portion of the Speak Up Initiatives. Speak up if you have questions or concerns. You (the patient) are the center of the health care team, not the nurse. Ask a trusted family member or friend to be your advocate (advisor or supporter), not a nurse. Medication errors are the most common health care mistakes, not inappropriate medical tests.

A nurse is teaching a patient who has low health literacy about chronic obstructive pulmonary disease (COPD) while giving COPD medications. Which technique is most appropriate for the nurse to use? a. Use complex analogies to describe COPD. b. Include the most important information on COPD at the beginning of the session. c. Ask for feedback to assess understanding of COPD at the end of the session. d. Offer pamphlets about COPD written at the eighth grade level with large type

ANS: B Include the most important information at the beginning of the session for illiterate patients or patients with a learning disability. Also, use visual cues and simple, not complex, analogies when appropriate. Another technique is to frequently ask patients for feedback to determine whether they comprehend the information. Additionally, provide teaching materials that reflect the reading level of the patient, with attention given to short words and sentences, large type, and simple format (generally, information written on a fifth grade reading level is recommended for adult learners).

5. Which action best indicates that learning has occurred? a. A nurse presents information about diabetes. b. A patient demonstrates how to inject insulin. c. A family member listens to a lecture on diabetes. d. A primary care provider hands a diabetes pamphlet to the patient.

ANS: B Learning is the purposeful acquisition of new knowledge, attitudes, behaviors, and skills. Complex patterns are required if the patient is to learn new skills, change existing attitudes, transfer learning to new situations, or solve problems. A new mother exhibits learning when she demonstrates how to bathe her newborn. A nurse presenting information and a primary care provider handing a pamphlet to a patient are examples of teaching. A family member listening to a lecture does not indicate that learning occurred; a change in knowledge, attitudes, behaviors, and/or skills must be evident.

A patient has been taught how to change a colostomy bag but is having trouble measuring and manipulating the equipment and has many questions. What is the nurse's next action? a. Refer to a mental health specialist. b. Refer to an ostomy specialist. c. Refer to a dietitian. d. Refer to a wound care specialist.

ANS: B Resources that specialize in a particular health need (e.g., wound care or ostomy specialists) are integral to successful patient education. A mental health specialist is helpful for emotional issues rather than for physical problems. A dietitian is a resource for nutritional needs. A wound care specialist provides complex wound care.

1. A nurse is asked about the goal of patient education. What is the nurse's best response? The goal of educating others is to help people a. Meet standards of the Nurse Practice Act. b. Achieve optimal levels of health. c. Become dependent on the health care team. d. Provide self-care only in the hospital.

ANS: B The goal of educating others about their health is to help individuals, families, or communities achieve optimal levels of health. Although all state Nurse Practice Acts acknowledge that patient teaching falls within the scope of nursing practice, this is the nurse's standard, not the goal of education. Patient education helps patients make informed decisions about their care and become healthier and more independent, not dependent. Nurses provide patients with information needed for self-care to ensure continuity of care from the hospital to the home.

14. A nurse is preparing to teach a patient about heart failure. Which environment is best for patient learning? a. A darkened, quiet room b. A well-lit, ventilated room c. A private room at 85 F temperature d. A group room for 10 to 12 patients with heart failure

ANS: B The ideal environment for learning is a room that is well lit and has good ventilation, appropriate furniture, and a comfortable temperature. Although quiet is appropriate, a darkened room interferes with the patient's ability to watch your actions, especially when you are demonstrating a skill or using visual aids such as posters or pamphlets. A room that is cold, hot, or stuffy makes the patient too uncomfortable to focus on the information being presented. Learning in a group of six or fewer is more effective than in larger groups and avoids outburst behaviors.

2. Which situation will cause the nurse to postpone a teaching session? (Select all that apply.) a. The patient is mildly anxious. b. The patient is fatigued. c. The patient is asking questions. d. The patient is hurting. e. The patient is febrile (high fever). f. The patient is in the acceptance phase.

ANS: B, D, E Any condition (e.g., pain, fatigue) that depletes a person's energy also impairs his or her ability to learn, so the session should be postponed until the pain is relieved and the patient is rested. Postpone teaching when an illness becomes aggravated by complications such as a high fever or respiratory difficulty. A mild level of anxiety motivates learning. When patients are ready to learn, they frequently ask questions. When the patient enters the stage of acceptance, the stage compatible with learning, introduce a teaching plan.

18. A nurse is planning care for a patient that uses displacement. Which information should the nurse consider when planning interventions? a. This copes with stress directly. b. This evaluates an event for its personal meaning. c. This protects against feelings of worthlessness and anxiety. d. This triggers the stress control functions of the medulla oblongata.

ANS: C Ego-defense mechanisms, like displacement, regulate emotional distress and thus give a person protection from anxiety and stress. Everyone uses them unconsciously to protect against worthlessness and feelings of anxiety. Ego-defense mechanisms help a person cope with stress indirectly and offer psychological protection from a stressful event. Evaluation of an event for its personal meaning is primary appraisal. The medulla oblongata controls heart rate, blood pressure, and respirations and is not triggered by ego defense mechanisms.

19. Which sociocultural finding in the history of a patient will alert the nurse to a possible developmental problem? a. Family relocation b. Childhood obesity c. Prolonged poverty d. Loss of stamina

ANS: C Environmental and social stressors often lead to developmental problems. Sociocultural refers to societal or cultural factors; poverty is a sociocultural factor. Stamina loss and obesity are health problems, and family relocation is a situational factor.

1. In a natural disaster relief facility, the nurse observes that an older-adult male has a recovery plan, while a 25-year-old male is still overwhelmed by the disaster situation. A nurse is planning care for both patients. Which factors will the nurse consider about the different coping reactions? a. Restorative care factors b. Strong financial resource factors c. Maturational and situational factors d. Immaturity and intelligence factors

ANS: C Maturational factors and situational factors can affect people differently depending on their life experiences. An older individual would have more life experiences to draw from and to analyze on why he was successful, whereas a younger individual would have fewer life experiences based on chronological age to analyze for patterns of previous success. Nothing in the scenario implies that either man is in restorative care, has strong financial resources, or is immature or intelligent.

15. A young male patient is diagnosed with testicular cancer. Which action will the nurse take first? a. Provide information to the patient. b. Allow time for the patient's friends. c. Ask about the patient's priority needs. d. Find support for the family and patient.

ANS: C Take time to understand a patient's meaning of the precipitating event and the ways in which stress is affecting his life. For example, in the case of a woman who has just been told that a breast mass was identified on a routine mammogram, it is important to know what the patient wants (priority needs) and needs most from the nurse. Providing information, allowing time with friends, and finding support may be implemented after finding out what the patient wants or needs.

3. The nurse teaches stress-reduction and relaxation training to a health education group of patients after cardiac bypass surgery. Which level of intervention is the nurse using? a. Primary b. Secondary c. Tertiary d. Quad

ANS: C Tertiary-level interventions assist the patient in readapting and can include relaxation training and time-management training. At the primary level of prevention, you direct nursing activities to identifying individuals and populations who are possibly at risk for stress. Nursing interventions at the secondary level include actions directed at symptoms such as protecting the patient from self-harm. Quad level does not exist.

16. A nurse is teaching the staff about a nursing theory that views a person, family, or community developing a normal line of defense. Which theory is the nurse describing? a. Ego defense model b. Immunity model c. Neuman Systems Model d. Pender's Health Promotion Model

ANS: C The Neuman Systems Model uses a systems approach, and it helps you understand your patients' individual responses to stressors and also families' and communities' responses. Every person develops a set of responses to stress that constitute the "normal line of defense." This line of defense helps to maintain health and wellness. Ego defense mechanisms are unconscious coping mechanisms. Immunity is a body's natural protection mechanism. Pender's Health Promotion Model focuses on promoting health and managing stress.

8. An adult male reports new-onset, seizure-like activity. An EEG and a neurology consultant's report rule out a seizure disorder. It is determined the patient is using conversion. Which action should the nurse take next? a. Suggest acupuncture. b. Confront the patient on malingering. c. Obtain history of any recent life stressors. d. Recommend a regular exercise program.

ANS: C Unconsciously repressing an anxiety-producing emotional conflict and transforming it into nonorganic symptoms (e.g., difficulty sleeping, loss of appetite) describes conversion. The nurse must assess the patient fully for emotional conflict and stress before implementing any nursing interventions (acupuncture or exercise program). Although the patient may be malingering, confrontation is nontherapeutic because the patient is using this type of defense mechanism in response to some type of stressor.

A nurse has taught a patient about healthy eating habits. Which learning objective/outcome is most appropriate for the affective domain? a. The patient will state three facts about healthy eating. b. The patient will identify two foods for a healthy snack. c. The patient will verbalize the value of eating healthy. d. The patient will cook a meal with low-fat oil.

ANS: C Affective learning deals with expression of feelings and acceptance of attitudes, opinions, or values. Having the patient value healthy eating habits falls within the affective domain. Stating three facts or identifying two foods for a healthy snack falls within the cognitive domain. Cooking falls within the psychomotor domain

10. A nurse wants the patient to begin to accept the chronic nature of diabetes. Which teaching technique should the nurse use to enhance learning? a. Lecture b. Demonstration c. Role play d. Question and answer session

ANS: C Affective learning deals with expression of feelings and acceptance of attitudes, opinions, or values. Role play and discussion (one-on-one and group) are effective teaching methods for the affective domain. Lecture and question and answer sessions are effective teaching methods for the cognitive domain. Demonstration is an effective teaching method for the psychomotor domain.

2. A nurse is teaching a group of healthy adults about the benefits of flu immunizations. Which purpose of patient education is the nurse fulfilling? a. Restoration of health b. Coping with impaired functions c. Promotion of health and illness prevention d. Health analogies

ANS: C As a nurse, you are a visible, competent resource for patients who want to improve their physical and psychological well-being. In the school, home, clinic, or workplace, you promote health and prevent illness by providing information and skills that enable patients to assume healthier behaviors. Injured and ill patients need information and skills to help them regain or maintain their level of health; this is referred to as restoration of health. Not all patients fully recover from illness or injury. Many have to learn to cope with permanent health alterations; this is known as coping with impaired functions. Analogies supplement verbal instruction with familiar images that make complex information more real and understandable. For example, when explaining arterial blood pressure, use an analogy of the flow of water through a hose.

A nurse is going to teach a patient about hypertension. Which action should the nurse implement first? a. Set mutual goals for knowledge of hypertension. b. Teach what the patient wants to know about hypertension. c. Assess what the patient already knows about hypertension. d. Evaluate the outcomes of patient education for hypertension.

ANS: C Assessment is the first step of any teaching session, then diagnosing, planning, implementation, and evaluation. An effective assessment provides the basis for individualized patient teaching. Assessing what the adult patient currently knows improves the outcomes of patient education.

A nurse is teaching an older adult patient about strokes. Which teaching technique is most appropriate for the nurse to use? a. Use a pamphlet about strokes with large font in blues and greens. b. Speak in a high tone of voice to describe strokes. c. Begin and end each teaching session with the most important information about strokes. d. Provide specific information about strokes in frequent, large amounts.

ANS: C Begin and end each teaching session with the most important information when teaching older adult patients. Also, if using written material, assess the patient's ability to read and use information that is printed in large type and in a color that contrasts highly with the background (e.g., black 14-point print on buff-colored paper). Avoid blues and greens because they are more difficult to see. Speak in a low tone of voice (lower tones are easier to hear than higher tones). Provide specific information in frequent, small (not large) amounts for older adult patients.

A nurse is teaching a culturally diverse patient about nutritional needs. What must the nurse do first before starting the teaching session? a. Obtain pictures of food. b. Get an interpreter. c. Establish a rapport. d. Refer to a dietitian.

ANS: C Establishing a rapport is important for all patients, especially culturally diverse patients, before starting teaching sessions. Obtaining pictures of food, getting an interpreter, and referring to a dietitian all occur after rapport is established.

15. Which situation indicates to the nurse that the patient is ready to learn? a. A patient has sufficient upper body strength to move from a bed to a wheelchair. b. A patient has the ability to grasp and apply the elastic bandage. c. A patient with a below-the-knee amputation is motivated about how to walk with assistive devices. d. A patient has normal eyesight to identify the markings on a syringe and coordination to handle a syringe.

ANS: C Motivation or readiness to learn sometimes results from social task mastery, or physical motives may be involved. Often patient motives are physical. Some patients are motivated to return to a level of physical normalcy. For example, a patient with a below-the-knee amputation is motivated to learn how to walk with assistive devices. Do not confuse readiness to learn with ability to learn. All the other answers are examples of ability to learn because this often depends on the patient's level of physical development and overall physical health. To learn psychomotor skills, a patient needs to possess a certain level of strength, coordination, and sensory acuity. For example, it is useless to teach a patient to transfer from a bed to a wheelchair if he or she has insufficient upper body strength. An older patient with poor eyesight or an inability to grasp objects tightly cannot learn to apply an elastic bandage or handle a syringe.

After a teaching session on taking blood pressures, the nurse tells the patient, "You took that blood pressure like an experienced nurse." What type of reinforcement did the nurse use? a. Material b. Activity c. Social d. Entrusting

ANS: C Three types of reinforcers are social, material, and activity. When a nurse works with a patient, most reinforcers are social and are used to acknowledge a learned behavior (e.g., smiles, compliments, words of encouragement). Examples of material reinforcers include food, toys, and music. Activity reinforcers rely on the principle that a person is motivated to engage in an activity if he or she has the opportunity to engage in a more desirable activity after completion of the task. The entrusting approach is a teaching approach that provides the patient the opportunity to manage self-care. It is not a type of reinforcement.

12. A toddler is going to have surgery on the right ear. Which teaching method is most appropriate for this developmental stage? a. Encourage independent learning. b. Use discussion throughout the teaching session. c. Apply a bandage to a doll's ear. d. Develop a problem-solving scenario.

ANS: C Use play to teach a procedure or activity (e.g., handling examination equipment, applying a bandage to a doll) to toddlers. Encouraging independent learning is for the middle-aged adult. Use of discussion is for older children, adolescents, and adults, not for toddlers. Use problem solving to help adolescents make choices. Problem solving is too advanced for a toddler.

17. An adult who was in a motor vehicle accident is brought into the emergency department by paramedics, who report the following in-transit vital signs: Oral temperature: 99.0° F Pulse: 102 beats/min Respiratory rate: 26 breaths/min Blood pressure: 140/106 Which hormones should the nurse consider as the most likely causes of the abnormal vital signs? a. ADH and ACTH b. ACTH and epinephrine c. ADH and norepinephrine d. Epinephrine and norepinephrine

ANS: D During the alarm stage, rising hormone levels result in increased blood volume, blood glucose levels, epinephrine and norepinephrine amounts, heart rate, blood flow to muscles, oxygen intake, and mental alertness. ACTH originates from the anterior pituitary gland and stimulates cortisol release; ADH originates from the posterior pituitary and increases renal reabsorption of water. ACTH, cortisol, and ADH do not increase heart rate.

4. A nurse is teaching guided imagery to a prenatal class. Which technique did the nurse describe? a. Singing b. Massaging back c. Listening to music d. Using sensory peaceful words

ANS: D Guided imagery is used as a means to create a relaxed state through the person's imagination, often using sensory words. Imagination allows the person to create a soothing and peaceful environment. Singing, back massage, and listening to music are other types of stress management techniques.

11. A female teen with celiac disease continues to eat food she knows will make her ill several hours after ingestion. While planning care, the nurse considers maturational and tertiary-level interventions. Which intervention will the nurse add to the care plan? a. Teach the teen about the food pyramid. b. Administer antidiarrheal medications with meals. c. Gently admonish the teen and her parents regarding the consistently poor diet choices. d. Assist the teen in meeting dietary restrictions while eating foods similar to those eaten by her friends.

ANS: D Tertiary-level interventions assist the patient in readapting to life with an illness. By adjusting the diet to meet dietary guidelines and also addressing adolescent maturational needs, the nurse will help the teen to eat an appropriate diet without health complications and see herself as a "typical and normal" teenager. Teaching about the food pyramid will not address the real issue, which is that the teen is still eating what she knows will make her ill and the food pyramid is usually a primary intervention. Administering antidiarrheal medications may help but is not a tertiary-level or maturational intervention. Admonishing the teen and parents is not a tertiary-level intervention, and because this approach is nontherapeutic, it may cause communication problems.

A nurse is assessing the ability to learn of a patient who has recently experienced a stroke. Which question/statement will best assess the patient's ability to learn? a. "What do you want to know about strokes?" b. "On a scale from 1 to 10, tell me where you rank your desire to learn." c. "Do you feel strong enough to perform the tasks I will teach you?" d. "Please read this handout and tell me what it means."

ANS: D A patient's reading level affects ability to learn. Reading level is often difficult to assess because patients who are functionally illiterate are often able to conceal it by using excuses such as not having the time or not being able to see. One way to assess a patient's reading level and level of understanding is to ask the patient to read instructions from an educational handout and then explain their meaning. Asking patients what they want to know identifies previous learning and learning needs and preferences; it does not assess ability to learn. Motivation is related to readiness to learn, not ability to learn. Just asking a patient if they feel strong is not as effective as actually assessing the patient's streng

7. A nurse teaches a patient with heart failure healthy food choices. The patient states that eating yogurt is better than eating cake. In this situation, which element represents feedback? a. The nurse b. The patient c. The nurse teaching about healthy food choices d. The patient stating that eating yogurt is better than eating cake

ANS: D Feedback should show the success of the learner in achieving objectives (i.e., the learner verbalizes information or provides a return demonstration of skills learned). The nurse is the sender. The patient is the receiver. The teaching is the message.

16. A nurse is teaching the staff about nursing and teaching processes. Which information should the nurse include regarding the teaching process? During the teaching process, what should the nurse do? a. Assess all sources of data. b. Identify that it is the same as the nursing process. c. Perform nursing care therapies. d. Focus on a patient's learning needs.

ANS: D The teaching process focuses on the patient's learning needs and willingness and capability to learn. Nursing and teaching processes are not the same. All the rest are components of the nursing process: Assess all sources of data and perform nursing care therapies

a nurse is caring for a patient with end stage lung disease. the patient wants to go home on oxygen and be comfortable. the family wants the patient tot have a new surgical procedure. the nurse explains the risk and benefits of the surgery to the family and discusses the patients wishes with them. the nurse is acting as the patients - Educator - Advocate - Caregiver - Case Manager

Advocate

A patient states that he does not believe in a higher power but instead believes that people bring meaning to what they do. This patient most likely is an

Agnostic.

Patient Confidentiality

All patients have a right to privacy and all information should remain privileged. Discuss patient information only with the patient's physician or office personnel that need certain information to do their job. Obtain a signed consent form to release medical information to the insurance company or other individual.

On the basis of an assessment, the nurse identifies an increase in the immigrant population group in the community. How would he or she determine some of the health needs of this population? (Select all that apply.) 1. Identify which two health needs the immigrant population views as most important 2. Apply information from Healthy People 2020 3. Determine how the population uses available health care resources 4. Identify perceived barriers for health care 5. Implement an exercise program to help with weight loss

Answer: 1, 2, 3, 4. All of these assess the health care needs of this population. It is necessary to identify their priorities and try to meet them, applying information from reliable guidelines such as Healthy People 2020. How or if a population uses available health care resources and the perceived barriers for health care are all important. These elements help identify factors that promote or impede health care for this group.

Using Healthy People 2020 as a guide, which of the following would improve delivery of care to a community? (Select all that apply.) 1. Community assessment 2. Implementing public health policies 3. Increasing access to care 4. Determining rates of specific illnesses 5. Reducing the number of fast food restaurants in the community

Answer: 1, 2, 3, 4. Improved delivery of health care occurs through assessment of health care needs of individuals, families, and communities; development and implementation of public health policies; and improved access to care. For example, assessment includes systematic data collection on the population, monitoring the health status of the population, and accessing available information about the health of the community.

The nurse is working with the county health department on a task force to fully integrate the goals of Healthy People 2020. How does the nurse determine which goals need to be included or updated? (Select all that apply.) 1. Assesses the health care resources within the community 2. Assesses the existing health care programs offered by the county health department 3. Compares existing resources and programs with Healthy People 2020 goals 4. Initiates new programs to meet Healthy People 2020 goals.

Answer: 1, 2, 3. The nurse must first assess for existing health care resources and educational programs in existence. Then he or she must compare these resources and programs with Healthy People 2020 goals. These processes determine if any new goals need to be added or updated.

A nursing student in the last semester of the baccalaureate nursing program is beginning the community health practicum and will be working in a community-based clinic with a focus on asthma and allergies. What is the focus of the community health nurse in this clinic setting? (Select all that apply.) 1. Decreasing the incidence of asthma attacks in the community 2. Increasing healthy food choices for school lunches 3. Assessing for factors that contribute to asthmatic attacks in the community 4. Providing asthma education programs for the teachers in the local schools

Answer: 1, 2, 4. All of these activities improve the level of health and quality of life for patients in this community. Controlling and managing symptoms of any disease improve the patient's quality of life. Assessing for and preventing risks and making educational programs available improve the level of health within a community. The example here was asthma, but managing chronic diseases in the community improves the overall level of health of that community. Community-based nursing care takes place in community settings such as the home or a clinic, where the focus is on the needs of the individual or family. It involves the safety needs, acute and chronic care of individuals and families, and enhancement of their capacity for self-care.

A nurse caring for a Bosnian community identifies that the children are undervaccinated and the community is unaware of resources. The nurse assesses the community and determines that there is a health clinic within a 5-mile radius. The nurse meets with the community leaders and explains the need for immunizations, the location of the clinic, and the process of accessing health care resources. Which of the following practices is the nurse providing? (Select all that apply.) 1. Educating about community resources 2. Teaching the community about illness prevention 3. Promoting autonomy in decision making 4. Improving the health care of the children in the community

Answer: 1, 2, 4. By teaching the community about available health care resources, such as the nearby clinic, and relevant illness prevention, such as immunizations, you increase the level of awareness not only about the disease but also about methods of treatment. As the community becomes more informed about the illness, prevention, and treatment methods, the health of the community increases.

In the community clinic the nurse provides care for a 40-yearold woman who takes insulin to manage her diabetes. The patient is having increased difficulty managing her disease, and the nurse wants her to consider a new insulin pump to help her control it. Which of the following increases the likelihood that the patient will accept this new insulin pump? (Select all that apply.) 1. Supporting the patient as she tries the insulin pump on a limited basis 2. Identifying why the patient is reluctant to use the insulin pump 3. Telling the patient that many other patients you know use the insulin pump successfully 4. The patient's perception that the insulin pump is more consistent with her health care goals than insulin administration

Answer: 1, 2, 4. To convince the patient to try the new technique, the patient has to change current practices. The best way to achieve this goal is to act as a change agent and apply principles to identify why the patient is resistant, hesitant, or reluctant; to support the patient in trying the new therapy; and finally to help the patient believe that the new therapy is consistent with personal health care goals.

The nurse in a community health clinic notices an increase in the number of positive tuberculosis skin tests from students in a local high school during the most recent academic year. After comparing these numbers to the previous years, a 10% increase in positive tests was found. The nurse contacts the school nurse and the director of the health department. Together they begin to expand their assessment to all students and employees of the school district. The community health nurse is acting in which nursing role(s)? (Select all that apply.) 1. Epidemiologist 2. Counselor 3. Collaborator 4. Case manager

Answer: 1, 3. Initially when the nurse noticed an increase in the number of positive tuberculosis (TB) skin tests, she was comparing current data with previous data to track positive skin test rates. Once the increase was noted, she collaborated with the school nurse and other members of the health department to assess the entire school district.

A patient is worried about her 76-year-old grandmother who is in very good health and wants to live at home. The patient's concerns are related to her grandmother's safety. The neighborhood does not have a lot of crime. Using this scenario, which of the following are the most relevant to assess for safety? 1. Crime rate, locks, lighting, neighborhood traffic 2. Lighting, locks, clutter, medications 3. Crime rate, medications, support system, clutter 4. Locks, lighting, neighborhood traffic, crime rate

Answer: 2. The crime statistics note that the grandmother's neighborhood does not have a lot of crime. Although you want to ensure that safe, working, effective locks are on the doors, you do not need to assess the crime rate. Being sure that her grandmother can navigate her home with appropriate lighting and ensuring clutter control reduce the risk for falls and help with physical safety. Finally, knowledge about medication determines if the grandmother is at risk for confusion, dizziness, or falls secondary to medication effects.

In an occupational health setting, the nurse determines that a large number of employees smoke and designs an employee assistance program for smoking cessation. This is an example of which nursing role? 1. Educator 2. Counselor 3. Collaborator 4. Case manager

Answer: 2. To convince a patient to participate in a smoking cessation program or any program that requires changing a behavior requires the nurse to act as a counselor to support the patient in changing a behavior (in this case a smoking habit). It means that the nurse does not educate the patient about the dangers of smoking; first he or she must actively counsel the patient to decide to change the behavior. Without support and counseling, the smoking cessation education may not be used effectively by the patient.

A community nurse in a diverse community is working with health care professionals to provide prenatal care for underemployed and under-insured South African women. Which overall goal of Healthy People 2020 does this represent? 1. Assess the health care needs of individuals, families, or communities 2. Develop and implement public health policies and improve access to care 3. Gather information on incident rates of certain diseases and social problems 4. Increase life expectancy and quality of life and eliminate health disparities

Answer: 4. By providing prenatal care to this group of women, the health professionals will improve the birth outcomes for the South American women's children and in turn the children's quality of life. Being underinsured represents a health disparity, and Healthy People 2020 aims to decrease this inequity.

Vulnerable populations of patients are those who are more likely to develop health problems as a result of: 1. Chronic diseases and homelessness 2. Poverty and acute illness 3. Lack of transportation, ability to perform self-care but are homeless 4. Excess health risks, limits in access to health care services, and dependency on others for care

Answer: 4. Vulnerability is a likelihood to develop health problems due to an excess of risks, limited access to health care services, and dependence on others for care.

While reviewing the side effects of a newly prescribed medication, a 72-year-old client notes that one of the side effects is a reduction in sexual drive. Which is the best response by the nurse? A. "How will this affect your present sexual activity?" B. "How active is your current sex life?" C. "How has your sex life changed as you have become older?" D. "Tell me about your sexual needs as an older adult."

Answer: A (A) offers an open-ended question most relevant to the client's statement. (B) does not offer the client the opportunity to express concerns. (C and D) are even less relevant to the client's statement.

Which nonverbal action should the nurse implement to demonstrate active listening? A. Sit facing the client. B. Cross arms and legs. C. Avoid eye contact. D. Lean back in the chair.

Answer: A Active listening is conveyed using attentive verbal and nonverbal communication techniques. To facilitate therapeutic communication and attentiveness, the nurse should sit facing the client (A), which lets the client know that the nurse is there to listen. Active listening skills include postures that are open to the client, such as keeping the arms open and relaxed, not (B), and leaning toward the client, not (D). To communicate involvement and willingness to listen to the client, eye contact should be established and maintained (C).

The nurse is obtaining a lie-sit-stand blood pressure reading on a client. Which action is most important for the nurse to implement? A. The family can provide the consent required in this situation because the older adult is in no condition to make such decisions. B. Because the client is mentally incompetent, the son has the right to waive informed consent for her. C. The court will allow the health care provider to make the decision to withhold informed consent under therapeutic privilege. D. If informed consent is withheld from a client, health care providers could be found guilty of negligence.

Answer: A Although all these measures are important, (A) is most important because it helps ensure client safety. (B) is necessary but does not have the priority of (A). (C and D) are important measures to ensure accuracy of the recording but are of less importance than providing client safety.

An older client who had abdominal surgery 3 days earlier was given a barbiturate for sleep and is now requesting to go to the bathroom. Which action should the nurse implement? A. Assist the client to walk to the bathroom and do not leave the client alone. B. Request that the UAP assist the client onto a bedpan. C. Ask if the client needs to have a bowel movement or void. D. Assess the client's bladder to determine if the client needs to urinate.

Answer: A Barbiturates cause central nervous system (CNS) depression and individuals taking these medications are at greater risk for falls. The nurse should assist the client to the bathroom (A). A bedpan (B) is not necessary as long as safety is ensured. Whether the client needs to void or have a bowel movement, (C) is irrelevant in terms of meeting this client's safety needs. There is no indication that this client cannot voice her or his needs, so assessment of the bladder is not needed (D).

In taking a client's history, the nurse asks about the stool characteristics. Which description should the nurse report to the health care provider as soon as possible? A. Daily black, sticky stool B. Daily dark brown stool C. Firm brown stool every other day D. Soft light brown stool twice a day

Answer: A Black sticky stool (melena) is a sign of gastrointestinal bleeding and should be reported to the health care provider promptly (A). (C) indicates constipation, which is a lesser priority. (B and D) are variations of normal.

Which client is most likely to be at risk for spiritual distress? A. Roman Catholic woman considering an abortion B. Jewish man considering hospice care for his wife C. Seventh-Day Adventist who needs a blood transfusion D. Muslim man who needs a total knee replacement

Answer: A In the Roman Catholic religion, any type of abortion is prohibited (A), so facing this decision may place the client at risk for spiritual distress. There is no prohibition of hospice care for members of the Jewish faith (B). Jehovah's Witnesses prohibit blood transfusions, not Seventh-Day Adventists (C). There is no conflict in the Muslim faith with regard to joint replacement (D).

A hospitalized client has had difficulty falling asleep for 2 nights and is becoming irritable and restless. Which action by the nurse is best? A. Determine the client's usual bedtime routine and include these rituals in the plan of care as safety allows. B. Instruct the UAP not to wake the client under any circumstances during the night. C. Place a "Do Not Disturb" sign on the door and change assessments from every 4 to every 8 hours. D. Encourage the client to avoid pain medication during the day, which might increase daytime napping.

Answer: A Including habitual rituals that do not interfere with the client's care or safety may allow the client to go to sleep faster and increase the quality of care (A). (B, C, and D) decrease the client's standard of care and compromise safety.

A nurse is working in an occupational health clinic when an employee walks in and states that he was struck by lightning while working in a truck bed. The client is alert but reports feeling faint. Which assessment will the nurse perform first? A. Pulse characteristics B. Open airway C. Entrance and exit wounds D. Cervical spine injury

Answer: A Lightning is a jolt of electrical current and can produce a "natural" defibrillation, so assessment of the pulse rate and regularity (A) is a priority. Because the client is talking, he has an open airway (B), so that assessment is not necessary. Assessing for (C and D) should occur after assessing for adequate circulation.

The nurse is aware that malnutrition is a common problem among clients served by a community health clinic for the homeless. Which laboratory value is the most reliable indicator of chronic protein malnutrition? A. Low serum albumin level B. Low serum transferrin level C. High hemoglobin level D. High cholesterol level

Answer: A Long-term protein deficiency is required to cause significantly lowered serum albumin levels (A). Albumin is made by the liver only when adequate amounts of amino acids (from protein breakdown) are available. Albumin has a long half-life, so acute protein loss does not significantly alter serum levels. (B) is a serum protein with a half-life of only 8 to 10 days, so it will drop with an acute protein deficiency. Neither (C or D) are clinical measures of protein malnutrition.

A client has a nursing diagnosis of Altered sleep patterns related to nocturia. Which client instruction is important for the nurse to provide? A. Decrease intake of fluids after the evening meal. B. Drink a glass of cranberry juice every day. C. Drink a glass of warm decaffeinated beverage at bedtime. D. Consult the health care provider about a sleeping pill.

Answer: A Nocturia is urination during the night. (A) is helpful to decrease the production of urine, thus decreasing the need to void at night. (B) helps prevent bladder infections. (C) may promote sleep, but the fluid will contribute to nocturia. (D) may result in urinary incontinence if the client is sedated and does not awaken to void.

A male client is laughing at a television program with his wife when the evening nurse enters the room. He says his foot is hurting and he would like a pain pill. How should the nurse respond? A. Ask him to rate his pain on a scale of 1 to 10. B. Encourage him to wait until bedtime so the pill can help him sleep. C. Attend to an acutely ill client's needs first because this client is laughing. D. Instruct him in the use of deep breathing exercises for pain control.

Answer: A Obtaining a subjective estimate of the pain experience by asking the client to rate his pain (A) helps the nurse determine which pain medication should be administered and also provides a baseline for evaluating the effectiveness of the medication. Medicating for pain should not be delayed so that it can be used as a sleep medication (B). (C) is judgmental. (D) should be used as an adjunct to pain medication, not instead of medication.

The nurse-manager of a skilled nursing (chronic care) unit is instructing UAPs on ways to prevent complications of immobility. Which intervention should be included in this instruction? A. Perform range-of-motion exercises to prevent contractures. B. Decrease the client's fluid intake to prevent diarrhea. C. Massage the client's legs to reduce embolism occurrence. D. Turn the client from side to back every shift.

Answer: A Performing range-of-motion exercises (A) is beneficial in reducing contractures around joints. (B, C, and D) are all potentially harmful practices that place the immobile client at risk of complications.

When emptying 350 mL of pale yellow urine from a client's urinal, the nurse notes that this is the first time the client has voided in 4 hours. Which action should the nurse take next? A. Record the amount on the client's fluid output record. B. Encourage the client to increase oral fluid intake. C. Notify the health care provider of the findings. D. Palpate the client's bladder for distention.

Answer: A The amount and appearance of the client's urine output is within normal limits, so the nurse should record the output (A), but no additional action is needed (B, C, and D).

Based on the nursing diagnosis of Risk for infection, which intervention is best for the nurse to implement when providing care for an older incontinent client? A. Maintain standard precautions. B. Initiate contact isolation measures. C. Insert an indwelling urinary catheter. D. Instruct client in the use of adult diapers.

Answer: A The best action to decrease the risk of infection in vulnerable clients is hand washing (A). (B) is not necessary unless the client has an infection. (C) increases the risk of infection. (D) does not reduce the risk of infection.

The nurse is using the Glasgow Coma Scale to perform a neurologic assessment. A comatose client winces and pulls away from a painful stimulus. Which action should the nurse take next? A. Document that the client responds to painful stimulus. B. Observe the client's response to verbal stimulation. C. Place the client on seizure precautions for 24 hours. D. Report decorticate posturing to the health care provider.

Answer: A The client has demonstrated a purposeful response to pain, which should be documented as such (A). Response to painful stimulus is assessed after response to verbal stimulus, not before (B). There is no indication for placing the client on seizure precautions (C). Reporting (D) is nonpurposeful movement.

While conducting an intake assessment of an adult male at a community mental health clinic, the nurse notes that his affect is flat, he responds to questions with short answers, and he reports problems with sleeping. He reports that his life partner recently died from pneumonia. Which action is most important for the nurse to implement? A. Encourage the client to see the clinic's grief counselor. B. Determine if the client has a family history of suicide attempts. C. Inquire about whether the life partner was suffering from AIDS. D. Consult with the health care provider about the client's need for antidepressant medications.

Answer: A The client is exhibiting normal grieving behaviors, so referral to a grief counselor (A) is the most important intervention for the nurse to implement. (B) is indicated, but is not a high-priority intervention. (C) is irrelevant at this time but might be important when determining the client's risk for contracting the illness. An antidepressant may be indicated (D), depending on further assessment, but grief counseling is a better action at this time because grief is an expected reaction to the loss of a loved one.

By rolling contaminated gloves inside-out, the nurse is affecting which step in the chain of infection? A. Mode of transmission B. Portal of entry C. Reservoir D. Portal of exit

Answer: A The contaminated gloves serve as the mode of transmission (A) from the portal of exit (D) of the reservoir (C) to a portal of entry (B).

In assisting an older adult client prepare to take a tub bath, which nursing action is most important? A. Check the bath water temperature. B. Shut the bathroom door. C. Ensure that the client has voided. D. Provide extra towels.

Answer: A To prevent burns or excessive chilling, the nurse must check the bath water temperature (A). (B, C, and D) promote comfort and privacy and are important interventions but are of less priority than promoting safety.

Which step(s) should the nurse take when administering ear drops to an adult client? (Select all that apply.) A. Place the client in a side-lying position. B. Pull the auricle upward and outward. C. Hold the dropper 6 cm above the ear canal. D. Place a cotton ball into the inner canal. E. Pull the auricle down and back.

Answer: A, B The correct answers (A and B) are the appropriate administration of ear drops. The dropper should be held 1 cm (½ inch) above the ear canal (C). A cotton ball should be placed in the outermost canal (D). The auricle is pulled down and back for a child younger than 3 years of age, but not an adult (E).

The nurse prepares to insert a nasogastric tube in a client with hyperemesis who is awake and alert. Which intervention(s) is(are) correct? (Select all that apply.) A. Place the client in a high Fowler's position. B. Help the patient assume a left side-lying position. C. Measure the tube from the tip of the nose to the umbilicus. D. Instruct the client to swallow after the tube has passed the pharynx. E. Assist the client in extending the neck back so the tube may enter the larynx.

Answer: A, D (A and D) are the correct steps to follow during nasogastric intubation. Only the unconscious or obtunded client should be placed in a left side-lying position (B). The tube should be measured from the tip of the nose to behind the ear and then from behind the ear to the xiphoid process (C). The neck should only be extended back prior to the tube passing the pharynx and then the client should be instructed to position the neck forward (E).

When assisting a client from the bed to a chair, which procedure is best for the nurse to follow? A. Place the chair parallel to the bed, with its back toward the head of the bed and assist the client in moving to the chair. B. With the nurse's feet spread apart and knees aligned with the client's knees, stand and pivot the client into the chair. C. Assist the client to a standing position by gently lifting upward, underneath the axillae. D. Stand beside the client, place the client's arms around the nurse's neck, and gently move the client to the chair.

Answer: B (B) describes the correct positioning of the nurse and affords the nurse a wide base of support while stabilizing the client's knees when assisting to a standing position. The chair should be placed at a 45-degree angle to the bed, with the back of the chair toward the head of the bed (A). Clients should never be lifted under the axillae (C); this could damage nerves and strain the nurse's back. The client should be instructed to use the arms of the chair and should never place his or her arms around the nurse's neck (D); this places undue stress on the nurse's neck and back and increases the risk for a fall.

After a needlestick occurs while removing the cap from a sterile needle, which action should the nurse implement? A. Complete an incident report. B. Select another sterile needle. C. Disinfect the needle with an alcohol swab. D. Notify the supervisor of the department immediately.

Answer: B After a needlestick, the needle is considered used, so the nurse should discard it and select another needle (B). Because the needle was sterile when the nurse was stuck and the needle was not in contact with any other person's body fluids, the nurse does not need to complete an incident report (A) or notify the occupational health nurse (D). Disinfecting a needle with an alcohol swab (C) is not in accordance with standards for safe practice and infection control.

The nurse is assessing several clients prior to surgery. Which factor in a client's history poses the greatest threat for complications to occur during surgery? A. Taking birth control pills for the past 2 years B. Taking anticoagulants for the past year C. Recently completing antibiotic therapy D. Having taken laxatives PRN for the last 6 months

Answer: B Anticoagulants (B) increase the risk for bleeding during surgery, which can pose a threat for the development of surgical complications. The health care provider should be informed that the client is taking these drugs. Although clients who take birth control pills (A) may be more susceptible to the development of thrombi, such problems usually occur postoperatively. A client with (C or D) is at less of a surgical risk than (B).

The health care provider has changed a client's prescription from the PO to the IV route of administration. The nurse should anticipate which change in the pharmacokinetic properties of the medication? A. The client will experience increased tolerance to the drug's effects and may need a higher dose. B. The onset of action of the drug will occur more rapidly, resulting in a more rapid effect. C. The medication will be more highly protein-bound, increasing the duration of action. D. The therapeutic index will be increased, placing the client at greater risk for toxicity.

Answer: B Because the absorptive process is eliminated when medications are administered via the IV route, the onset of action is more rapid, resulting in a more immediate effect (B). Drug tolerance (A), protein binding (C), and the drug's therapeutic index (D) are not affected by the change in route from PO to IV. In addition, an increased therapeutic index reduces the risk of drug toxicity.

When turning an immobile bedridden client without assistance, which action by the nurse best ensures client safety? A. Securely grasp the client's arm and leg. B. Put bed rails up on the side of bed opposite from the nurse. C. Correctly position and use a turn sheet. D. Lower the head of the client's bed slowly.

Answer: B Because the nurse can only stand on one side of the bed, bed rails should be up on the opposite side to ensure that the client does not fall out of bed (B). (A) can cause client injury to the skin or joint. (C and D) are useful techniques while turning a client but have less priority in terms of safety than use of the bed rails.

The nurse identifies a potential for infection in a patient with partial-thickness (second-degree) and full-thickness (third-degree) burns. What intervention has the highest priority in decreasing the client's risk of infection? A. Administration of plasma expanders B. Use of careful hand washing technique C. Application of a topical antibacterial cream D. Limiting visitors to the client with burns

Answer: B Careful hand washing technique (B) is the single most effective intervention for the prevention of contamination to all clients. (A) reverses the hypovolemia that initially accompanies burn trauma but is not related to decreasing the proliferation of infective organisms. (C and D) are recommended by various burn centers as possible ways to reduce the chance of infection. (B) is a proven technique to prevent infection.

The nurse is instructing a client with cholecystitis regarding diet choices. Which meal best meets the dietary needs of this client? A. Steak, baked beans, and a salad B. Broiled fish, green beans, and an apple C. Pork chops, macaroni and cheese, and grapes D. Avocado salad, milk, and angel food cake

Answer: B Clients with cholecystitis (inflammation of the gallbladder) should follow a low-fat diet, such as (B). (A) is a high-protein diet and (C and D) contain high-fat foods, which are contraindicated for this client.

During evacuation of a group of clients from a medical unit because of a fire, the nurse observes an ambulatory client walking alone toward the stairway at the end of the hall. Which action should the nurse take? A. Assign an unlicensed assistive personnel to transport the client via a wheelchair. B. Remind the client to walk carefully down the stairs until reaching a lower floor. C. Ask the client to help by assisting a wheelchair-bound client to a nearby elevator. D. Open the closest fire doors so that ambulatory clients can evacuate more rapidly.

Answer: B During evacuation of a unit because of fire, ambulatory clients should be evacuated via the stairway if at all possible and reminded to walk carefully (B). Ambulatory clients do not require the assistance of a wheelchair to be evacuated (A). Elevators (C) should not be used during a fire and fire doors should be kept closed (D) to help contain the fire.

The nurse teaches the use of a gait belt to a male caregiver whose wife has right-sided weakness and needs assistance with ambulation. The caregiver performs a return demonstration of the skill. Which observation indicates that the caregiver has learned how to perform this procedure correctly? A. Standing on his wife's strong side, the caregiver is ready to hold the gait belt if any evidence of weakness is observed. B. Standing on his wife's weak side, the caregiver provides security by holding the gait belt from the back. C. Standing behind his wife, the caregiver provides balance by holding both sides of the gait belt. D. Standing slightly in front and to the right of his wife, the caregiver guides her forward by gently pulling on the gait belt.

Answer: B His wife is most likely to lean toward the weak side and needs extra support on that side and from the back (B) to prevent falling. (A, C, and D) provide less security for her.

A female nurse is assigned to care for a close friend, who says, "I am worried that friends will find out about my diagnosis." The nurse tells her friend that legally she must protect a client's confidentiality. Which resource describes the nurse's legal responsibilities? A. Code of Ethics for Nurses B. State Nurse Practice Act C. Patient's Bill of Rights D. ANA Standards of Practice

Answer: B The State Nurse Practice Act (B) contains legal requirements for the protection of client confidentiality and the consequences for breaches in confidentiality. (A) outlines ethical standards for nursing care but does not include legal guidelines. (C and D) describe expectations for nursing practice but do not address legal implications.

The nurse selects the best site for insertion of an IV catheter in the client's right arm. Which documentation should the nurse use to identify placement of the IV access? A. Left brachial vein B. Right cephalic vein C. Dorsal side of the right wrist D. Right upper extremity

Answer: B The cephalic vein is large and superficial and identifies the anatomic name of the vein that is accessed, which should be included in the documentation (B). The basilic vein of the arm is used for IV access, not the brachial vein (A), which is too deep to be accessed for IV infusion. Although veins on the dorsal side of the right wrist (C) are visible, they are fragile and using them would be painful, so they are not recommended for IV access. (D) is not specific enough for documenting the location of the IV access.

The nurse is instructing a client in the proper use of a metered-dose inhaler. Which instruction should the nurse provide the client to ensure the optimal benefits from the drug? A. "Fill your lungs with air through your mouth and then compress the inhaler." B. "Compress the inhaler while slowly breathing in through your mouth." C. "Compress the inhaler while inhaling quickly through your nose." D. "Exhale completely after compressing the inhaler and then inhale."

Answer: B The medication should be inhaled through the mouth simultaneously with compression of the inhaler (B). This will facilitate the desired destination of the aerosol medication deep in the lungs for an optimal bronchodilation effect. (A, C, and D) do not allow for deep lung penetration.

The nurse is counting a client's respiratory rate. During a 30-second interval, the nurse counts six respirations and the client coughs three times. In repeating the count for a second 30-second interval, the nurse counts eight respirations. Which respiratory rate should the nurse document? A. 14 B. 16 C. 17 D. 28

Answer: B The most accurate respiratory rate is the second count obtained by the nurse, which was not interrupted by coughing. Because it was counted for 30 seconds, the rate should be doubled (B). (A, C, and D) are inaccurate recordings.

A 65-year-old client who attends an adult daycare program and is wheelchair-mobile has redness in the sacral area. Which instruction is most important for the nurse to provide? A. Take a vitamin supplement tablet once a day. B. Change positions in the chair at least every hour. C. Increase daily intake of water or other oral fluids. D. Purchase a newer model wheelchair.

Answer: B The most important teaching is to change positions frequently (B) because pressure is the most significant factor related to the development of pressure ulcers. Increased vitamin and fluid intake (A and C) may also be beneficial promote healing and reduce further risk. (D) is an intervention of last resort because this will be very expensive for the client.

A seriously ill female client tells the nurse, "I am so tired and in so much pain! Please help me to die." Which is the best response for the nurse to provide? A. Administer the prescribed maximum dose of pain medication. B. Talk with the client about her feelings related to her own death. C. Collaborate with the health care provider about initiating antidepressant therapy. D. Refer the client to the ethics committee of her local health care facility.

Answer: B The nurse should first assess the client's feelings about her death and determine the extent to which this statement expresses her true feelings (B). The client may need additional pain management, but further assessment is needed before implementing (A). (C and D) are both premature interventions and should not be implemented until further assessment is obtained.

The nurse assesses a 2-year-old who is admitted for dehydration and finds that the peripheral IV rate by gravity has slowed, even though the venous access site is healthy. What should the nurse do next? A. Apply a warm compress proximal to the site. B. Check for kinks in the tubing and raise the IV pole. C. Adjust the tape that stabilizes the needle. D. Flush with normal saline and recount the drop rate.

Answer: B The nurse should first check the tubing and height of the bag on the IV pole (B), which are common factors that may slow the rate. Gravity infusion rates are influenced by the height of the bag, tubing clamp closure or kinks, needle size or position, fluid viscosity, client blood pressure (crying in the pediatric client), and infiltration. Venospasm can slow the rate and often responds to warmth over the vessel (A), but the nurse should first adjust the IV pole height. The nurse may need to adjust the stabilizing tape on a positional needle (C) or flush the venous access with normal saline (D), but less invasive actions should be implemented first.

Ten minutes after signing an operative permit for a fractured hip, an older client states, "The aliens will be coming to get me soon!" and falls asleep. Which action should the nurse implement next? A. Make the client comfortable and allow the client to sleep. B. Assess the client's neurologic status. C. Notify the surgeon about the comment. D. Ask the client's family to co-sign the operative permit.

Answer: B This statement may indicate that the client is confused. Informed consent must be provided by a mentally competent individual, so the nurse should further assess the client's neurologic status (B) to be sure that the client understands and can legally provide consent for surgery. (A) does not provide sufficient follow-up. If the nurse determines that the client is confused, the surgeon must be notified (C) and permission obtained from the next of kin (D).

The nurse observes a UAP taking a client's blood pressure in the lower extremity. Which observation of this procedure requires the nurse's intervention? A. The cuff wraps around the girth of the leg. B. The UAP auscultates the popliteal pulse with the cuff on the lower leg. C. The client is placed in a prone position. D. The systolic reading is 20 mm Hg higher than the blood pressure in the client's arm.

Answer: B When obtaining the blood pressure in the lower extremities, the popliteal pulse is the site for auscultation when the blood pressure cuff is applied around the thigh. The nurse should intervene with the UAP who has applied the cuff on the lower leg (B). (A) ensures an accurate assessment, and (C) provides the best access to the artery. Systolic pressure in the popliteal artery is usually 10 to 40 mm Hg higher (D) than in the brachial artery.

Which action should the nurse implement when providing wound care instructions to a client who does not speak English? A. Ask an interpreter to provide wound care instructions. B. Speak directly to the client, with an interpreter translating. C. Request the accompanying family member to translate. D. Instruct a bilingual employee to read the instructions.

Answer: B Wound care instructions should be given directly to the client by the nurse with an interpreter (B) who is trained to provide accurate and objective translation in the client's primary language, so that the client has the opportunity to ask questions during the teaching process. The interpreter usually does not have any health care experience, so the nurse must provide client teaching (A). Family members should not be used to translate instructions (C) because the client or family member may alter the instructions during conversation or be uncomfortable with the topics discussed. The employee should be a trained interpreter (D) to ensure that the nurse's instructions are understood accurately by the client.

The nurse is teaching an obese client, newly diagnosed with arteriosclerosis, about reducing the risk of a heart attack or stroke. Which health promotion brochure is most important for the nurse to provide to this client? A. "Monitoring Your Blood Pressure at Home" B. "Smoking Cessation as a Lifelong Commitment" C. "Decreasing Cholesterol Levels Through Diet" D. "Stress Management for a Healthier You"

Answer: C A health promotion brochure about decreasing cholesterol (C) is most important to provide this client, because the most significant risk factor contributing to development of arteriosclerosis is excess dietary fat, particularly saturated fat and cholesterol. (A) does not address the underlying causes of arteriosclerosis. (B and D) are also important factors for reversing arteriosclerosis but are not as important as lowering cholesterol (C).

A female client with frequent urinary tract infections (UTIs) asks the nurse to explain her friend's advice about drinking a glass of juice daily to prevent future UTIs. Which response is best for the nurse provide? A. Orange juice has vitamin C that deters bacterial growth. B. Apple juice is the most useful in acidifying the urine. C. Cranberry juice stops pathogens' adherence to the bladder. D. Grapefruit juice increases absorption of most antibiotics.

Answer: C Cranberry juice (C) maintains urinary tract health by reducing the adherence of Escherichia coli bacteria to cells within the bladder. (A, B, and D) have not been shown to be as effective as cranberry juice (C) in preventing UTIs.

When taking a client's blood pressure, the nurse is unable to distinguish the point at which the first sound was heard. Which is the best action for the nurse to take? A. Deflate the cuff completely and immediately reattempt the reading. B. Reinflate the cuff completely and leave it inflated for 90 to 110 seconds before taking the second reading. C. Deflate the cuff to zero and wait 30 to 60 seconds before reattempting the reading. D. Document the exact level visualized on the sphygmomanometer where the first fluctuation was seen.

Answer: C Deflating the cuff for 30 to 60 seconds (C) allows blood flow to return to the extremity so that an accurate reading can be obtained on that extremity a second time. (A) could result in a falsely high reading. (B) reduces circulation, causes pain, and could alter the reading. (D) is not an accurate method of assessing blood pressure.

Urinary catheterization is prescribed for a postoperative female client who has been unable to void for 8 hours. The nurse inserts the catheter, but no urine is seen in the tubing. Which action will the nurse take next? A. Clamp the catheter and recheck it in 60 minutes. B. Pull the catheter back 3 inches and redirect upward. C. Leave the catheter in place and reattempt with another catheter. D. Notify the health care provider of a possible obstruction.

Answer: C It is likely that the first catheter is in the vagina, rather than the bladder. Leaving the first catheter in place will help locate the meatus when attempting the second catheterization (C). The client should have at least 240 mL of urine after 8 hours. (A) does not resolve the problem. (B) will not change the location of the catheter unless it is completely removed, in which case a new catheter must be used. There is no evidence of a urinary tract obstruction if the catheter could be easily inserted (D).

One week after being told that she has terminal cancer with a life expectancy of 3 weeks, a female client tells the nurse, "I think I will plan a big party for all my friends." How should the nurse respond? A. "You may not have enough energy before long to hold a big party." B. "Do you mean to say that you want to plan your funeral and wake?" C. "Planning a party and thinking about all your friends sounds like fun." D. "You should be thinking about spending your last days with your family."

Answer: C Setting goals that bring pleasure are appropriate and should be encouraged by the nurse (C) as long as the nurse does not perpetuate a client's denial. (A) is a negative response, implying that the client should not plan a party. (B) puts words in the client's mouth that may not be accurate. The nurse should support the client's goals rather than telling the client how to spend her time (D).

A community hospital is opening a mental health services department. Which document should the nurse use to develop the unit's nursing guidelines? A. Americans with Disability Act of 1990 B. ANA Code of Ethics with Interpretative Statements C. ANA's Scope and Standards of Nursing Practice D. Patient's Bill of Rights of 1990

Answer: C The ANA Scope of Standards of Practice for Psychiatric-Mental Health Nursing (C) serves to direct the philosophy and standards of psychiatric nursing practice. (A and D) define the client's rights. (B) provides ethical guidelines for nursing.

A nurse stops at a motor vehicle collision site to render aid until the emergency personnel arrive and applies pressure to a groin wound that is bleeding profusely. Later the client has to have the leg amputated and sues the nurse for malpractice. Which is the most likely outcome of this lawsuit? A. The Patient's Bill of Rights protects clients from malicious intents, so the nurse could lose the case. B. The lawsuit may be settled out of court, but the nurse's license is likely to be revoked. C. There will be no judgment against the nurse, whose actions were protected under the Good Samaritan Act. D. The client will win because the four elements of negligence (duty, breach, causation, and damages) can be proved.

Answer: C The Good Samaritan Act (C) protects health care professionals who practice in good faith and provide reasonable care from malpractice claims, regardless of the client outcome. Although the Patient's Bill of Rights protects clients, this nurse is protected by the Good Samaritan Act (A). The state Board of Nursing has no reason to revoke a registered nurse's license (B) unless there was evidence that actions taken in the emergency were not done in good faith or that reasonable care was not provided. All four elements of malpractice were not shown (D).

When bathing an uncircumcised boy older than 3 years, which action should the nurse take? A. Remind the child to clean his genital area. B. Defer perineal care because of the child's age. C. Retract the foreskin gently to cleanse the penis. D. Ask the parents why the child is not circumcised.

Answer: C The foreskin (prepuce) of the penis should be gently retracted to cleanse all areas that could harbor bacteria (C). The child's cognitive development may not be at the level at which (A) would be effective. Perineal care needs to be provided daily regardless of the client's age (B). (D) is not indicated and may be perceived as intrusive.

An older adult who recently began self-administration of insulin calls the nurse daily to review the steps that should be taken when giving an injection. The nurse has assessed the client's skills during two previous office visits and knows that the client is capable of giving the daily injection. Which response by the nurse is likely to be most helpful in encouraging the client to assume total responsibility for the daily injections? A. "I know you are capable of giving yourself the insulin." B. "Giving yourself the injection seems to make you nervous." C. "When I watched you give yourself the injection, you did it correctly." D. "Tell me what you want me to do to help you give yourself the injection at home."

Answer: C The nurse needs to focus on the client's positive behaviors, so focusing on the client's demonstrated ability to self-administer the injection (C) is likely to reinforce his level of competence without sounding punitive. (A) does not focus on the specific behaviors related to giving the injection and could be interpreted as punitive. (B) uses reflective dialogue to assess the client's feelings, but telling the client that he is nervous may serve as a negative reinforcement of this behavior. (D) reinforces the client's dependence on the nurse.

The nurse who is preparing to give an adolescent client a prescribed antipsychotic medication notes that parental consent has not been obtained. Which action should the nurse take? A. Review the chart for a signed consent for hospitalization. B. Get the health care provider's permission to give the medication. C. Do not give the medication and document the reason. D. Complete an incident report and notify the parents.

Answer: C The nurse should not give the medication and should document the reason (C) because the client is a minor and needs a guardian's permission to receive medications. Permission to give medications is not granted by a signed hospital consent (A) or a health care provider's permission (B), unless conditions are met to justify coerced treatment. (D) is not necessary unless the medication had previously been administered.

After the nurse tells an older client that an IV line needs to be inserted, the client becomes very apprehensive, loudly verbalizing a dislike for all health care providers and nurses. How should the nurse respond? A. Ask the client to remain quiet so the procedure can be performed safely. B. Concentrate on completing the insertion as efficiently as possible. C. Calmly reassure the client that the discomfort will be temporary. D. Tell the client a joke as a means of distraction from the procedure.

Answer: C The nurse should respond with a calm demeanor (C) to help reduce the client's apprehension. After responding calmly to the client's apprehension, the nurse may implement (A, B, or D) to ensure safe completion of the procedure.

In completing a client's preoperative routine, the nurse finds that the operative permit is not signed. The client begins to ask more questions about the surgical procedure. Which action should the nurse take next? A. Witness the client's signature to the permit. B. Answer the client's questions about the surgery. C. Inform the surgeon that the operative permit is not signed and the client has questions about the surgery. D. Reassure the client that the surgeon will answer any questions before the anesthesia is administered.

Answer: C The surgeon should be informed immediately that the permit is not signed (C). It is the surgeon's responsibility to explain the procedure to the cliesxnt and obtain the client's signature on the permit. Although the nurse can witness an operative permit (A), the procedure must first be explained by the health care provider or surgeon, including answering the client's questions (B). The client's questions should be addressed before the permit is signed (D).

A client in a long-term care facility reports to the nurse that he has not had a bowel movement in 2 days. Which intervention should the nurse implement first? A. Instruct the caregiver to offer a glass of warm prune juice at mealtimes. B. Notify the health care provider and request a prescription for a large-volume enema. C. Assess the client's medical record to determine the client's normal bowel pattern. D. Instruct the caregiver to increase the client's fluids to five 8-ounce glasses per day.

Answer: C This client may not routinely have a daily bowel movement, so the nurse should first assess this client's normal bowel habits before attempting any intervention (C). (A, B, or D) may then be implemented, if warranted.

Which instruction is most important for the nurse to include when teaching a client with limited mobility strategies to prevent venous thrombosis? A. Perform cough and deep breathing exercises hourly. B. Turn from side to side in bed at least every 2 hours. C. Dorsiflex and plantarflex the feet 10 times each hour. D. Drink approximately 4 ounces of water every hour.

Answer: C To reduce the risk of venous thrombosis, the nurse should instruct the client in measures that promote venous return, such as dorsiflexion and plantar flexion (C). (A, B, and D) are helpful to prevent other complications of immobility but are less effective in preventing venous thrombus formation than (C).

The nurse is preparing to administer 10 mL of liquid potassium chloride (Kay Ciel) through a feeding tube, followed by 10 mL of liquid acetaminophen (Tylenol). Which action should the nurse include in this procedure? A. Dilute each of the medications with sterile water prior to administration. B. Mix the medications in one syringe before opening the feeding tube. C. Administer water between the doses of the two liquid medications. D. Withdraw any fluid from the tube before instilling each medication.

Answer: C Water should be instilled into the feeding tube between administering the two medications (C) to maintain the patency of the feeding tube and ensure that the total dose of medication enters the stomach and does not remain in the tube. These liquid medications do not need to be diluted (A) when administered via a feeding tube and should be administered separately (B), with water instilled between each medication (D).

The nurse is assisting a client to the bathroom. When the client is 5 feet from the bathroom door, he states, "I feel faint." Before the nurse can get the client to a chair, the client starts to fall. Which is the priority action for the nurse to take? A. Check the client's carotid pulse. B. Encourage the client to get to the toilet. C. In a loud voice, call for help. D. Gently lower the client to the floor.

Answer: D (D) is the most prudent intervention and is the priority nursing action to prevent injury to the client and the nurse. Lowering the client to the floor should be done when the client cannot support his own weight. The client should be placed in a bed or chair only when sufficient help is available to prevent injury. (A) is important but should be done after the client is in a safe position. Because the client is not supporting himself, (B) is impractical. (C) is likely to cause chaos on the unit and might alarm the other clients.

During a routine assessment, an obese 50-year-old female client expresses concern about her sexual relationship with her husband. Which is the best response by the nurse? A. Reassure the client that many obese people have concerns about sex. B. Remind the client that sexual relationships need not be affected by obesity. C. Determine the frequency of sexual intercourse. D. Ask the client to talk about specific concerns.

Answer: D (D) provides an opportunity for the client to verbalize her concerns and provides the nurse with more assessment data. (A and B) may not be related to her current concern, assume that obesity is the problem, and are communication blocks. (C) may be appropriate after discussing the concerns she is having.

A 20-year-old female client with a noticeable body odor has refused to shower for the last 3 days. She states, "I have been told that it is harmful to bathe during my period." Which action should the nurse take first? A. Accept and document the client's wish to refrain from bathing. B. Offer to give the client a bed bath, avoiding the perineal area. C. Obtain written brochures about menstruation to give to the client. D. Teach the importance of personal hygiene during menstruation with the client.

Answer: D Because a shower is most beneficial for the client in terms of hygiene, the client should receive teaching first (D), respecting any personal beliefs such as cultural or spiritual values. After client teaching, the client may still choose (A or B). Brochures reinforce the teaching (C).

A client's blood pressure reading is 156/94 mm Hg. Which action should the nurse take first? A. Tell the client that the blood pressure is high and that the reading needs to be verified by another nurse. B. Contact the health care provider to report the reading and obtain a prescription for an antihypertensive medication. C. Replace the cuff with a larger one to ensure an ample fit for the client to increase arm comfort. D. Compare the current reading with the client's previously documented blood pressure readings.

Answer: D Comparing this reading with previous readings (D) will provide information about what is normal for this client; this action should be taken first. (A) might unnecessarily alarm the client. (B) is premature. Further assessment is needed to determine if the reading is abnormal for this client. (C) could falsely decrease the reading and is not the correct procedure for obtaining a blood pressure reading.

When the health care provider diagnoses metastatic cancer and recommends a gastrostomy for an older female client in stable condition, the son tells the nurse that his mother must not be told the reason for the surgery because she "can't handle" the cancer diagnosis. Which legal principle is the court most likely to uphold regarding this client's right to informed consent? A. The family can provide the consent required in this situation because the older adult is in no condition to make such decisions. B. Because the client is mentally incompetent, the son has the right to waive informed consent for her. C. The court will allow the health care provider to make the decision to withhold informed consent under therapeutic privilege. D. If informed consent is withheld from a client, health care providers could be found guilty of negligence.

Answer: D Health care providers may be found guilty of negligence (D), specifically assault and battery, if they carry out a treatment without the client's consent. The client's condition is stable, so (A) is not a valid rationale. Advanced age does not automatically authorize the son to make all decisions for his mother, and there is no evidence that the client is mentally incompetent (B). Although (C) may have been upheld in the past, when paternalistic medical practice was common, today's courts are unlikely to accept it.

Which nursing diagnosis has the highest priority when planning care for a client with an indwelling urinary catheter? A. Self-care deficit B. Functional incontinence C. Fluid volume deficit D. High risk for infection

Answer: D Indwelling urinary catheters are a major source of infection (D). (A and B) are both problems that may require an indwelling catheter. (C) is not affected by an indwelling catheter.

The nurse transcribes the postoperative prescriptions for a client who returns to the unit following surgery and notes that an antihypertensive medication that was prescribed preoperatively is not listed. Which action should the nurse take? A. Consult with the pharmacist about the need to continue the medication. B. Administer the antihypertensive medication as prescribed preoperatively. C. Withhold the medication until the client is fully alert and vital signs are stable. D. Contact the health care provider to renew the prescription for the medication.

Answer: D Medications prescribed preoperatively must be renewed postoperatively, so the nurse should contact the health care provider if the antihypertensive medication is not included in the postoperative prescriptions (D). The pharmacist (A) does not prescribe medications or renew prescriptions. The nurse must have a current prescriptions before administering any medications (B and C).

Which serum laboratory value should the nurse monitor carefully for a client who has a nasogastric (NG) tube to suction for the past week? A. White blood cell count B. Albumin C. Calcium D. Sodium

Answer: D Monitoring serum sodium levels (D) for hyponatremia is indicated during prolonged NG suctioning because of loss of fluids. Changes in levels of (A, B, or C) are not typically associated with prolonged NG suctioning.

The nurse is preparing an older client for discharge. Which method is best for the nurse to use when evaluating the client's ability to perform a dressing change at home? A. Determine how the client feels about changing the dressing. B. Ask the client to describe the procedure in writing. C. Seek a family member's evaluation of the client's ability to change the dressing. D. Observe the client change the dressing unassisted.

Answer: D Observing the client directly (D) will allow the nurse to determine if mastery of the skill has been obtained and provide an opportunity to affirm the skill. (A) may be therapeutic but will not provide an opportunity to evaluate the client's ability to perform the procedure. (B) may be threatening to an older client and will not determine his ability. (C) is not as effective as direct observation by the nurse.

The nurse determines that a postoperative client's respiratory rate has increased from 18 to 24 breaths/min. Based on this assessment finding, which intervention is most important for the nurse to implement? A. Encourage the client to increase ambulation in the room. B. Offer the client a high-carbohydrate snack for energy. C. Force fluids to thin the client's pulmonary secretions. D. Determine if pain is causing the client's tachypnea.

Answer: D Pain, anxiety, and increasing fluid accumulation in the lungs (D) can cause tachypnea (increased respiratory rate). Encouraging (A) when the respiratory rate is rising above normal limits puts the client at risk for further oxygen desaturation. (B) can increase the client's carbon metabolism, so an alternative source of energy, such as Pulmocare liquid supplement, should be offered instead. (C) could increase respiratory congestion in a client with a poorly functioning cardiopulmonary system, placing the client at risk of fluid overload.

During a clinic visit, the mother of a 7-year-old reports to the nurse that her child is often awake until midnight playing and is then very difficult to awaken in the morning for school. Which assessment data should the nurse obtain in response to the mother's report? A. The occurrence of any episodes of sleep apnea B. The child's blood pressure, pulse, and respirations C. Length of rapid eye movement (REM) sleep that the child is experiencing D. Description of the family's home environment

Answer: D School-age children often resist bedtime. The nurse should begin by assessing the environment of the home (D) to determine factors that may not be conducive to the establishment of bedtime rituals that promote sleep. (A) often causes daytime fatigue rather than resistance to going to sleep. (B) is unlikely to provide useful data. The nurse cannot determine (C).

When performing sterile wound care in the acute care setting, the nurse obtains a bottle of normal saline from the bedside table that is labeled "opened" and dated 48 hours prior to the current date. Which is the best action for the nurse to take? A. Use the normal saline solution once more and then discard. B. Obtain a new sterile syringe to draw up the labeled saline solution. C. Use the saline solution and then relabel the bottle with the current date. D. Discard the saline solution and obtain a new unopened bottle.

Answer: D Solutions labeled as opened within 24 hours may be used for clean procedures, but only newly opened solutions are considered sterile. This solution is not newly opened and is out of date, so it should be discarded (D). (A, B, and C) describe incorrect procedures.

The mental health nurse plans to discuss a client's depression with the health care provider in the emergency department. There are two clients sitting across from the emergency department desk. Which nursing action is best? A. Only refer to the client by gender. B. Identify the client only by age. C. Avoid using the client's name. D. Discuss the client another time.

Answer: D The best nursing action is to discuss the client another time (D). Confidentiality must be observed at all times, so the nurse should not discuss the client when the conversation can be overheard by others. Details can identify the client when referring to the client by gender (A) or age (B), and even when not using the client's name (C).

A client becomes angry while waiting for a supervised break to smoke a cigarette outside and states, "I want to go outside now and smoke. It takes forever to get anything done here!" Which intervention is best for the nurse to implement? A. Encourage the client to use a nicotine patch. B. Reassure the client that it is almost time for another break. C. Have the client leave the unit with another staff. D. Review the schedule of outdoor breaks with the client.

Answer: D The best nursing action is to review the schedule of outdoor breaks (D) and provide concrete information about the schedule. (A) is contraindicated if the client wants to continue smoking. (B) is insufficient to encourage a trusting relationship with the client. (C) is preferential for this client only and is inconsistent with unit rules.

The nurse is administering the 0900 medications to a client who was admitted during the night. Which client statement indicates that the nurse should further assess the medication order? A. "At home I take my pills at 8:00 am." B. "It costs a lot of money to buy all of these pills." C. "I get so tired of taking pills every day." D. "This is a new pill I have never taken before."

Answer: D The client's recognition of a "new" pill requires further assessment (D) to verify that the medication is correct, if it is a new prescription or a different manufacturer, or if the client needs further instruction. The time difference may not be as significant in terms of its effect, but this should be explained (A). Although comments about cost (B) should be considered when developing a discharge plan, (D) is a higher priority. The client's feelings (C) should be acknowledged, but observation of the five rights of medication administration is most essential.

The nurse is teaching a client how to perform progressive muscle relaxation techniques to relieve insomnia. A week later the client reports that he is still unable to sleep, despite following the same routine every night. Which action should the nurse take first? A. Instruct the client to add regular exercise as a daily routine. B. Determine if the client has been keeping a sleep diary. C. Encourage the client to continue the routine until sleep is achieved. D. Ask the client to describe the routine that the client is currently following.

Answer: D The nurse should first evaluate whether the client has been adhering to the original instructions (D). A verbal report of the client's routine will provide more specific information than the client's written diary (B). The nurse can then determine which changes need to be made (A). The routine practiced by the client is clearly unsuccessful, so encouragement alone is insufficient (C).

A client has a nasogastric tube connected to low intermittent suction. When administering medications through the nasogastric tube, which action should the nurse do first? A. Clamp the nasogastric tube. B. Confirm placement of the tube. C. Use a syringe to instill the medications. D. Turn off the intermittent suction device.

Answer: D The nurse should first turn off the suction (D) and then confirm placement of the tube in the stomach (B) before instilling the medications (C). To prevent immediate removal of the instilled medications and allow absorption, the tube should be clamped for a period of time (A) before reconnecting the suction.

The nurse finds a client crying behind a locked bathroom door. The client will not open the door. Which action should the nurse implement first? A. Instruct an unlicensed assistive personnel (UAP) to stay and keep talking to the client. B. Sit quietly in the client's room until the client leaves the bathroom. C. Allow the client to cry alone and leave the client in the bathroom. D. Talk to the client and attempt to find out why the client is crying.

Answer: D The nurse's first concern should be for the client's safety, so an immediate assessment of the client's situation is needed (D). (A) is incorrect; the nurse should implement the intervention. The nurse may offer to stay nearby after first assessing the situation more fully (B). Although (C) may be correct, the nurse should determine if the client's safety is compromised and offer assistance, even if it is refused.

Which intervention is most important to include in the plan of care for a client at high risk for the development of postoperative thrombus formation? A. Instruct in the use of the incentive spirometer. B. Elevate the head of the bed during all meals. C. Use aseptic technique to change the dressing. D. Encourage frequent ambulation in the hallway.

Answer: D Thrombus (clot) formation can occur in the lower extremities of immobile clients, so the nurse should plan to encourage activities to increase mobility, such as frequent ambulation (D) in the hallway. (A) helps promote alveolar expansion, reducing the risk for atelectasis. (B) reduces the risk for aspiration. (C) reduces the risk for postoperative infection.

After receiving written and verbal instructions from a clinic nurse about a newly prescribed medication, a client asks the nurse what to do if questions arise about the medication after getting home. How should the nurse respond? A. Provide the client with a list of Internet sites that answer frequently asked questions about medications. B. Advise the client to obtain a current edition of a drug reference book from a local bookstore or library. C. Reassure the client that information about the medication is included in the written instructions. D. Encourage the client to call the clinic nurse or health care provider if any questions arise.

Answer: D To ensure safe medication use, the nurse should encourage the client to call the nurse or health care provider (D) if any questions arise. (A, B, and C) may all include useful information, but these sources of information cannot evaluate the nature of the client's questions and the follow-up needed.

The nurse in a new community-based clinic is requested to complete a community assessment. Order the steps for completing this assessment. 1. Structure or locale 2. Social systems 3. Population

Answer: The correct order is (1) locale, (3) population, and (2) social systems. To begin a community assessment the structure and geographical boundaries of the community are identified. Look at the structures in the community (e.g., schools, churches, types of residences). Next obtain data about the population and the demographics of the community. Who are the residents, what is the age range, and what types of ethnicity are represented? Finally review the social systems in the community. Although the number of schools and churches were obtained when reviewing the community structure, it is important to know what, if any, community services are provided by the schools and churches, including parenting classes, support groups, and play days.

A 58-year-old patient with nerve deafness has come to his doctor's office for a routine examination. The patient wears two hearing aids. The advanced practice nurse who is conducting the assessment uses which of the following approaches while conducting the interview with this patient? (Select all that apply.) A) Maintain a neutral facial expression B) Lean forward when interacting with the patient C) Acknowledge the patient's answers through head nodding. D) Limit direct eye contact

B) Lean forward when interacting with the patient C) Acknowledge the patient's answers through head nodding

A nurse checks a patient's intravenous (IV) line in his right arm and sees inflammation where the catheter enters the skin. She uses her finger to apply light pressure (i.e., palpation) just above the IV site. The patient tells her the area is tender. The nurse checks to see if the IV line is running at the correct rate. This is an example of what type of assessment? A) Agenda setting B) Problem-focused C) Objective D) Use of a structured database format

B) Problem-focused

In the United States, there has never been a president of Asian or Hispanic culture. This is an example of: a. Social inequality b. Marginalization c. Under inclusion d. Social location

B. Not having a U.S. president representing an Asian or Hispanic culture is an example of marginalization—when a group is left out from facets of society. Social inequality is when people have unequal access to resources, services, and positions. Under inclusion is when a group has been overlooked in research and the design of interventions. Social location occurs when a person's place in society is based on his or her membership in a social group that determines access to resources.

The nurse educator is providing in-service education to the nursing staff regarding transcultural nursing care; a staff member asks the nurse educator to provide an example of the concept of acculturation. The nurse educator should make which *most appropriate* response? A. "A group of individuals identifying as a part of the Iroquois tribe among Native Americans." B. "A person who moves from China to the United States (U.S.) and learns about and adapts to the culture in the U.S." C. "A group of individuals living in the Azores that identify autonomously but are a part of the larger population of Portugal." D. "A person who has grown up in the Philippines and chooses to stay there because of the sense of belonging to his or her cultural group."

B. "A person who moves from China to the United States (U.S.) and learns about and adapts to the culture in the U.S."

A couple comes to the family planning clinic and asks about sterilization procedures. Which question by the nurse should determine whether this method of family planning would be *most appropriate*? A. "Did you ever had surgery?" B. "Do you plan to have any other children?" C. "Do either of you have diabetes mellitus?" D. "Do either of you have problems with high blood pressure?"

B. "Do you plan to have any other children?"

The nurse should make which statement to a pregnant client found to have a gynecoid pelvis? A. "Your type of pelvis has a narrow pubic arch." B. "Your type of pelvis is the most favorable for labor and birth." C. "Your type of pelvis is a wide pelvis, but it has a short diameter." D. "You will need a cesarean section because this type of pelvis is not favorable for a vaginal delivery."

B. "Your type of pelvis is the most favorable for labor and birth."

The nurse is making initial rounds at the beginning of the shift and notes that the parenteral nutrition (PN) bag of an assigned client is empty. Which solution should the nurse hang until another PN solution is mixed and delivered to the nursing unit? A. 5% dextrose in water B. 10% dextrose in water C. 5% dextrose in Ringer's lactate D. 5% dextrose in 0.9% sodium chloride

B. 10% dextrose in water

A client has been admitted to the hospital for urinary tract infection and dehydration. The nurse determines that the client has received adequate volume replacement if the blood urea nitrogen (BUN) level drops to which value? A. 3 mg/dL (1.05 mmol/L) B. 15 mg/dL (5.25 mmol/L) C. 29 mg/dL (10.15 mmol/L) D. 35 mg/dL (12.25 mmol/L)

B. 15 mg/dL (5.25 mmol/L)

A client is being admitted to the hospital for treatment of acute cellulitis of the lower leg and asks the admitting nurse to explain what cellulitis means. The nurse bases the response on the understanding that cellulitis has which characteristic? A. An inflammation of the epidermis only B. A skin infection of the dermis and underlying hypodermis C. An acute superficial infection of the dermis and lymphatics D. An epidermal and lymphatic infection caused by Staphylococcus

B. A skin infection of the dermis and underlying hypodermis

When communicating with a client who speaks a different language, which *best* practice should the nurse implement? A. Speak loudly and slowly B. Arrange for an interpreter to translate C. Speak to the client and family together D. Stand close to the client and speak loudly

B. Arrange for an interpreter to translate

A client is brought to the emergency department with partial thickness burns to his face, neck, arms, and chest after trying to put out a car fire. The nurse should implement which nursing actions for this client? *Select all that apply.* A. Restrict fluids B. Assess for airway patency C. Administer oxygen as prescribed D. Place a cooling blanket on the client E. Elevate extremities if no fractures are present F. Prepare to give oral pain medication as prescribed

B. Assess for airway patency C. Administer oxygen as prescribed E. Elevate extremities if no fractures are present

The nurse is caring for a postoperative client who is receiving demand-dose hydromorphone via a patient-controlled analgesia (PCA) pump for pain control. The nurse enters the client's room and finds the client drowsy and records the following vital signs: T 97.2F orally, pulse 52 BPM, BP 101/58 mmHg, respiratory rate 11 BPM, and SpO2 of 93% on 3 liters of oxygen via nasal cannula. Which action should the nurse take *next?* A. Document the findings B. Attempt to arouse the client C. Contact the health care provider (HCP) immediately D. Check the medication administration history on the PCA pump

B. Attempt to arouse the client

A client with metastatic breast cancer is receiving tamoxifen. The nurse specifically monitors which laboratory value while the client is taking this medication? A. Glucose level B. Calcium level C. Potassium level D. Prothrombin time

B. Calcium level Rationale: Tamoxifen may increase calcium, cholesterol, and triglyceride levels. Before the initiation of therapy, a complete blood count, platelet count, and serum calcium level should be assessed. These blood levels, along with cholesterol and triglyceride levels, should be monitored periodically during therapy. The nurse should assess for hypercalcemia while the client is taking this medication. Signs of hypercalcemia include increased urine volume, excessive thirst, nausea, vomiting, constipation, hypotonicity of muscles, and deep bone and flank pain.

The nurse arrives at work and is told to report (float) to the intensive care unit (ICU) for the day because the ICU is understaffed and needs additional nurses to care for the clients. The nurse has never worked in the ICU. The nurse should take which *best* action? A. Refuse to float to the ICU based on lack of unit orientation B. Clarify with the team leader to make a safe ICU client assignment C. Ask the nursing supervisor to review the hospital policy on floating D. Submit a written protest to nursing administration, and then call the hospital lawyer

B. Clarify with the team leader to make a safe ICU client assignment

The nurse is preparing to initiate an intravenous (IV) line containing a high dose of potassium chloride and plans to use an IV infusion pump. The nurse brings the pump to the bedside, prepares to plug the pump cord into the wall, and notes that no receptacle is available in the wall socket. The nurse should take which action? A. Initiate the intravenous line without the use of a pump B. Contact the electrical maintenance department for assistance C. Plug in the pump cord in the available plug above the room sink D. Use an extension cord from the nurses' lounge for the pump plug

B. Contact the electrical maintenance department for assistance

Intravenous (IV) fluids have been infusing at 100 mL/hour via a central line catheter in the right internal jugular for approximately 24 hours to increase urine output and maintain the client's blood pressure. Upon entering the client's room, the nurse notes that the client is breathing rapidly and coughing. For which additional signs of a complication should the nurse assess based on the previously known data? A. Excessive bleeding B. Crackles in the lungs C. Incompatibility of the infusion D. Chest pain radiating to the left arm

B. Crackles in the lungs

Which nursing action is *essential* prior to initiating a new prescription for 500 mL of fat emulsion (lipids) to infuse at 50 mL/hour? A. Ensure that the client does not have diabetes B. Determine whether the client has an allergy to eggs C. Add regular insulin to the fat emulsion, using aseptic technique D. Contact the health care provider (HCP) to have a central line inserted for fat emulsion infusion

B. Determine whether the client has an allergy to eggs

The nurse has made an error in a narrative documentation of an assessment finding on a client and obtains the client's record to correct the error. The nurse should take which action to correct the error? *Select all that apply.* A. Document a late entry into the client's record B. Draw 1 line through the error, initialing and dating it C. Try to erase the error for space to write in the correct data D. Use without to delete the error to write in the correct data E. Write a concise statement to explain why the correction was needed F. Document the correct information and end with the nurse's signature and title

B. Draw 1 line through the error, initialing and dating it F. Document the correct information and end with the nurse's signature and title

The nurse is reviewing the history of a client with bladder cancer. The nurse expects to note documentation of which *most* common sign or symptom of this type of cancer? A, Dysuria B. Hematuria C. Urgency on urination D. Frequency of urination

B. Hematuria

A client with atrial fibrillation who is receiving maintenance therapy of warfarin sodium has a prothrombin time (PT) of 35 seconds and an international normalized ratio (INR) of 3.5. On the basis of these laboratory values, the nurse anticipates which prescription? A. Adding a dose of heparin sodium B. Holding the next dose of warfarin C. Increasing the next dose of warfarin D. Administering the next dose of warfarin

B. Holding the next dose of warfarin

The nurse is providing medication instructions to a client with breast cancer who is receiving cyclophosphamide. The nurse should tell the client to take which action? A. Take the medication with food B. Increase fluid intake to 2000 to 3000 mL daily C. Decrease sodium intake while taking the medication D. Increase potassium intake while taking the medication

B. Increase fluid intake to 2000 to 3000 mL daily Rationale: Hemorrhagic cystitis is an adverse effect that can occur with the use of cyclophosphamide. The client needs to be instructed to drink copious amounts of fluid during the administration of this medication. Clients also should monitor urine output for hematuria. The medication should be taken on an empty stomach, unless gastrointestinal upset occurs. Hyperkalemia can result from the use of the medication; therefore, the client would not be told to increase potassium intake. The client would not be instructed to alter sodium intake.

The community health nurse is providing a teaching session about anthrax to members of the community and asks the participants about the methods of transmission. Which answers by the participants would indicate that teaching was effective? *Select all that apply.* A. Bites from ticks or deer flies B. Inhalation of bacterial spores C. Through a cut or abrasion in the skin D. Direct contact with an infected individual E. Sexual contact with an infected individual F. Ingestion of contaminated undercooked meat

B. Inhalation of bacterial spores C. Through a cut or abrasion in the skin F. Ingestion of contaminated undercooked meat

A client returns to the clinic for follow-up treatment following a skin biopsy of a suspicious lesion performed 1 week ago The biopsy report indicates that the lesion is a melanoma. The nurse understands that melanoma has which characteristics? A. Lesion is painful to touch B. Lesion is highly metastatic C. Lesion is a nevus that has changes in color D. Skin under the lesion is reddened and warm to touch E. Lesion occurs in body area exposed to outdoor sunlight

B. Lesion is highly metastatic C. Lesion is a nevus that has changes in color

The nurse is making initial rounds on the nursing unit to assess the condition of assigned clients. Which assessment findings are consistent with infiltration? *Select all that apply.* A. Pain and erythema B. Pallor and coolness C. Numbness nad pain D. Edema and blanched skin E. Formation of a red streak and purulent drainage

B. Pallor and coolness C. Numbness nad pain D. Edema and blanched skin

A client is diagnosed as having a bowel tumor. The nurse should monitor the client for which complications of this type of tumor? *Select all that apply.* A. Flatulence B. Peritonitis C. Hemorrhage D. Fistula formation E. Bowel perforation F. Lactose intolerance

B. Peritonitis C. Hemorrhage D. Fistula formation E. Bowel perforation

The clinic nurse notes that the health care provider has documented a diagnosis of herpes zoster (shingles) in the client's chart. Based on an understanding of the cause of this disorder, the nurse determines that this definitive diagnosis was made by which diagnostic test? A. Positive patch test B. Positive culture results C. Abnormal biopsy results D. Wood's light examination indicative of infection

B. Positive culture results

The nurse is assessing a client's peripheral intravenous (IV) site after completion of a vancomycin infusion and notes that the area is reddened, warm, painful, and slightly edematous proximal to the insertion point of the IV catheter. At this time, which action by the nurse is *best?* A. Check for the presence of blood return B. Remove the IV site and restart at another site C. Document the findings and continue to monitor the IV site D. Call the health care provider (HCP) and request that the vancomycin be given orally

B. Remove the IV site and restart at another site

The nurse obtains a prescription from a health care provider to restrain a client and instructs an unlicensed assistive personnel (UAP) to apply the safety device to the client. Which observation of unsafe application of the safety device would indicate that *further instruction is required* by the UAP? A. Placing a safety knot in the safety device straps B. Safely securing the safety device straps to the side rails C. Applying safety device straps that do not tighten when force is applied against them D. Securing so that 2 fingers can slide easily between the safety device and the client's skin

B. Safely securing the safety device straps to the side rails

Nursing staff members are sitting in the lounge taking their morning break. An unlicensed assistive personnel (UAP) tells the group that she thinks that the unit secretary has acquired immunodeficiency syndrome (AIDS) and proceeds to tell the nursing staff that the secretary probably contracted the disease from her husband, who is supposedly a drug addict. The registered nurse should inform the UAP that making this accusation has violated which legal tort? A. Libel B. Slander C. Assault D. Negligence

B. Slander

The nurse is explaining the appropriate methods for measuring an accurate temperature to an unlicensed assistive personnel (UAP). Which method, if noted by the UAP as being an appropriate method, indicates the *need for further teaching?* A. Taking a rectal temperature for a client who has undergone nasal surgery B. Taking an oral temperature for a client with a cough and nasal congestion C. Taking an axillary temperature for a client who has just consumed hot coffee D. Taking a temporal temperature on the neck behind the ear for a client who is diaphoretic

B. Taking an oral temperature for a client with a cough and nasal congestion

The nurse is instructing a client to perform a testicular self-examination (TSE). The nurse should provide the client with which information about the procedure? A. To examine the testicles while lying down B. That the best time for the examination is after a shower C. To gently feel the testicle with 1 finger to feel for a growth D. Tat TSEs should be done at least every 6 months

B. That the best time for the examination is after a shower

A client who has been receiving radiation therapy for bladder cancer tells the nurse that it feels as if she is voiding through the vagina. The nurse interprets that the client may be experiencing which condition? A. Rupture of the bladder B. The development of a vesicovaginal fistula C. Extreme stress caused by the diagnosis of cancer D. Altered perineal sensation as a side effect of ration therapy

B. The development of a vesicovaginal fistula

Isotretinoin is prescribed for a client with severe acne. Before the administration of this medication, the nurse anticipates that which Laboratory test will be prescribed? A. Potassium level B. Triglyceride level C. Hemoglobin A1C D. Total cholesterol level

B. Triglyceride level Rationale: isotretinoin can Elevate triglyceride levels period blood triglyceride levels should be measured before treatment and periodically thereafter until the effect on triglycerides has been evaluated. There is no indication that isotretinoin effects potassium, hemoglobin A1c, or total cholesterol levels.

The health education nurse provides instructions to a group of clients regarding measures that will assist in preventing skin cancer. Which instructions should the nurse provide? *Select all that apply.* A. Sunscreen should be applied every eight hours B. Use sunscreen when participating in outdoor activities C. Wear a hat, opaque clothing, and sunglasses when in the sun D. Avoid sun exposure in the late afternoon and early evening hours E. Examine your body monthly for any lesions that may be suspicious

B. Use sunscreen when participating in outdoor activities C. Wear a hat, opaque clothing, and sunglasses when in the sun E. Examine your body monthly for any lesions that may be suspicious Rationale: The client should be instructed to avoid sun exposure between the hours of brightest sunlight: 10 a.m. and 4 p.m. Sunscreen, a hat, opaque clothing, and sunglasses should be warm for outdoor activities. The client should be instructed to examine the body monthly for appearances of any cancerous or pre-cancerous lesions. Sunscreen should be reapplied every 2 to 3 hours and after swimming or sweating; otherwise, the duration of protection is reduced.

When you care for a patient who does not speak English, it is necessary to call on a professional interpreter. Which of the following are proper principles for working with interpreters? (Select all that apply.) Expect the interpreter to interpret your statements word-for-word so there is no misunderstanding by the patient. If you feel an interpretation is not correct, stop and address the situation directly with the interpreter. Pace a conversation so there is time for the patient's response to be interpreted. Direct your questions to the interpreter. Ask the patient for feedback and clarification at regular intervals.

BCE You should not expect the interpreter to interpret your statements word for word. Although the interpreter must ensure that everything that was said is interpreted, he or she may need to use more or fewer words to convey the meaning of your conversation with a patient. Direct your questions to the patient. Look at the patient instead of at the interpreter

You are caring for a hospitalized patient who is Muslim and has diabetes. Which of the following items do you need to remove from the meal tray when it is delivered to the patient?

Bacon and eggs

A nurse has worked in a home health agency for a number of years. She goes to visit a patient who has diabetes and who lives in a public housing facility. This is the first time the nurse has cared for the patient. The patient has four other family members who live with her in the one-bedroom apartment. Which of the following, based on Campinha-Bacote's (2002) model of cultural competency, is an example of cultural awareness? a. The nurse begins a discussion with the patient by asking, "Tell me about your family members who live with you?" b. The nurse asks, "What do you believe is needed to make you feel better?" c. The nurse silently reflects about how her biases regarding poverty can influence how she assesses the patient. d. The nurse uses a therapeutic and caring approach to how she interacts with the patient.

C Cultural awareness involves becoming more self-aware of your biases and attitudes about human behavior and considering these factors when you interact with patients

A new nurse is caring for a hospitalized obese patient who is homeless. This is the first time the patient has been admitted to the hospital, and the patient is scheduled for surgery. Which of the following is a universal skill that will help the nurse work effectively with this patient? a. The nurse shifts her focus to understanding the patient by asking her, "Describe for me the course of your illness." b. The nurse tells the patient, "Your choices of foods and unwillingness to exercise are adding to your health problems." c. The nurse asks the patient, "Tell me about the main problems you have had with your health from not having a home." d. The nurse explains, "Because you have obesity, it is important to know the effects it has on wound healing because of reduced tissue perfusion."

C This response enables the nurse to elicit the patient's explanation of her health problems and their causes. The nurse saying "Describe for me the course of your illness"uses a biomedical explanatory model instead of the patient's explanatory model. The nurse saying "Your choices of foods and unwillingness to exercise are adding to your health problems" shows the nurse's disrespect and unwillingness to understand the patient's perceptions and health beliefs.

What type of interview techniques does the nurse use when asking these questions, "Do you have pain or cramping?" "Does the pain get worse when you walk?" (Select all that apply.) A) Active listening B) Open-ended questioning C) Closed-ended questioning D) Problem-oriented questioning

C) Closed-ended questioning Problem-oriented questioning

The nurse asks a patient, "Describe for me your typical diet over a 24-hour day. What foods do you prefer? Have you noticed a change in your weight recently?" This series of questions would likely occur during which phase of a patient-centered interview? A) Setting the stage B) Gathering information about the patient's chief concerns C) Collecting the assessment D) Termination

C) Collecting the assessment

A patient tells the nurse during a visit to the clinic that he has been sick to his stomach for 3 days and he vomited twice yesterday. Which of the following responses by the nurse is an example of probing? A) So you've had an upset stomach and began vomiting—correct? B) Have you taken anything for your stomach? C) Is anything else bothering you? D) Have you taken any medication for your vomiting?

C) Is anything else bothering you?

During the review of systems in a nursing history, a nurse learns that the patient has been coughing mucus. Which of the following nursing assessments would be best for the nurse to use to confirm a lung problem? (Select all that apply.) A) Family report B) Chest x-ray film C) Physical examination with auscultation of the lungs D) Medical record summary of x-ray film findings

C) Physical examination with auscultation of the lungs D) Medical record summary of x-ray film findings

The nurse makes the following statement during a change of shift report to another nurse. "I assessed Mr. Diaz, my 61-year-old patient from Chile. He fell at home and hurt his back 3 days ago. He has some difficulty turning in bed, and he says that he has pain that radiates down his leg. He rates his pain at a 6, but I don't think it's that severe. You know that back patients often have chronic pain. He seems fine when talking with his family. Have you cared for him before?" What does the nurse's conclusion suggest? A) The nurse is making an accurate clinical inference. B) The nurse has gathered cues to identify a potential problem area. C) The nurse has allowed stereotyping to influence her assessment. D) The nurse wants to validate her information with the other nurse.

C) The nurse has allowed stereotyping to influence her assessment.

During an encounter with an elderly patient, the nurse recognizes that a thorough cultural assessment is necessary because the patient has recently come to the United States from Russia and has never been hospitalized before. The nurse wants to discuss cultural similarities between herself and the patient. Which step of the LEARN mnemonic is this? a. Listen b. Explain c. Acknowledge d. Recommend treatment e. Negotiate agreement

C. The nurse's desire to discuss cultural similarities is an example of Acknowledging

The nurse is assessing a client who has a new ureterostomy. Which statement by the client indicates the *need for more education* about urinary stoma care? A. "I change my pouch every week." B. "I change the appliance in the morning." C. "I empty the urinary collection bag when it is two-thirds full." D. "When I'm in the shower I direct the flow of water away from my stoma."

C. "I empty the urinary collection bag when it is two-thirds full."

The nursing instructor asks the student to describe fetal circulation, specifically the ductus venosus. Which statement by the student indicates an understanding of the ductus venosus? A. "It connects the pulmonary artery to the aorta." B. "It is an opening between the right and left atria." C. "It connects the umbilical vein to the inferior vena cava." D. "It connects the umbilical artery to the inferior vena cava."

C. "It connects the umbilical vein to the inferior vena cava."

The nurse is conducting a prenatal class on the female reproductive system. When a client in the class asks why the fertilized ovum stays in the fallopian tube for 3 days, what is the nurse's best response? A. "It promotes the fertilized ovum's chances of survival." B. "It promotes the fertilized ovum's exposure to estrogen and progesterone." C. "It promotes the fertilized ovum's normal implantation in the top portion of the uterus." D. "It promotes the fertilized ovum's exposure to luteinizing hormone and follicle-stimulating hormone."

C. "It promotes the fertilized ovum's normal implantation in the top portion of the uterus."

A client calls the emergency department and tells the nurse that he came directly into contact with poison ivy shrubs. The client tells the nurse that he cannot see anything on the skin and asks the nurse what to do. The nurse should make which response? A. "Come to the emergency department." B. "Apply calamine lotion immediately to the exposed skin areas." C. "Take a shower immediately, lathering and rinsing several times." D. "It is not necessary to do anything if you cannot see anything on your skin."

C. "Take a shower immediately, lathering and rinsing several times."

A clinic nurse prepares a teaching plan for a client receiving an antineoplastic medication. When implementing the plan, the nurse should make which statement to the client? A. "You can take aspirin as needed for headache." B. "You can drink beverages containing alcohol in moderate amounts each evening." C. "You need to consult with the health care provider (HCP) before receiving immunizations." D. "It is fine to receive a flu vaccine at the local health fair without HCP approval because the flu is so contagious."

C. "You need to consult with the health care provider (HCP) before receiving immunizations." Rationale: Because antineoplastic medications lower the resistance of the body, clients must be informed not to receive immunizations without an HCP's approval. Clients also need to avoid contact with individuals who have recently received a live virus vaccine. Clients need to avoid aspirin and aspirin-containing products to minimize the risk of bleeding, and they need to avoid alcohol to minimize the risk of toxicity and side/adverse effects.

An adult client was burned in an explosion. The burn initiall affected the client's entire face (anterior half of the head) and the upper half of the anterior torso, and there were cirumferential clothes to the lower half of both arms. The client's clothes caught on fire, and the client ran, causing subsequent burn injuries to the posterior surface of the head and upper half of the posterior torso. Using the rule of nines, what would be the extent of the burn injury? A. 18% B. 24% C. 36% d. 48%

C. 36%

The nurse is monitoring the laboratory results of a client receiving an antineoplastic medication by the intravenous route. The nurse plans to initiate bleeding precautions if which laboratory result is noted? A. A clotting time of 10 minutes B. An ammonia level of 20 mcg/dL (6 mcmol/L) C. A platelet count of 50,000 cells/mm^3 (50 x 10^9/L) D. A white blood cell count of 5000 cells/mm^3 (5.0 x 10^9/L)

C. A platelet count of 50,000 cells/mm^3 (50 x 10^9/L) Rationale: Bleeding precautions need to be initiated when the platelet count decreases. The normal platelet count is 150,000 to 450,000 cells/mmc) When the platelet count decreases, the client is at risk for bleeding. The normal white blood cell count is 4500 to 11,000 cells/mmc) When the white blood cell count drops, neutropenic precautions need to be implemented. The normal clotting time is 8 to 15 minutes. The normal ammonia value is 10 to 80 mcg/dL.

The nurse is performing a neurological assessment on a client and elicits a positive Romberg's sign. The nurse makes this determination based on which observation? A. An involuntary rhythmic, rapid, twitching of the eyeballs B. A dorsiflexion of the ankle and great toe with fanning of the other toes C. A significant sway when the client stands erect with feet together, arms at the side, and the eyes closed D. A lack of normal sense of position when the client is unable to return extended fingers to a point of reference

C. A significant sway when the client stands erect with feet together, arms at the side, and the eyes closed

The nurse is caring for a client with heart failure. On assessment, the nurse notes that the client is dyspneic, and crackles are audible on auscultation. What additional manifestations would the nurse expect to note in this client if excess fluid volume is present? A. Weight loss and dry skin B. Flat neck and hand veins and decreased urinary output C. An increase in blood pressure and increased respirations D. Weakness and decreased central venous pressure (CVP)

C. An increase in blood pressure and increased respirations

The nurse employed in a hospital is waiting to receive a report from the laboratory via the facsimile (fax) machine. The fax machine activates and the nurse expects the report, but instead receives a sexually oriented photograph. Which is the *most appropriate initial* nursing action? A. Call the police B. Cut up the photograph and throw it away C. Call the nursing supervisor and report the incident D. Call the laboratory and ask for the name of the individual who sent the photograph

C. Call the nursing supervisor and report the incident

A preoperative client expresses anxiety to the nurse about upcoming surgery. Which response by the nurse is *most likely* to stimulate further discussion between the client and the nurse? A. If it's any help, everyone is nervous before surgery B. I will be happy to explain the entire surgical procedure to you C. Can you share with me what you've been told about your surgery D. Let me tell you about the care you'll receive after surgery and the amount of pain you can anticipate

C. Can you share with me what you've been told about your surgery

A gastrectomy is performed on a client with gastric cancer. In the immediate postoperative period, the nurse notes bloody drainage from the nasogastric tube. The nurse should take which *most appropriate* action? A. Measure abdominal girth B. Irrigate the NG tube C. Continue to monitor the drainage D. Notify the health care provider (HCP)

C. Continue to monitor the drainage

A client who is recovering from surgery has been advanced from a clear liquid diet to a full liquid diet. The client is looking forward to the diet change because he has been "bored" with the clear liquid diet. The nurse should offer which full liquid item to the client? A. Tea B. Gelatin C. Custard D. Ice pop

C. Custard

Which is the *best* nursing intervention regarding complementary and alternative medicine? A. Advising the client about "good" versus "bad" therapies B. Discouraging the client from using any alternative therapies C. Educating the client about therapies that he or she is using or is interested in using D. Identifying herbal remedies that the client should request form the health care provider

C. Educating the client about therapies that he or she is using or is interested in using

The nurse is preparing to hang the first bag of parenteral nutrition (PN) solution via the central line of an assigned client. The nurse should obtain which *most essential* piece of equipment before hanging the solution? A. Urine test strips B. Blood glucose meter C. Electronic infusion pump D. Noninvasive blood pressure monitor

C. Electronic infusion pump

An antihypertensive medication has been prescribed for a client with hypertension. The client tells the clinic nurse that he would like to take an herbal substance to help lower his blood pressure. The nurse should take which action? A. Advise the client to read the labels of herbal therapies closely B. Tel the client that herbal substances are not safe and should never be used C. Encourage the client to discuss the use of an herbal substance with the health care provider (HCP) D. Tell the client that if he takes the herbal substance hw ill need to have his blood pressure checked frequently

C. Encourage the client to discuss the use of an herbal substance with the health care provider (HCP)

The nurse is monitoring the status of a client's fat emulsion (lipid) infusion and notes that the infusion is 1 hour behind. Which action should the nurse take? A. Adjust the infusion rate to catch up over the next hour B. Increase the infusion rate to catch up over the next 2 hours C. Ensure that the fat emulsion rate is infusing at the prescribed rate D. Adjust the infusion rate to run wide open until the solution is back on time

C. Ensure that the fat emulsion rate is infusing at the prescribed rate

An Asian American client is experiencing a fever. The nurse plans care so that the client can self-treat the disorder using which method? A. Prayer B. Magnetic therapy C. Foods considered to be yin D. Foods considered to be yang

C. Foods considered to be yin

The nurse is caring for a client with heart failure who is receiving high doses of a diuretic. On assessment, the nurse notes that the client has flat neck veins, generalized muscle weakness, and diminished deep tendon reflexes. the nurse suspects Hyponatremia. What additional signs would then nurse expect to note in a client with Hyponatremia? A. Muscle twitches B. Decreased urinary output C. Hyperactive bowel sounds D. Increased specific gravity of the urine

C. Hyperactive bowel sounds

A client receiving parenteral nutrition (PN) complains of a headache. The nurse notes that the client has an increased BP, bounding pulse, jugular vein distention, and crackles bilaterally. The nurse determines that the client is experiencing which complication of PN therapy? A. Sepsis B. Air embolism C. Hypervolemia D. Hyperglycemia

C. Hypervolemia

The nurse has conducted preoperative teaching for a client scheduled for surgery in 1 week. The client has a history of arthritis and has been taking acetylsalicylic acid. The nurse determines that the client *needs additional teaching* if the client makes which statement? A. Aspirin can cause bleeding after surgery B.Aspirin can cause my ability to clot blood to be abnormal C. I need to continue to take aspirin until the day of surgery D. I need to check with my health care provider about the need to stop the aspirin before the scheduled surgery

C. I need to continue to take aspirin until the day of surgery

The nurse is caring for a client following an autograft and grafting to a burn wound on the right knee. What would the nurse anticipate to be prescribed for the client? A. Out-of-bed activities B. Bathroom privileges C. Immobilization of the affected leg D. Placing the affected leg in a dependent position

C. Immobilization of the affected leg

The nurse is analyzing the laboratory results of a client with leukemia who has received a regimen of chemotherapy. Which laboratory value would the nurse specifically note as a result of the massive cell destruction that occurred from the chemotherapy? A. Anemia B. Decreased platelets C. Increased uric acid level D. Decreased leukocyte count

C. Increased uric acid level Rationale: Hyperuricemia is especially common following treatment for leukemias and lymphomas because chemotherapy results in massive cell kill. Although options 1, 2, and 4 also may be noted, an increased uric acid level is related specifically to cell destruction.

A client is undergoing fluid replacement after being burned on 20% of her body 12 hours ago. The nursing assessment reveals a blood pressure of 90/50 mm Hg, a pulse rate of 110 beats/minute, and a urine output of 20 mL over the past hour. The nurse reports the findings to the health care provider and anticipates which prescription? A. Transfusing 1 unit packed red blood cells B. Administering a diuretic to increase urine output C. Increasing the amount of intravenous (IV) lactated ringer's solution administered per hour D. Changing the IV lactated ringer's solution to one that contains dextrose in water

C. Increasing the amount of intravenous (IV) lactated ringer's solution administered per hour

An adult female has a hemoglobin level of 10.8 g/dL (108 mmol/L). The nurse interprets that this result is *most likely* caused by which condition noted in the client's history? A. Dehydration B. Heart failure C. Iron deficiency anemia D. Chronic obstructive pulmonary disease

C. Iron deficiency anemia

The nurse is planning to teach client with malabsorption syndrome about the necessity of following a low-fat diet. The nurse develops a list of high-fat foods to avoid and should include which food items on the list? *Select all that apply.* A. Oranges B. Broccoli C. Margarine D. Cream cheese E. Luncheon meats F. Broiled haddock

C. Margarine D. Cream cheese E. Luncheon meats

Chemotherapy dosage is frequently based on total body surface area (BSA), so it is important for the nurse to perform which assessment before administering chemotherapy? A. Measure the client's abdominal girth B. Calculate the client's body mass index C. Measure the client's current weight and height D. Ask the client about his or her weight and height

C. Measure the client's current weight and height Rationale: To ensure that the client receives optimal doses of chemotherapy, dosing is usually based on the total body surface area (BSA), which requires a current accurate height and weight for BSA calculation (before each medication administration). Asking the client about his or her height and weight may lead to inaccuracies in determining a true BSA and dosage. Calculating body mass index and measuring abdominal girth will not provide the data needed.

A Spanish-speaking client arrives at the ridge desk in the emergency department and states to the nurse, "No speak English, need interpreter." Which is the *best* action for the nurse to take? A. Have one of the client's family members interpret B. Have the Spanish-speaking triage receptionist interpret C. Page an interpreter from the hospital's interpreter services D. Obtain a Spanish-English dictionary and attempt to triage the client

C. Page an interpreter from the hospital's interpreter services

The nurse is caring for a restless client who is beginning nutritional therapy with parenteral nutrition (PN). The nurse should plan to ensure that which action is taken to prevent the client from sustaining injury? A. Calculate daily intake and output B. Monitor the temperature once daily C. Secure all connections in the PN system D. Monitor blood glucose levels every 12 hours

C. Secure all connections in the PN system

The nurse is instructing a client with hypertension on the importance of choosing food slow in sodium. The nurse should teach the client to limit intake of which food? A. Apples B. Bananas C. Smoked sausage D. Steamed vegetables

C. Smoked sausage

Which meal tray should the nurse deliver to a client of Orthodox Judaism faith who follows a kosher diet? A. Pork roast, rice, vegetables, mixed fruit, milk B. Crab salad on a croissant, vegetables with dip, potato salad, milk C. Sweet and sour chicken with rice and vegetables, mixed fruit, juice D. Noodles and cream sauce with shrimp and vegetables, salad, mixed fruit, iced tea

C. Sweet and sour chicken with rice and vegetables, mixed fruit, juice

The nurse reviews the electrolyte results fan assigned client and notes that the potassium level is 5.7 mEq/L (5.7 mol/L). Which patterns would the nurse watch for on the cardiac monitor as a result of the laboratory value? *Select all that apply.) A. ST depression B. Prominent U wave C. Tall peaked T waves D. Prolonged ST segment E. Widened QRS complexes

C. Tall peaked T waves E. Widened QRS complexes

The nurse should plan to implement which intervention in the care of a client experiencing neutropenia as a result of chemotherapy? A. Restrict all visitors B. Restrict fluid intake C. Teach the client and family about the need for hand hygiene D. Insert an indwelling urinary catheter to prevent skin breakdown

C. Teach the client and family about the need for hand hygiene

The nurse is assessing a client for meningeal irritation and elicits a positive Brudzinski's sign. Which finding did the nurse observe? A. The client rigidly extends the arms with pronated forearms and plantar flexion of the feet B. The client flexes a leg at the hip and knee and reports pain in the vertebral column when the leg is extended C. The client passively flexes the hip and knee in response to neck flexion and reports pain in the vertebral column D. The client's upper arms are flexed and held tightly to the sides of the body and the legs are extended and internally rotated

C. The client passively flexes the hip and knee in response to neck flexion and reports pain in the vertebral column

The nurse hears a client calling out for help, hurries down the hallway to the client's room, and finds the client lying on the floor. The nurse performs an assessment, assists the client back to bed, notifies the health care provider of the incident, and completes an incident report. Which statement should the nurse document on the incident report? A. The client fell out of bed B. The client climbed over the side rails C. The client was found lying on the floor D. The client became restless and tried to get out of bed

C. The client was found lying on the floor

A client is brought to the emergency department by emergency medical services (EMS) after being hit by a car. The name of the client is unknown, and the client has sustained a severe head injury and multiple fractures and is unconscious. An emergency craniotomy is required. Regarding informed consent for the surgical procedure, which is the *best* action? A. Obtain a court order for the surgical procedure B. Ask the EMS team to sign the informed consent C. Transport the victim to the operating room for surgery D. Call the police to identify the client and locate the family

C. Transport the victim to the operating room for surgery

A client with severe acne is seen in the clinic and the healthcare provider prescribes isotretinoin. The nurse reviews the clients medication record and would contact the health care provider if the client is also taking which medication? A. Digoxin B. Phenytoin C. Vitamin A D. Furosemide

C. Vitamin A Rationale: Isotretinoin is a metabolite of vitamin A and can produce generalized intensification of isotretinoin toxicity. Because of the potential for increased toxicity, vitamin A supplements should be discontinued before isotretinoin therapy. There are no contraindications associated with digoxin, phenytoin, or Furosemide.

The nurse manager is observing a new nursing graduate caring for a burn client in protective isolation. The nurse manager intervenes if the new nursing graduate planned to implement which unsafe component of protective isolation technique? A. Using sterile sheets and linens B. Performing strict hand washing technique C. Wearing gloves and a gown only when giving direct care to the client D. Wearing protective garb, including mask, gloves, cap, shoes covers, gowns, and plastic apron

C. Wearing gloves and a gown only when giving direct care to the client

A client with a diagnosis of asthma is admitted to the hospital with respiratory distress. Which type of adventitious lung sounds should the nurse expect to hear when performing a respiratory assessment on this client? A. Stridor B. Crackles C. Wheezes D. Diminished

C. Wheezes

A client scheduled for surgery is to receive an IV infusion of 0.9% sodium chloride with prochlorperazine edisylate (Compazine) 10 mg/50 ml over 30 minutes. The nurse should program the infusion pump to deliver how many ml/hour? (Enter numeric value only.)

Calculate: 50 ml : 30 minutes :: X ml : 60 minutes 50/X :: 30/60 3000 = 15X X = 100 ml

The nurse identified that the patient has pain on a scale of 7, he winces during movement, and he expresses discomfort over the incisional area. He guards the area by resisting movement. The incision appears to be healing, but there is natural swelling. Write a three-part nursing diagnostic statement using the PES format.

Correct Answer(s): P, acute pain; E, related to incisional trauma; S, evidenced by pain reported at 7, with guarding, and restricted turning and positioning. The PES format stands for: P (problem), E (etiology or related factor), and S (symptoms or defining characteristics).

For each of the following interventions, note which are direct and which are indirect nursing interventions. Place a D for direct or I for indirect in the space provided. 1. A nurse checks the monthly performance improvement report on fall occurrences on a unit. _______________ 2. A nurse discusses with the patient exercise restrictions to follow on return home. _______________ 3. A nurse consults with a dietitian about a patient's therapeutic diet food choices. _______________ 4. A nurse administers a tube feeding. _______________ 5. A nurse assists a colleague in applying a complex dressing to a patient's wound. _______________

Correct Answer(s): 1 (I), 2 (D), 3 (I), 4 (D), 5 (D). Direct activities are those that nurses perform through patient interactions. Indirect care measures include nurse actions aimed at management of the patient care environment and interdisciplinary collaborative actions that support the effectiveness of direct care interventions.

Match the activity on the left with the source of diagnostic error on the right: Activity a. Nurse listens to lungs for first time and is not sure if abnormal lung sounds are present. b. After reviewing objective data, nurse selects diagnosis of fear before asking patient to discuss feelings. c. Nurse identifies incorrect diagnostic label. d. Nurse does not consider patient's cultural background when reviewing cues. e. Nurse prepares to complete decision on diagnosis and realizes that clinical criteria are grouped incorrectly to form a pattern. Source of Diagnostic Error __ 1. Collecting data __ 2. Interpreting __ 3. Clustering __ 4. Labeling

Correct Answer(s): 1 a, 2 b and d, 3 e, 4 c. Choice a is an example of lack of skill, an error in collecting data. Choice b is an example of using an insufficient number of cues, an error in interpretation. Choice c is an example of not accurately identifying the problem, a labeling error. Choice d is an example of not incorporating cultural information into the diagnostic process, an error in interpretation. Choice e is an example of incorrect clustering, a clustering error.

A 72-year-old patient has come to the health clinic with symptoms of a productive cough, fever, increased respiratory rate, and shortness of breath. His respiratory distress increases when he walks. He lives alone and did not come to the clinic until his neighbor insisted. He reports not getting his pneumonia vaccine this year. Blood tests show the patient's oxygen saturation to be lower than normal. The physician diagnoses the patient as having pneumonia. Match the priority level with the nursing diagnoses identified for this patient: Nursing Diagnoses 1. Impaired gas exchange _____ 2. Risk for activity intolerance _____ 3. Ineffective self-health management _____ Priority Level a. Long term b. Short term c. Intermediate

Correct Answer(s): 1b, 2c, 3a. The patient's oxygenation status is the priority in this situation. The patient's condition creates the risk for activity intolerance, making this an intermediate priority for which the nurse must monitor. Ineffective self-help management is a long-term goal that might be applicable if the patient has physical limitations at the time of discharge.

An 82-year-old patient who resides in a nursing home has the following three nursing diagnoses: risk for fall, impaired physical mobility related to pain, and wandering related to cognitive impairment. The nursing staff identified several goals of care. Match the goals on the left with the appropriate outcome statements on the right. Goals 1. Patient will ambulate independently in 3 days. _____ 2. Patient will be injury free for 1 month. _____ 3. Patient will be less agitated. _____ 4. Patient will achieve pain relief. _____ Outcomes a. Patient will express fewer nonverbal signs of discomfort. b. Patient will follow a set care routine. c. Patient will walk correctly using a walker. d. Patient will exit a low bed without falling.

Correct Answer(s): 1c, 2d, 3b, 4a

A nurse is assigned to a new patient admitted to the nursing unit following admission through the emergency department. The nurse collects a nursing history and interviews the patient. Place the following steps for making a nursing diagnosis in the correct order. _____ 1. Considers context of patient's health problem and selects a related factor _____ 2. Reviews assessment data, noting objective and subjective clinical criteria _____ 3. Clusters clinical criteria that form a pattern _____ 4. Chooses diagnostic label

Correct Answer(s): 2, 3, 4, 1

The nurse follows a series of steps to objectively evaluate the degree of success in achieving outcomes of care. Place the steps in the correct order. 1. The nurse judges the extent to which the condition of the skin matches the outcome criteria. 2. The nurse tries to determine why the outcome criteria and actual condition of skin do not agree. 3. The nurse inspects the condition of the skin. 4. The nurse reviews the outcome criteria to identify the desired skin condition. 5. The nurse compares the degree of agreement between desired and actual condition of the skin.

Correct Answer(s): 4, 3, 5, 1, 2

The nurse completed the following assessment: 63-year-old female patient has had abdominal pain for 6 days. She reports not having a bowel movement for 4 days, whereas she normally has a bowel movement every 2 to 3 days. She has not been hospitalized in the past. Her abdomen is distended. She reports being anxious about upcoming tests. Her temperature was 37° C, pulse 82 and regular, blood pressure 128/72. Which additional data do you collect to add to the cluster of information?

Correct Answer(s): The best way to understand the answer to this question is to have a list of NANDA-I nursing diagnoses and their defining characteristics. For example, the nursing diagnosis of constipation is a possible choice. Examples of additional defining characteristics for which the nurse might assess include checking the quality of bowel sounds, palpating the abdomen for a possible mass, observing the character of any stool that is passed, asking the patient if she is passing flatus.

The following nursing diagnoses all apply to one patient. As the nurse adds these diagnoses to the care plan, which diagnoses will not include defining characteristics? A) Risk for aspiration B) Acute confusion C) Readiness for enhanced coping D) Sedentary lifestyle

Correct Answer(s): A A risk diagnosis does not have defining characteristics, but instead risk factors. Risk factors are the environmental, physiological, psychological, genetic, or chemical elements that place a person at risk for a health problem.

A patient signals the nurse by turning on the call light. The nurse enters the room and finds the patient's drainage tube disconnected, 100 mL of fluid in the intravenous (IV) line, and the patient asking to be turned. Which of the following does the nurse perform first? A) Reconnect the drainage tubing B) Inspect the condition of the IV dressing C) Improve the patient's comfort and turn onto her side. D) Obtain the next IV fluid bag from the medication room

Correct Answer(s): A The priority is to reconnect the drainage tube. This can be done quickly and prevents fluid loss and reduces risk of infection spreading up into the tube. Next the nurse turns the patient for comfort. With 100 mL of fluid remaining, the nurse has time to perform these tasks. The nurse can inspect the IV dressing last, after going to obtain the next IV fluid bag.

Before consulting with a physician about a patient's need for urinary catheterization, the nurse considers the fact that the patient has urinary retention and has been unable to void on her own. The nurse knows that evidence for alternative measures to promote voiding exists, but none has been effective, and that before surgery the patient was voiding normally. This scenario is an example of which implementation skill? A) Cognitive B) Interpersonal C) Psychomotor D) Consultative

Correct Answer(s): A Thinking and anticipating how to approach implementation involve a cognitive implementation skill. The nurse considers the rationale for an intervention and evidence in nursing science that supports that intervention or alternatives.

Unmet and partially met goals require the nurse to do which of the following? (Select all that apply.) A) Redefine priorities B) Continue intervention C) Discontinue care plan D) Gather assessment data on a different nursing diagnosis E) Compare the patient's response with that of another patient

Correct Answer(s): A, B When you determine that a goal has not been met or has been met only partially, intervention must continue; and the fact that the health problem still exists suggests that priorities may need to be redefined. You do not discontinue a plan unless a goal has been achieved. Evaluation never involves comparing a patient's data with that of another patient. A patient may develop new diagnoses at any time, but assessment of a new diagnosis does not address goals for an existing diagnosis.

During the implementation step of the nursing process, a nurse reviews and revises the nursing plan of care. Place the following steps of review and revision in the correct order: A) Review the care plan. B) Decide if the nursing interventions remain appropriate. C) Reassess the patient. D) Compare assessment findings to validate existing nursing diagnoses.

Correct Answer(s): A, B, C, D After reassessing a patient, the nurse reviews the care plan and compares assessment data to validate the nursing diagnoses and determine whether the nursing interventions remain the most appropriate for the clinical situation. If the patient's status has changed and the nursing diagnosis and related nursing interventions are no longer appropriate, the nurse modifies the nursing care plan.

A nurse on a cancer unit is reviewing and revising the written plan of care for a patient who has the nursing diagnosis of nausea. Place the following steps in their proper order: A) The nurse revises approaches in the plan for controlling environmental factors that worsen nausea. B) The nurse enters data in the assessment column showing new information about the patient's nausea. C) The nurse adds the current date to show that the diagnosis of nausea is still relevant. D) The nurse decides to use the patient's self-report of appetite and fluid intake as evaluation measures.

Correct Answer(s): A, B, C, D These steps follow the sequence needed for modifying an existing care plan.

Review the following list of nursing diagnoses and identify those stated incorrectly. (Select all that apply.) A) Acute pain related to lumbar disk repair B) Sleep deprivation related to difficulty falling asleep C) Constipation related to inadequate intake of liquids D) Potential nausea related to nasogastric tube insertion

Correct Answer(s): A, B, D Acute pain related to lumbar disk repair uses a medical diagnosis as a related factor. Sleep deprivation related to difficulty falling asleep uses a clinical sign rather than a treatable etiology such as "excess noise in environment." Potential nausea related to nasogastric tube insertion uses a diagnostic study as the etiology. None of the etiologies can be managed or treated by nursing intervention.

A nurse is starting on the evening shift and is assigned to care for a patient with a diagnosis of impaired skin integrity related to pressure and moisture on the skin. The patient is 72 years old and had a stroke. The patient weighs 250 pounds and is difficult to turn. As the nurse makes decisions about how to implement skin care for the patient, which of the following actions does the nurse implement? (Select all that apply.) A) Review the set of all possible nursing interventions for the patient's problem B) Review all possible consequences associated with each possible nursing action C) Consider own level of competency D) Determine the probability of all possible consequences

Correct Answer(s): A, B, D When making decisions about implementation, reviewing all possible interventions and consequences and determining the probability of consequences are necessary steps. The nurse is responsible for having the necessary knowledge and clinical competency to perform an intervention, but this is not part of the decision making involved.

A nurse identifies several interventions to resolve the patient's nursing diagnosis of impaired skin integrity. Which of the following are written in error? (Select all that apply.) A) Turn the patient regularly from side to back to side. B) Provide perineal care, using Dove soap and water, every shift and after each episode of urinary incontinence. C) Apply a pressure-relief device to bed. D) Apply transparent dressing to sacral pressure ulcer.

Correct Answer(s): A, C The statements "Turn the patient regularly from side to back to side" and "Apply a pressure-relief device to bed" do not provide specific guidelines for the frequency or type of intervention. The other two options identify specific intervention methods.

A patient has been in the hospital for 2 days because of newly diagnosed diabetes. His medical condition is unstable, and the medical staff is having difficulty controlling his blood sugar. The physician expects that the patient will remain hospitalized at least 3 more days. The nurse identifies one nursing diagnosis as deficient knowledge regarding insulin administration related to inexperience with disease management. What does the nurse need to determine before setting the goal of "patient will self-administer insulin?" (Select all that apply.) A) Goal within reach of the patient B) The nurse's own competency in teaching about insulin C) The patient's cognitive function D) Availability of family members to assist

Correct Answer(s): A, C, D A goal must be realistic and one that the patient has cognitive and sociocultural potential to reach. The nurse's competency does not influence the patient's goal. However, it may mean that the nurse must consult with a diabetes educator or a more qualified nurse before beginning instruction.

Which of the following statements correctly describe the evaluation process? (Select all that apply.) A) Evaluation is an ongoing process. B) Evaluation usually reveals obvious changes in patients. C) Evaluation involves making clinical decisions. D) Evaluation requires the use of assessment skills.

Correct Answer(s): A, C, D Evaluation often reveals changes that are not obvious. Changes are often subtle and occur over a period of time.

Which steps does the nurse follow when he or she is asked to perform an unfamiliar procedure? (Select all that apply.) A) Seeks necessary knowledge B) Reassesses the patient's condition C) Collects all necessary equipment D) Delegates the procedure to a more experienced staff member E) Considers all possible consequences of the procedure

Correct Answer(s): A, C, E You require additional knowledge and skills in situations in which you are less experienced. When you are asked to administer a new procedure with which you are unfamiliar, follow the three choices: seek necessary knowledge, collect necessary equipment, and consider all possible consequences of the procedure. Collecting necessary equipment and considering potential consequences is needed for any procedure.

Review the following nursing diagnoses and identify the diagnoses that are stated correctly. (Select all that apply.) A) Anxiety related to fear of dying B) Fatigue related to chronic emphysema C) Need for mouth care related to inflamed mucosa D) Risk for infection

Correct Answer(s): A, D The diagnosis "Anxiety related to fear of dying" is stated correctly, with the related factor being the patient's response to a health problem. Risk for infection is a risk factor for an at-risk diagnosis. In all cases the related factor or risk factor is a condition for which the nurse can implement preventive measures. Fatigue related to chronic emphysema is incorrect since chronic emphysema is a medical diagnosis. Need for mouth care related to inflamed mucosa is not a NANDA-I-approved nursing diagnosis.

A nurse from home health is talking with a nurse who works on an acute medical division within a hospital. The home health nurse is making a consultation. Which of the following statements describes the unique difference between a nursing care plan from a hospital versus one for home care? A) The goals of care will always be more long term. B) The patient and family need to be able to independently provide most of the health care. C) The patient's goals need to be mutually set with family members who will care for him or her. D) The expected outcomes need to address what can be influenced by interventions.

Correct Answer(s): B A community-based health care setting such as home health must work with patients and their families to set goals and outcomes that ultimately lead to a plan that allows them to provide the majority of care themselves. Goals of care will not always be more long term; goals will be short term and long term, depending on the patient's condition. Mutually setting goals with caregiving family members is true for any health care setting. The statement "The expected outcomes need to address what can be influenced by interventions" is incorrect; the outcomes allow you to direct your evaluation of care.

A patient is being discharged after abdominal surgery. The abdominal incision is healing well with no signs of redness or irritation. Following instruction, the patient has demonstrated effective care of the incision, including cleansing the wound and applying dressings correctly to the nurse. These behaviors are an example of: A) Evaluative measure. B) Expected outcome. C) Reassessment. D) Standard of care.

Correct Answer(s): B An expected outcome is an end result that is measureable, desirable, observable, and translates into observable patient behaviors. It is a measure that tells you if the educational interventions led to successful goal achievement, the patient's self-care of the wound. An evaluative measure would be the process of observing the patient. Reassessment is a behavior performed by the nurse. The type of wound cleanser and dressings would be a standard of care.

The nurse reviews a patient's medical record and sees that tube feedings are to begin after a feeding tube is inserted. In recent past experiences the nurse has seen patients on the unit develop diarrhea from tube feedings. The nurse consults with the dietitian and physician to determine the initial rate that will be ordered for the feeding to lessen the chance of diarrhea. This is an example of what type of direct care measure? A) Preventive B) Controlling for an adverse reaction C) Consulting D) Counseling

Correct Answer(s): B Anticipating the need to start the feeding at a slower rate is an example of controlling for an adverse reaction, which in this case would be a harmful or unintended effect (diarrhea) of therapeutic intervention.

A nurse checks a physician's order and notes that a new medication was ordered. The nurse is unfamiliar with the medication. A nurse colleague explains that the medication is an anticoagulant used for postoperative patients with risk for blood clots. The nurse's best action before giving the medication is to: A) Have the nurse colleague check the dose with her before giving the medication. B) Consult with a pharmacist to obtain knowledge about the purpose of the drug, the action, and the potential side effects. C) Ask the nurse colleague to administer the medication to her patient. D) Administer the medication as prescribed and on time.

Correct Answer(s): B When a nurse performs a new or unfamiliar procedure, such as giving a new medication, it is important to assess personal competency and determine if new knowledge or assistance is needed. The nurse's best action is to check with the pharmacist about the medication. Having another nurse check the dosage is appropriate if the nurse is still uncertain about the medication. Once the nurse feels prepared, the medication is administered as prescribed. You never ask a colleague to give a medication to a patient to whom you are assigned.

A patient is recovering from surgery for removal of an ovarian tumor. It is 1 day after her surgery. Because she has an abdominal incision and dressing and a history of diabetes, the nurse has selected a nursing diagnosis of risk for infection. Which of the following is an appropriate goal statement for the diagnosis? A) Patient will remain afebrile to discharge. B) Patient's wound will remain free of infection by discharge. C) Patient will receive ordered antibiotic on time over next 3 days. D) Patient's abdominal incision will be covered with a sterile dressing for 2 days.

Correct Answer(s): B When selecting an at-risk diagnosis, the goal is to avoid or prevent the condition at risk, in this case infection. The statement "Patient will remain afebrile to discharge" is a potential outcome measure for the goal. The patient receiving an ordered antibiotic and having the abdominal incision covered are both interventions.

A nurse is assigned to a patient who has returned from the recovery room following surgery for a colorectal tumor. After an initial assessment the nurse anticipates the need to monitor the patient's abdominal dressing, intravenous (IV) infusion, and function of drainage tubes. The patient is in pain, reporting 6 on a scale of 0 to 10, and will not be able to eat or drink until intestinal function returns. The family has been in the waiting room for an hour, wanting to see the patient. The nurse establishes priorities first for which of the following situations? (Select all that apply.) A) The family comes to visit the patient. B) The patient expresses concern about pain control. C) The patient's vital signs change, showing a drop in blood pressure. D) The charge nurse approaches the nurse and requests a report at end of shift.

Correct Answer(s): B, C Pain control is a priority, because it is severe and affects the patient's ability to rest after surgery and be able to perform necessary activities. A change in vital signs is a priority, and the change could be related to the patient's pain. However, because of the nature of surgery, the nurse has to reassess for any bleeding, which lowers blood pressure. Attending to the family is important to lend the patient needed support, but it is not the initial priority. Finally the nurse must attend to urgent patient needs before completing a report.

In which of the following examples is a nurse applying critical thinking attitudes when preparing to insert an intravenous (IV) catheter? (Select all that apply.) A) Following the procedural guideline for IV insertion B) Seeking necessary knowledge about the steps of the procedure from a more experienced nurse C) Showing confidence in performing the correct IV insertion technique D) Being sure that the IV dressing covers the IV site completely

Correct Answer(s): B, C Seeking necessary knowledge about the steps of the procedure shows humility. The nurse recognizes that she needs clarification from a senior colleague. Another example of a critical thinking attitude is confidence. In this case confidently inserting an IV line allows the nurse to convey expertise and a sense of calm, leading the patient to trust the nurse. Following policy and procedure is an example of following standards of care, not of a critical thinking attitude. Making sure that the dressing is covered is a step in following good standards of IV care but is not a critical thinking attitude.

A nurse assesses a 78-year-old patient who weighs 240 pounds (108.9 kg) and is partially immobilized because of a stroke. The nurse turns the patient and finds that the skin over the sacrum is very red and the patient does not feel sensation in the area. The patient has had fecal incontinence on and off for the last 2 days. The nurse identifies the nursing diagnosis of risk for impaired skin integrity. Which of the following goals are appropriate for the patient? (Select all that apply.) A) Patient will be turned every 2 hours within 24 hours. B) Patient will have normal bowel function within 72 hours. C) Patient's skin will remain intact through discharge. D) Patient's skin condition will improve by discharge.

Correct Answer(s): B, C The skin remaining intact is an appropriate goal for the patient's at-risk diagnosis. A return of normal bowel functioning is also appropriate since it indicates removal of a risk factor. Turning the patient is an intervention; skin condition improving by discharge is a poorly written goal that is not measurable.

A clinic nurse assesses a patient who reports a loss of appetite and a 15-pound weight loss since 2 months ago. The patient is 5 feet 10 inches tall and weighs 135 pounds (61.2 kg). She shows signs of depression and does not have a good understanding of foods to eat for proper nutrition. The nurse makes the nursing diagnosis of imbalanced nutrition: less than body requirements related to reduced intake of food. For the goal of, "Patient will return to baseline weight in 3 months," which of the following outcomes would be appropriate? (Select all that apply.) A) Patient will discuss source of depression by next clinic visit. B) Patient will achieve a calorie intake of 2400 daily in 2 weeks. C) Patient will report improvement in appetite in 1 week. D) Patient will identify food protein sources.

Correct Answer(s): B, C With the related factor of reduced intake of food, the outcomes should focus on behaviors that reflect an increase in intake. Thus achieving an increase in calories and an improved appetite for food would be appropriate. The patient's depression probably contributes to the loss of appetite, but being able to discuss the source of depression is not an outcome for improving her baseline weight. Being able to identify protein sources would improve any knowledge deficit the patient might have but would not help her gain weight.

The nurse enters a patient's room and finds that the patient was incontinent of liquid stool. The patient has recurrent redness in the perineal area, and there is concern that he is developing a pressure ulcer. The nurse cleanses the patient, inspects the skin, and applies a skin barrier ointment to the perineal area. She calls the ostomy and wound care specialist and asks that he visit the patient to recommend skin care measures. Which of the following describe the nurse's actions? (Select all that apply.) A) The application of the skin barrier is a dependent care measure. B) The call to the ostomy and wound care specialist is an indirect care measure. C) The cleansing of the skin is a direct care measure. D) The application of the skin barrier is a direct care measure.

Correct Answer(s): B, C, D The call to the ostomy and wound care specialist is an indirect care measure involving collaborative care. Cleansing the skin is an independent direct care measure. Applying the skin barrier is an independent nursing measure involving direct care.

The nurse in a geriatric clinic collects the following information from an 82-year-old patient and her daughter, the family caregiver. The daughter explains that the patient is "always getting lost." The patient sits in the chair but gets up frequently and paces back and forth in the examination room. The daughter says, "I just don't know what to do because I worry she will fall or hurt herself." The daughter states that, when she took her mother to the store, they became separated, and the mother couldn't find the front entrance. The daughter works part time and has no one to help watch her mother. Which of the data form a cluster, showing a relevant pattern? (Select all that apply.) A) Daughter's concern of mother's risk for injury B) Pacing C) Patient getting lost easily D) Daughter working part time E) Getting up frequently

Correct Answer(s): B, C, E Pacing, getting lost, and hyperactivity are a cluster of defining characteristics that point to the diagnostic label of wandering.

A nurse caring for a patient with pneumonia sits the patient up in bed and suctions the patient's airway. After suctioning, the patient describes some discomfort in his abdomen. The nurse auscultates the patient's lung sounds and gives him a glass of water. Which of the following would be appropriate evaluative criteria used by the nurse? (Select all that apply.) A) Patient drinks contents of water glass. B) Patient's lungs are clear to auscultation in bases. C) Patient reports abdominal pain on scale of 0 to 10. D) Patient's rate and depth of breathing are normal with head of bed elevated.

Correct Answer(s): B, D The criteria of clear lung sounds and rate and depth of breathing are evaluative criteria for determining if the patient's airway is clear. Drinking the contents of the water glass is a completed intervention. The patient's report of pain is assessment data.

Which of the following outcome statements for the goal, "Patient will achieve a gain of 10 lbs (4.5 kg) in body weight in a month" are worded incorrectly? (Select all that apply.) A) Patient will eat at least three fourths of each meal by 1 week. B) Patient will verbalize relief of nausea and have no episodes of vomiting in 1 week. C) Patient will eat foods with high-calorie content by 1 week. D) Give patient liquid supplements 3 times a day

Correct Answer(s): B, D The statement "Patient will verbalize relief of nausea and have no episodes of vomiting in 1 week" is not singular. The statement "Give patient liquid supplements 3 times a day" is an intervention.

A patient comes to a medical clinic with the diagnosis of asthma. The nurse practitioner decides that the patient's obesity adds to the difficulty of breathing; the patient is 5 feet 7 inches tall and weighs 200 pounds (90.7 kg). Based on the nursing diagnosis of imbalanced nutrition: more than body requirements, the practitioner plans to place the patient on a therapeutic diet. Which of the following are evaluative measures for determining if the patient achieves the goal of a desired weight loss? (Select all that apply.) A) The patient eats 2000 calories a day. B) The patient is weighed during each clinic visit. C) The patient discusses factors that increase the risk of an asthma attack. D) The patient's food diary that tracks intake of daily meals is reviewed.

Correct Answer(s): B, D Weighing the patient during each clinic visit and reviewing a food diary indicate whether weight loss is occurring and if the patient is eating the proper foods designed to reduce his or her weight.

The nurse completed the following assessment: 63-year-old female patient has had abdominal pain for 6 days. She reports not having a bowel movement for 4 days, whereas she normally has a bowel movement every 2 to 3 days. She has not been hospitalized in the past. Her abdomen is distended. She reports being anxious about upcoming tests. Her temperature was 37° C, pulse 82 and regular, blood pressure 128/72. Which of the following data form a cluster, showing a relevant pattern? (Select all that apply.) A) Vital sign results B) Abdominal distention C) Age of patient D) Change in bowel elimination pattern E) Abdominal pain F) No past history of hospitalization

Correct Answer(s): B, D, E The presence of abdominal pain, distention, and a change in bowel elimination pattern forms a cluster, suggesting an elimination problem.

The nurse enters a patient's room, and the patient asks if he can get out of bed and transfer to a chair. The nurse takes precautions to use safe patient handling techniques and transfers the patient. This is an example of which physical care technique? A) Meeting the patient's expressed wishes B) Indirect care measure C) Protecting a patient from injury D) Staying organized when implementing a procedure

Correct Answer(s): C A common method for administering physical care techniques appropriately includes protecting you and your patients from injury, which involves safe patient handling. Transferring a patient is a direct care measure. Organization is an aspect of physical care but not an example of this nurse's action. Although meeting patient needs is important, it is not a physical care technique.

A nursing student is talking with one of the staff nurses who works on a surgical unit. The student's care plan is to include nursing-sensitive outcomes for the nursing diagnosis of acute pain. A nursing-sensitive outcome suitable for this diagnosis would be: A) Patient will achieve pain relief by discharge. B) Patient will be free of a surgical wound infection by discharge. C) Patient will report reduced pain severity in 2 days. D) Patient will describe purpose of pain medicine by discharge.

Correct Answer(s): C An example of a nursing-sensitive outcome is one that is influenced and sensitive to nursing interventions. Such is the case with "reduction in pain severity." The patient achieving pain relief by discharge is a goal. The patient being free of a surgical wound infection by discharge is a medical outcome. The patient describing the purpose of pain medication by discharge is an outcome for a knowledge problem but not for the diagnosis of acute pain.

Which outcome allows you to measure a patient's response to care more precisely? A) The patient's wound will appear normal within 3 days. B) The patient's wound will have less drainage within 72 hours. C) The patient's wound will reduce in size to less than 4 cm (1 ½ inches) by day 4. D) The patient's wound will heal without redness or drainage by day 4.

Correct Answer(s): C An outcome must have terms describing quality, quantity, frequency, length, or weight to allow for precise measurement. The statement "The patient's wound will reduce in size to less than 4 cm (1 ½ inches) by day 4" identifies a specific wound size, which indicates a degree of healing. The outcome statements concerning the wound appearing normal and having less drainage are vague and not measurable. The statement "The patient's wound will heal without redness or drainage by day 4" has more than one outcome.

A nurse caring for a patient with pneumonia sits the patient up in bed and suctions his airway. After suctioning, the patient describes some discomfort in his abdomen. The nurse auscultates the patient's lung sounds and gives him a glass of water. Which of the following is an evaluative measure used by the nurse? A) Suctioning the airway B) Sitting patient up in bed C) Auscultating lung sounds D) Patient describing type of discomfort

Correct Answer(s): C Auscultation was the measure used to determine if the suctioning of the airway was effective. Suctioning and sitting the patient up are interventions. The nurse did not ask the patient or evaluate the nature of the pain.

Two nurses are having a discussion at the nurses' station. One nurse is a new graduate who added, "Patient needs improved bowel function related to constipation" to a patient's care plan. The nurse's colleague, the charge nurse says, "I think your diagnosis is possibly worded incorrectly. Let's go over it together." A correctly worded diagnostic statement is: A) Need for improved bowel function related to change in diet. B) Patient needs improved bowel function related to alteration in elimination. C) Constipation related to inadequate fluid intake. D) Constipation related to hard infrequent stools.

Correct Answer(s): C Constipation related to inadequate fluid intake is an accurate NANDA-I approved nursing diagnosis with an appropriate etiology. Need for improved bowel function related to change in diet is a goal with an etiologic factor. Patient needs improved bowel function related to alteration in elimination is a goal with a diagnostic statement. Constipation related to hard infrequent stools is a nursing diagnostic label with a clinical sign.

A nurse is preparing for change-of-shift rounds with the nurse who is assuming care for his patients. Which of the following statements or actions by the nurse are characteristics of ineffective handoff communication? A) This patient is anxious about his pain after surgery; you need to review the information I gave him about how to use a patient-controlled analgesia (PCA) pump this evening. B) The nurse refers to the electronic care plan in the electronic health record (EHR) to review interventions for the patient's care. C) During walking rounds the nurse talks about the problem the patient care technicians created by not ambulating the patient. D) The nurse gives her patient a pain medication before report so there is likely to be no interruption during rounding.

Correct Answer(s): C Creating a culture of blame does not support questioning, which is needed for good handoff communication. Talking about the patient's anxiety during handoff is patient centered and thus appropriate, referring to the EHR to review interventions ensures that essential information is included, and administering a pain medication before the report allows the nurse to be organized and uninterrupted during rounds.

When does implementation begin as the fourth step of the nursing process? A) During the assessment phase B) Immediately in some critical situations C) After the care plan has been developed D) After there is mutual goal setting between nurse and patient

Correct Answer(s): C Implementation begins after the nurse has developed the plan of care. Even in emergent situations a nurse assesses a situation quickly, considers options, and then implements nursing measures. Goal setting is part of planning.

A nurse reviews data gathered regarding a patient's pain symptoms. The nurse compares the defining characteristics for acute pain with those for chronic pain and in the end selects acute pain as the correct diagnosis. This is an example of the nurse avoiding an error in: A) Data collection. B) Data clustering. C) Data interpretation. D) Making a diagnostic statement.

Correct Answer(s): C In the review of data, the nurse compares defining characteristics for the two nursing diagnoses and selects one based on the interpretation of data. Making a diagnostic statement is incorrect because the nurse has not included a related factor.

A patient has been in the hospital for 2 days because of newly diagnosed diabetes. His medical condition is unstable, and the medical staff is having difficulty controlling his blood sugar. The physician expects that the patient will remain hospitalized at least 3 more days. The nurse identifies one nursing diagnosis as deficient knowledge regarding insulin administration related to inexperience with disease management. Which of the following patient care goals are long term? A) Patient will explain relationship of insulin to blood glucose control. B) Patient will self-administer insulin. C) Patient will achieve glucose control. D) Patient will describe steps for preparing insulin in a syringe.

Correct Answer(s): C It will take time for the patient who is medically unstable to achieve glucose control. Explaining the relationship of insulin to blood glucose control and self-administering insulin are short term goals and should be met before discharge. Describing steps for preparing insulin in a syringe is not a goal but an outcome statement for the goal that the patient will self-administer insulin.

A nurse is orienting a new graduate nurse to the unit. The graduate nurse asks, "Why do we have standing orders for cases when patients develop life-threatening arrhythmias? Is not each patient's situation unique?" What is the nurse's best answer? A) Standing orders are used to meet our physician's preferences. B) Standing orders ensure that we are familiar with evidence-based guidelines for care of arrhythmias. C) Standing orders allow us to respond quickly and safely to a rapidly changing clinical situation. D) Standing orders minimize the documentation we have to provide.

Correct Answer(s): C Standing orders are preprinted documents containing orders for the conduct of routine therapies, monitoring guidelines, and/or diagnostic procedures for specific patients with identified clinical problems. They are common in critical care settings and other specialized practice settings in which patients' needs change rapidly and require immediate attention.

A patient is being discharged today. In preparation the nurse removes the intravenous (IV) line from the right arm and documents that the site was "clean and dry with no signs of redness or tenderness." On discharge the nurse reviews the care plan for goals met. Which of the following goals can be evaluated with what you know about this patient? A) Patient expresses acceptance of health status by day of discharge. B) Patient's surgical wound will remain free of infection. C) Patient's IV site will remain free of phlebitis. D) Patient understands when to call physician to report possible complications.

Correct Answer(s): C To achieve the goal of preventing phlebitis the nurse evaluates for signs of phlebitis, which include redness or inflammation. The outcome for this goal would be stated as, "IV site will show no signs of inflammation to discharge."

The nurse checks the intravenous (IV) solution that is infusing into the patient's left arm. The IV solution of 9% NS is infusing at 100 mL/hr as ordered. The nurse reviews the nurses' notes from the previous shift to determine if the dressing over the site was changed as scheduled per standard of care. While in the room, the nurse inspects the condition of the dressing and notes the date on the dressing label. In what ways did the nurse evaluate the IV intervention? (Select all that apply.) A) Checked the IV infusion location in left arm B) Checked the type of IV solution C) Confirmed from nurses' notes the time of dressing change and checked label D) Inspected the condition of the IV dressing

Correct Answer(s): C, D The evaluation of interventions examines two factors: the appropriateness of the interventions selected (whether the IV dressing was changed as the standard of care requires) and the correct application of the intervention (whether the dressing was in place and secure). Checking the IV infusion location in the left arm is an evaluation measure, and checking the type of IV solution is an assessment step to ensure that correct fluid is infusing.

In the following examples, which nurses are making nursing diagnostic errors? (Select all that apply.) A) The nurse who listens to lung sounds after a patient reports "difficulty breathing" B) The nurse who considers conflicting cues in deciding which diagnostic label to choose C) The nurse assessing the edema in a patient's lower leg who is unsure how to assess the severity of edema D) The nurse who identifies a diagnosis on the basis of a single defining characteristic

Correct Answer(s): C, D When the nurse assesses edema without knowing how to assess the severity, the nurse fails to validate her assessment findings of edema, either by using a scale to measure the severity or by asking a colleague to validate her findings. In identifying a diagnosis on the basis of a single defining characteristic, the nurse prematurely closes clustering, which can lead to an inaccurate diagnosis. By listening to lung sounds after the patient reports "difficulty breathing" the nurse validates findings to make an accurate diagnosis. The nurse interprets cue clusters to make an accurate diagnosis when considering conflicting cues to make a diagnosis.

A patient has limited mobility as a result of a recent knee replacement. The nurse identifies that he has altered balance and assists him in ambulation. The patient uses a walker presently as part of his therapy. The nurse notes how far the patient is able to walk and then assists him back to his room. Which of the following is an evaluative measure? A) Uses walker during ambulation B) Presence of altered balance C) Limited mobility in lower extremities D) Observation of distance patient is able to walk

Correct Answer(s): D An evaluative measure determines a patient's response to therapy, in this case how well the patient is able to ambulate (distance walked).

The nurse writes an expected-outcome statement in measurable terms. An example is: A) Patient will be pain free. B) Patient will have less pain. C) Patient will take pain medication every 4 hours. D) Patient will report pain acuity less than 4 on a scale of 0 to 10.

Correct Answer(s): D Answer 4 is measurable because it is the only outcome statement that allows the nurse to obtain an actual measure of the patient's pain. The patient being pain free is a goal; the patient having less pain is written vaguely, and the patient taking pain medication every 4 hours is an intervention.

A nurse is reviewing a patient's list of nursing diagnoses in the medical record. The most recent nursing diagnosis is diarrhea related to intestinal colitis. This is an incorrectly stated diagnostic statement, best described as: A) Identifying the clinical sign instead of an etiology. B) Identifying a diagnosis based on prejudicial judgment. C) Identifying the diagnostic study rather than a problem caused by the diagnostic study. D) Identifying the medical diagnosis instead of the patient's response to the diagnosis.

Correct Answer(s): D In this example intestinal colitis is a medical diagnosis and thus an incorrect diagnostic statement.

A patient has the nursing diagnosis of nausea. The nurse develops a care plan with the following interventions. Which are examples of collaborative interventions? A) Provide frequent mouth care. B) Maintain intravenous (IV) infusion at 100 mL/hr. C) Administer prochlorperazine (Compazine) via rectal suppository. D) Consult with dietitian on initial foods to offer patient. E) Control aversive odors or unpleasant visual stimulation that triggers nausea.

Correct Answer(s): D Providing frequent mouth care and controlling outside stimulation that triggers nausea are independent interventions. Maintaining an IV infusion and administering the rectal suppository are dependent interventions.

When a nurse properly positions a patient and administers an enema solution at the correct rate for the patient's tolerance, this is an example of what type of implementation skill? A) Interpersonal B) Cognitive C) Collaborative D) Psychomotor

Correct Answer(s): D Psychomotor skills require the integration of cognitive and motor activities to ensure safe intervention.

A goal specifies the expected behavior or response that indicates: A) The specific nursing action was completed. B) The validation of the nurse's physical assessment. C) The nurse has made the correct nursing diagnoses. D) Resolution of a nursing diagnosis or maintenance of a healthy state.

Correct Answer(s): D The success in meeting a goal is reflected in achieving expected outcomes—the physiological responses or behaviors that indicate that a nursing diagnosis has been resolved and the patient's health is improving.

Setting a time frame for outcomes of care serves which of the following purposes? A) Indicates which outcome has priority B) Indicates the time it takes to complete an intervention C) Indicates how long a nurse is scheduled to care for a patient D) Indicates when the patient is expected to respond in the desired manner

Correct Answer(s): D The time frame indicates when you expect a response to your nursing interventions. Time frames help to organize priorities but do not indicate which problem is most important. Time frames for outcomes are not used to gauge the time it takes to complete interventions, and they are unrelated to a nurse's work schedule.

The evaluation process includes interpretation of findings as one of its five elements. Which of the following is an example of interpretation? A) Evaluating the patient's response to selected nursing interventions B) Selecting an observable or measurable state or behavior that reflects goal achievement C) Reviewing the patient's nursing diagnoses and establishing goals and outcome statements D) Matching the results of evaluative measures with expected outcomes to determine patient's status

Correct Answer(s): D When interpreting findings, you compare the patient's behavioral responses and physiological signs and symptoms that you expect to see with those actually seen from your evaluation.

A nurse is preparing to perform a cultural assessment of a patient. Which of the following questions is an example of a contrast question? a. Tell me about your ethnic background. b. Have you had this problem in the past? c. Where do other members of your family live? d. How different is this problem from the one you had previously?

D "Tell me about your ethnic background" is an example of an ethnohistory statement. "Have you had this problem in the past?" is an example of a focused question, and "Where do other members of your family live?" is an example of a social organization question

The nurse is assessing the character of a patient's migraine headache and asks, "Do you feel nauseated when you have a headache?" The patient's response is "yes." In this case the finding of nausea is which of the following? A) An objective finding B) A clinical inference C) A validation D) A concomitant symptom

D) A concomitant symptom

A nurse assesses a patient who comes to the pulmonary clinic. "I see that it's been over 6 months since you've been in, but your appointment was for every 2 months. Tell me about that. Also I see from your last visit that the doctor recommended routine exercise. Can you tell me how successful you have been following his plan?" The nurse's assessment covers which of Gordon's functional health patterns? A) Value-belief pattern B) Cognitive-perceptual pattern C) Coping-stress-tolerance pattern D) Health perception-health management pattern

D) Health perception-health management pattern

A hospitalized client tells the nurse that an instructional directive is being prepared and that the lawyer will be bringing the document to the hospital today for witness signatures. The client asks the nurse for assistance in obtaining a witness to the will. Which is the *most appropriate* response to the client? A. "I will sign as a witness to your signature" B. "You will need to find a witness on your own" C. "Whoever is available at the time will sign as a witness for you" D. "I will call the nursing supervisor to seek assistance regarding your request"

D. "I will call the nursing supervisor to seek assistance regarding your request"

The nurse educator asks a student to list the 5 main categories of complementary and alternative medicine (CAM), developed by the National Center for Complementary and Alternative Medicine. Which statement, if made by the nursing student, indicates a *need for further teaching* regarding CAM categories? A. "CAM includes biologically based practices" B. "Whole medical systems are a component of CAM" C. "Mind-body medicine is part of the CAM approach" D. "Magnetic therapy and massage therapy are a focus of CAM"

D. "Magnetic therapy and massage therapy are a focus of CAM"

The community health nurse is instructing a group of young female clients about breast self-examination. The nurse should instruct the clients to perform the examination at which time? A. At the onset of menstruation B. Every month during ovulation C. Weekly at the same time of day D. 1 week after menstruation begins

D. 1 week after menstruation begins

A client arrives at the emergency department following a burn injury that occurred in the basement at home, and an inhalation injury is suspected. What would the nurse anticipate to be prescribed for the client? A. 100% oxygen via an aerosol mask B. Oxygen via nasal cannula at 6 L/minute C. Oxygen via nasal cannula at 15 L/minute D. 100% oxygen via a tight-fitting, nonrebreather face mask

D. 100% oxygen via a tight-fitting, nonrebreather face mask

The nurse employed in a long-term care facility is planning assignments for the clients on a nursing unit. The nurse needs to assign four clients and has a licensed practical (vocational) nurse and 3 unlicensed assistive personnel (UAPs) on a nursing team. Which client would the nurse *most appropriately* assign to the licensed practical (vocational) nurse? A. A client who requires a bed bath B. An older client requiring frequent ambulation C. A client who requires hourly vital sign measurements D. A client requiring abdominal wound irrigations and dressing changes every 3 hours

D. A client requiring abdominal wound irrigations and dressing changes every 3 hours

The nurse has received the assignment for the day shift. After making initial rounds and checking all of the assigned clients, which client should the nurse plan to care for *first*? A. A client who is ambulatory demonstrating steady gait B. A postoperative client who has just received an opioid pain medication C. A client scheduled for physical therapy for hte first crutch-walking session D. A client with a white blood cell count of 14,000 mm^3 (14 x 10^9/L) and a temperature of 38.4°C

D. A client with a white blood cell count of 14,000 mm^3 (14 x 10^9/L) and a temperature of 38.4°C

The nurse employed in an emergency department is assigned to triage clients coming to the emergency department for treatment on the evening shift. The nurse should assign *priority* to which client? A. A client complaining of muscle aches, a headache, and history of seizures B. A client who twisted her ankle when rollerblading and is requesting medication for pain C. A client with a minor laceration on the index finger sustained while cutting an eggplant D. A client with chest pain who states that he just ate pizza that was made with a very spicy sauce

D. A client with chest pain who states that he just ate pizza that was made with a very spicy sauce

When assessing a lesion diagnosed as basal cell carcinoma, the nurse *most likely* expects to note which findings? *Select all that apply.* A. An irregularly shaped lesion B. A small papule with a dry, rough scale C. A firm, nodular lesion topped with crust D. A pearly papule with a central crater and a waxy border E. Location in the bald spot atop the head that is exposed to outdoor sunlight

D. A pearly papule with a central crater and a waxy border E. Location in the bald spot atop the head that is exposed to outdoor sunlight

A client arriving at the emergency department has experienced frostbite to the right hand. Which finding would the nurse note on assessment of the client's hand? A. A pink, edematous hand B. Fiery red skin with edema in the nail beds C. Black fingertips surrounded by an erythematous rash D. A white color to the skin, which is insensitive to touch

D. A white color to the skin, which is insensitive to touch

During the admission assessment of a client with advanced ovarian cancer, the nurse recognizes which manifestation as typical of the disease? A. Diarrhea B. Hypermenorrhea C. Abnormal bleeding D. Abdominal distention

D. Abdominal distention

The Camp nurse asked the children preparing to swim in the lake if they have applied sunscreen. The nurse reminds the children that chemical sunscreens are *most effective* when applied at which times? A. Immediately before swimming B. 5 minutes before exposure to the sun C. Immediately before exposure to the sun D. At least 30 minutes before exposure to the sun

D. At least 30 minutes before exposure to the sun. Rationale: Sunscreens are most effective when applied at least 30 minutes before exposure to the sun so that they can penetrate the skin. All sunscreen should be reapplied after swimming or sweating.

The nurse is inserting an intravenous (IV) line into a client's vein. After the initial stick, the nurse would continue to advance the catheter in which situation? A. The catheter advances easily B. The vein is distended under the needle C. The client does not complain of discomfort D. Blood return shows in the backlash chamber of the catheter

D. Blood return shows in the backlash chamber of the catheter

The nurse manager has implemented a change in the method of the nursing delivery system from functional to team nursing. An unlicensed assistive personnel (UAP) is resistant to the change and is not taking an active part in facilitating the process of change. Which is the *best* approach in dealing with the UAP? A. Ignore the resistance B. Exert coercion on the UAP C. Provide a positive reward system for the UAP D. Confront the UAP to encourage verbalization of feelings regarding the change

D. Confront the UAP to encourage verbalization of feelings regarding the change

The nurse is giving a bed bath to an assigned client when an unlicensed assistive personnel (UAP) enters the client's room and tells the nurse that another assigned client is in pain and needs pain medication. Which is the *most appropriate* nursing action? A. Finish the bed bath and then administer the pain medication to the other client B. Ask the UAP to find out when the last pain medication was given to the client C. Ask the UAP to tell the client in pain that medication will be administered as soon as the bed bath is complete D. Cover the client, raise the side rails, tell the client that you will return shortly, and administer the pain medication to the other client

D. Cover the client, raise the side rails, tell the client that you will return shortly, and administer the pain medication to the other client

A client receiving parenteral nutrition (PN) in the home setting has a weight gain of 5 lb in 1 week. The nurse should *next* assess the client for the presence of which condition? A. Thirst B. Polyuria C. Decreased BP D. Crackles on auscultation of the lungs

D. Crackles on auscultation of the lungs

A client with non-Hodgkin's lymphoma is receiving daunorubicin (DaunoXome). Which finding would indicate to the nurse that the client is experiencing an adverse effect related to the medication? A. Fever B. Sores in the mouth and throat C. Complaints of nausea and vomiting D. Crackles on auscultation of the lungs

D. Crackles on auscultation of the lungs Rationale: Cardiotoxicity noted by abnormal electrocardiographic findings or cardiomyopathy manifested as heart failure (lung crackles) is an adverse effect of daunorubicin. Bone marrow depression is also an adverse effect. Fever is a frequent side effect and sores in the mouth and throat can occur occasionally. Nausea and vomiting is a frequent side effect associated with the medication that begins a few hours after administration and lasts 24 to 48 hours. Options 1, 2, and 3 are not adverse effects.

A client brought to the emergency department states that he has accidentally been taking 2 times his prescribed dose of warfarin for the past week. After noting that the client has no evidence of obvious bleeding, the nurse plans to take which action? A. Prepare to administer an antidote B. Draw a sample for type and crossmatch and transfuse the client C. Draw a sample for an activated partial thromboplastin time (aPTT) level D. Draw a sample for prothrombin time (PT) and international normalized ratio (INR)

D. Draw a sample for prothrombin time (PT) and international normalized ratio (INR)

The nurse manager is teaching the nursing staff about signs and symptoms related to hypercalcemia in a client with metastatic prostate cancer, and tells the staff that which is a *late* sign or symptom of this oncological emergency? A. Headache B. Dysphagia C. Constipation D. Electrocardiographic changes

D. Electrocardiographic changes

The nurse is caring for a client who sustained superficial partial-thickness burns on the anterior lower legs and anterior thorax. Which finding does the nurse expect to note during the resuscitation/emergent phase of the burn? A. Decrease heart rate B. Increased urinary output C. Increased blood pressure D. Elevated hematocrit levels

D. Elevated hematocrit levels

A client is admitted to the hospital with a suspected diagnosis of Hodgkin's disease. Which assessment finding would the nurse expect to note specifically in the client? A. Fatigue B. Weakness C. Weight gain D. Enlarged lymph nodes

D. Enlarged lymph nodes

The nurse is giving a report to an unlicensed assistive personnel (UAP) who will be caring for client who has hand restraints (safety devices). The nurse instructs the UAP to check the skin integrity of the restrained hands how frequently? A. Every 2 hours B. Every 3 hours C. Every 4 hours D. Every 30 minutes

D. Every 30 minutes

The nurse is conducting a session about the principles of first aid and is discussing the interventions for a snakebite to an extremity. The nurse should inform those attending the session that the *first priority* intervention is the event of this occurrence is which action? A. Immobilize the affected extremity B. Remove jewelry and constricting clothing from the victim C. Place the extremity in a position so that it is below the level of the heart D. Move the victim to a safe area away from the snake and encourage the victim to rest

D. Move the victim to a safe area away from the snake and encourage the victim to rest

Megestrol acetate, an antineoplastic medication, is prescribed for a client with metastatic endometrial carcinoma. The nurse reviews the client's history and should contact the health care provider if which diagnosis is documented in the client's history? A. Gout B. Asthma C. Thrombophlebitis D. Myocardial infarction

D. Myocardial infarction Rationale: Megestrol acetate suppresses the release of luteinizing hormone from the anterior pituitary by inhibiting pituitary function and regressing tumor size. Megestrol is used with caution if the client has a history of venous thromboembolism. Options 1, 2, and 3 are not contraindications for this medication.

A nursing instructor delivers a lecture to nursing students regarding the issue of client's rights and asks a nursing student to identify a situation that represents an example of invasion of client privacy. Which situation, if identified by the student, indicates an understanding of a violation of this client right? A. Performing a procedure without consent B. Threatening to give a client a medication C. Telling the client that he or she cannot leave the hospital D. Observing care provided to the client without the client's permission

D. Observing care provided to the client without the client's permission

A client with a gastric ulcer is scheduled for surgery. The client cannot sign the operative consent form because of sedation from opioid analgesics that have been administered. The nurse should take which *most appropriate* action in the care of this client? A. Obtain a court order for the surgery B. Have the charge nurse sign the informed consent immediately C. Send the client to surgery without the consent being signed D. Obtain a telephone consent from a family member, following agency policy

D. Obtain a telephone consent from a family member, following agency policy

The evening nurse reviews the nursing documentation in the client's chart and notes that the day nurse has documented that the client has a stage 2 pressure ulcer in the sacral area. Which finding would the nurse expect to note on assessment of the client's sacral area? A. Intact skin B. Full-thickness skin loss C. Exposed bone, tendon, or muscle D. Partial-thickness skin loss of the dermis

D. Partial-thickness skin loss of the dermis

A client with ovarian cancer is being treated with vincristine (Vincasar). The nurse monitors the client, knowing that which manifestation indicates an adverse effect specific to this medication? A. Diarrhea B. Hair loss C. Chest pain D. Peripheral neuropathy

D. Peripheral neuropathy Rationale: An adverse effect specific to vincristine is peripheral neuropathy, which occurs in almost every client. Peripheral neuropathy can be manifested as numbness and tingling in the fingers and toes. Depression of the Achilles tendon reflex may be the first clinical sign indicating peripheral neuropathy. Constipation rather than diarrhea is most likely to occur with this medication, although diarrhea may occur occasionally. Hair loss occurs with nearly all the antineoplastic medications. Chest pain is unrelated to this medication.

While giving care to a client with an internal cervical radiation implant, the nurse finds the implant in the bed. the nurse should take which *initial* action? A. Call the health care provider (HCP) B. Reinsert the implant into the vagina C. Pick up the implant with gloved hands and flush it down the toilet D. Pick up the implant with long-handled forceps and place it in a lead container

D. Pick up the implant with long-handled forceps and place it in a lead container

A client with a history of gastrointestinal bleeding has a platelet count of 300,000 mm^3 (300 x 10^9/L). The nurse should take which action after seeing the laboratory results? A. Report the abnormally low count B. Report the abnormally high count C. Place the client on bleeding precautions D. Place the normal report in the client's medical record

D. Place the normal report in the client's medical record

A client receiving parenteral nutrition (PN) suddenly develops a fever. The nurse notifies the health care provider (HCP), and the HCP initially prescribes that the solution and tubing be changed. What should the nurse do with the discontinued materials? A. Discard them in the unit trash B. Return them to the hospital pharmacy C. Save them for return to the manufacturer D. Prepare to send them to the laboratory for culture

D. Prepare to send them to the laboratory for culture

A client with diabetes mellitus has a glycosylated hemoglobin A1c level of 9%. On the basis of this test result, the nurse plans to teach the client about the need for which measure? A. Avoiding infection B. Taking in adequate fluids C. Preventing and recognizing hypoglycemia D. Preventing and recognizing hyperglycemia

D. Preventing and recognizing hyperglycemia

The nurse is caring for a client with hypocalcemia. Which patterns would the nurse watch for on the electrocardiogram as a result of the laboratory value? *Select all that apply.* A. U waves B. Widened T wave C. Prominent U wave D. Prolonged QT interval E. Prolonged ST segment

D. Prolonged QT interval E. Prolonged ST segment

A client with squamous cell carcinoma of the larynx is receiving bleomycin intravenously. The nurse caring for the client anticipates that which diagnostic study will be prescribed? A. Echocardiography B. Electrocardiography C. Cervical radiography D. Pulmonary function studies

D. Pulmonary function studies Rationale: Bleomycin is an antineoplastic medication that can cause interstitial pneumonitis, which can progress to pulmonary fibrosis. Pulmonary function studies along with hematological, hepatic, and renal function tests need to be monitored. The nurse needs to monitor lung sounds for dyspnea and crackles, which indicate pulmonary toxicity. The medication needs to be discontinued immediately if pulmonary toxicity occurs. Options 1, 2, and 3 are unrelated to the specific use of this medication.

A health care provider has written a prescription to discontinue an intravenous (IV) line. The nurse should obtain which item from the unit supply area for applying pressure to the site after removing the IV catheter? A. Elastic wrap B. Povidone iodine swab C. Adhesive bandage D. Sterile 2x2 gauge

D. Sterile 2x2 gauge

A client with hypertension has been told to maintain a diet low in sodium. The nurse who is teaching this client about foods that are allowed should include which food item in a list provided to the client? A. Tomato soup B. Boiled shrimp C. Instant oatmeal D. Summer squash

D. Summer squash

The nurse is preparing to change the parenteral nutrition (PN) solution bag and tubing. The client's central venous line is located in the right subclavian vein. The nurse asks the client to take which *essential* action during the tubing change? A. Breathe normally B. Turn the head to the right C. Exhale slowly and evenly D. Take a deep breath, hold it, and bear down

D. Take a deep breath, hold it, and bear down

The nurse is caring for a client having respiratory distress related to an anxiety attic. Recent arterial blood gas values are pH=7.53, PaO2=72 mmHg, PaCO2=32mmHg, and HCO3-=28 mEq/L. Which conclusion about the client should the nurse make? A. The client has acidotic blood B. The client is probably overreacting C. The client is fluid volume overloaded D. The client is probably hyperventilating

D. The client is probably hyperventilating

On review of the clients' medical records, the nurse determines that which client is at risk for fluid volume excess? A. The client taking diuretics and has tenting of the skin B. The client with an ileostomy from a recent abdominal surgery C. The client who requires intermittent gastrointestinal suctioning D. The client with kidney disease and a 12-year history of diabetes mellitus

D. The client with kidney disease and a 12-year history of diabetes mellitus

Silver sulfadiazine is prescribed for a client with a burn injury. Which laboratory finding requires the *need for follow-up* by the nurse? A. Glucose level of 99 mg/dL (5.65 mmol/L) B. Magnesium level of 1.5 mEq/L (0.75 mmol/L) C. Platelet level of 300,000 mm^3 (300 x 10^9/L) D. White blood cell count of 3000mm^3 (3.0 x 10^9/L)

D. White blood cell count of 3000mm^3 (3.0 x 10^9/L) Rationale: Silver sulfadiazine is used for the treatment of burn injuries. Adverse effects of this medication include rash and itching, blue green or gray skin discoloration, leukopenia, and interstitial nephritis. The nurse should monitor a complete blood count, particularly the white blood cells, frequently for the client taking this medication. If leukopenia develops, the healthcare provider is notified and the medication is usually discontinued. The white blood cell count noted an option for is indicative of leukopenia. The other laboratory values are not specific to this medication, and are also within normal limits.

A male patient has been laid off from his construction job and has many unpaid bills. He is going through a divorce from his marriage of 15 years and has been seeing his pastor to help him through this difficult time. He does not have a primary health care provider because he has never really been sick and his parents never took him to see a physician when he was a child. Which external variables influence the patient's health practice?

Difficulty paying his bills; Family practice of not routinely seeing a health care provider.

the nurse spends time with the patient and family reviewing the dressing change procedure for the patients wound. the patients spouse demonstrates how to change the dressing. the nurse is acting in which role? - Educator - Advocate - Caregiver - Case Manager

Educator

A client has a prescription for inamrinone (Inocor) 0.75 mg/kg IV bolus to be delivered over 3 minutes. The client weighs 80 kg. Inamrinone is available for injection 100 mg/20 ml. How many ml of inamrinone should the nurse administer? (Enter numeric value only. If rounding is required, round to the nearest tenth.)

First calculate the mg/kg dosage needed: 0.75 mg x 80 kg = 60 mg needed. Then calculate Desired / Have x ml 60 mg / 100 mg x 20 ml = 12 ml.

0.9% normal saline with inamrinone (Inocor) 0.1 grams/100 ml is prescribed for a client with heart failure. The medication is to be delivered at a rate of 400 mcg/minute. The nurse should program the infusion pump to deliver how many ml/hour? (Enter numeric value only. If rounding is required, round to the nearest whole number.)

First calculate the number of mcg/hour: 400 mcg x 60 minutes = 24000 mcg/hour. Next calculate the number of ml/hour needed to administer 24,000 mcg/hour: 100,000 mcg: 100ml :: 24,000 mcg : X 100,000/24,000 :: 100/X 100,000X = 2,400,000 X = 24 ml/hour.

A client experiencing cardiogenic shock receives a prescription for an IV infusion of 0.9% normal saline with milrinone (Primacor) 10 mg in 100 ml at a rate of 46 mcg/minute. The nurse should program the infusion pump to deliver how many ml/hour? (Enter numeric value only. If rounding is required, round to the nearest whole number.)

First calculate the number of mcg/hour: 46 mcg x 60 minutes = 2760 mcg/hour. Next calculate the number of ml/hour needed to administer 2760 mcg/hour: 10,000 mcg : 100 ml :: 2760 mcg : X 10,000 / 2760 :: 100 / X 10,000X = 276000 X = 27.6 (rounded to) 28 ml/hour.

A client who had mitral valve replacement surgery receives a prescription for dextrose 5% in water with 0.5 grams of dobutamine in 250 ml for IV infusion at a rate of 5 mcg/kg/minute. The client weighs 75 kg. The nurse should program the infusion pump to deliver how many ml/hr? (Enter numeric value only. If rounding is required, round to the nearest tenth.)

First calculate the number of mcg/kg/minute: 5 x 75 = 375 mcg/minute. Calculate the number of mcg/hour: 375 x 60 minutes = 22,500 mcg/hour. Next calculate the number of ml/hour needed to administer 22,500 mcg/hour: 500,000 mcg : 250 ml :: 22,500 mcg/ X ml 500,000X = 5,625,000 X = 11.25 (rounded to 11.3 ml/hour).

A healthcare provider prescribes a continuous infusion of 0.9% sodium chloride with pancuronium (Pavulon) 25 mg/250 ml at a rate of 0.1 mg/kg/hr for a client with coronary artery bypass grafting. The client weighs 78 kg. The nurse should program the infusion pump to deliver how many ml/hour? (Enter numeric value only. If rounding is required, round to the nearest tenth.)

First calculate the number of mg the client will receive in 1 hour: 0.1 mg x 78 kg = 7.8 mg/hour. Next calculate the number of ml containing 7.8 mg of medication: 25 mg : 250 ml :: 7.8 mg : X 25 mg/ 7.8 mg :: 250 ml/X 25X = 1950 X = 78 ml/hour.

An IV infusion of dextrose 5% in normal saline with oxytocin (Pitocin) 20 units in 1,000 ml is prescribed for a client to control postpartum bleeding. The solution is to be administered at a rate of 2 mu/minute. The nurse should program the infusion pump to administer how many ml/hour? (Enter numeric value only. If rounding is required, round to the nearest whole number.)

First calculate the number of units/hour. 0.002 units/minute x 60 minutes = 0.12 units/hour. Next calculate the number of ml/hour needed to administer 0.12 units/hour. 20 units : 1,000 ml :: 0.12 units : X 20/0.12 :: 1,000/X 20X : 120 X = 6

A client with Mycobacterium avium complex (MAC) is receiving an infusion of 5 % dextrose in water with amikacin (Amikin) 0.572 grams/100 ml every 12 hours. The nurse prepares the dose of amikacin using a vial labeled, 250 mg/ml. How many ml should the nurse add to the infusion? (Enter numeric value only. If rounding is required, round to the nearest tenth.)

First convert gram to milligram: 1 gm = 1000mg .572 gram x 1000 = 572 mg. Next use the formula D/H x Q: 572/250 x 1 = 2.288 (rounded to) 2.3 ml.

A client returns to the nursing unit from surgery with a prescription for Lactated Ringer's solution 1,000 ml to be infused over 10 hours. The IV administration set delivers 60 gtts/ml. The nurse should regulate the infusion to deliver how many gtts/minute?

First convert hours to minutes: 10 hrs x 60 = 600 minutes. Using the formula Volume/Time x drop factor: 1000 ml /600 minutes = 1.66666 x 60 = 100 gtts minute

A client with septic shock receives a prescription for dextrose 5% in water with dopamine (Intropin) 0.6 grams/250 ml at a rate of 5 mcg/kg/minute. The client's weight is 148 lbs. The nurse should set the infusion pump to deliver how many ml/hour? (Enter numeric value only. If rounding is required, round to the nearest whole number.)

First convert the weight into kilograms: 148 pounds/2.2 kg = 67 kg. Next calculate the number of mcg/kg/minute: 5 x 67 x 1 minute = 335 mcg/min. Calculate the number of mcg/hour: 335 x 60 minutes = 20,100 mcg/hour. Now calculate the number of ml/hour needed to administer 20,100 mcg/hour: 600,000 mcg : 250 ml :: 20,100 mcg : X ml 600,000/20100 :: 250/X 600,000X = 5,02500 X = 8.375 (rounded to) 8 ml/hour.

The application of utilitarianism does not always resolve and ethical dilemma. Which of the following statements best explains why? 1. Utilitarianism refers to usefulness and therefore eliminate the need to talk about spiritual values 2. In a diverse community it can be difficult to find agreement on a definition of usefulness, the focus of utilitarianism 3. Even when the agreement about a definition of usefulness exists in a community law, laws prohibit an application of utilitarianism 4. Difficult ethical decisions cannot be resolved by talking about the usefulness of a procedure

In a diverse community it can be difficult to find agreement on a definition of usefulness, the focus of utilitarianism

A patient is laboring with her first baby, who is about to be delivered 2 weeks early. Her husband is in the military and might not get back in time, and both families are unable to be with her during labor. The doctor decides to call in which of the following people employed by the birthing area as a support person to be present during labor? Nurse Midwife Geneticist Lay doula

Lay doula (support person who is present during labor to assist women who have no other source of support)

Which of the following statements is true regarding Magnet status recognition for a hospital? A). Nursing is run by a Magnet manager who makes decisions for the nursing units B). Nurses in Magnet hospitals make all of the decisions on the clinical units C). Magnet is a term that is used to describe hospitals that are able to hire the nurses they need D). Magnet is a special designation for hospitals that achieve excellence in nursing practice

Magnet is a special designation for hospitals that achieve excellence in nursing practice - Magnet status is a process and review that hospitals go through that shows achievement of excellence in nursing practice. The designation is given by the American Nurses Credentialing Center and focuses on demonstration of quality patient care, nursing excellence, and innovations in professional practice.

Nurses on a nursing unit are discussing the processes that led up to a near-miss error on the clinical unit. They are outlining strategies that will prevent this in the future. This is an example of nurses working on what issue in the health care system? A). Patient safety B). Evidence-based practice C). Patient satisfaction D). Maintenance of competency

Patient safety - Nursing work groups or councils who had a commitment to patient safety were a positive characteristic of the patient safety climate on the nursing unit. Open communication and nurses involved in problem solving related to errors were other factors contributing to patient safety.

A nurse is providing restorative care to a patient following an extended hospitalization for an acute illness. Which of the following is an appropriate goal for restorative care? A). Patient will be able to walk 200 feet without shortness of breath B). Wound will heal without signs of infection C). Patient will express concerns related to return to home D). Patient will identify strategies to improve sleep habits

Patient will be able to walk 200 feet without shortness of breath - Restorative interventions focus on returning patients to their previous level of function or reaching a new level of function limited by their illness or disability. The goals of restorative care are to help individuals regain maximal functional status and enhance quality of life through promotion of independence.

The nurse is caring for a 50-year-old woman visiting the outpatient medicine clinic. The patient has had type 1 diabetes since age 13. She has numerous complications from her disease, including reduced vision, heart disease, and severe numbness and tingling of the extremities. Knowing that spirituality helps patients cope with their chronic illness, which of the following principles should the nurse apply in practice? (Select all that apply.)

Pay attention to the patient's spiritual identity throughout the course of her illness Listen to the patient's story each visit to the clinic and offer a compassionate presence

A nurse is presenting information to a management class of nursing students on the topic of financial reimbursement for achievement of established, measurable patient outcomes. The nurse is presenting information to the class on which topic? A). Prospective payment system B). Pay for performance C). Capitation payment system D). Managed care systems

Pay for performance - Pay for performance programs and public reporting of hospital quality data are designed to promote quality, effective, and safe patient care by physicians and health care organizations. These programs are quality improvement strategies that reward excellence through financial incentives to motivate change to achieve measurable improvements.

A group of staff nurses notice an increased incidence of medication errors on their unit. After further investigation it is determined that the nurses are not consistently identifying the patient correctly. A change is needed quickly. What type of quality improvement method would be most appropriate? A). PDSA B). Six Sigma C). Rapid-improvement event D). A randomized controlled trial

Rapid-improvement event - RIEs are very intense, usually week-long events, in which a group gets together to evaluate a problem with the intent of making radical changes to current processes. Changes are made within a very short time. The effects of the changes are measured quickly, results are evaluated, and further changes are made when necessary. An RIE is appropriate to use when a serious problem such as the increased occurrence of medication errors exists that greatly affects patient safety and needs to be solved quickly.

A nurse begins a night shift, assuming care for a critically ill patient who was resuscitated earlier in the day from cardiac arrest. He survived and is physically stable, alert, oriented, and responding appropriately to the nurse's questions. Knowing that the patient experienced a period when his heart stopped beating, what would be the best approach for the nurse to use with him?

Sit and encourage the patient to share what he experienced during resuscitation

A patient has just learned she has been diagnosed with a malignant brain tumor. She is alone; her family will not be arriving from out of town for an hour. You have cared for her for only 2 hours but have a good relationship with her. What might be the most appropriate intervention for support of her spiritual well-being at this time?

Sit down and talk with the patient; have her discuss her feelings and listen attentively

A 34-year-old female executive has a job with frequent deadlines. She notes that, when the deadlines appear, she has a tendency to eat high-fat, high-carbohydrate foods. She also explains that she gets frequent headaches and stomach pain during these deadlines. The nurse provides a number of options for the executive, and she chooses yoga. In this scenario yoga is used as a (n): Outpatient referral. Counseling technique. Health promotion activity. Stress-management technique.

Stress-management technique

Select the three factors that are evident when a healing relationship develops between nurse and patient.

The nurse being able to realistically mobilize hope for the patient Correct Finding an interpretation or understanding of the patient's illness that is acceptable to the patient Correct Helping the patient use spiritual resources that he or she chooses Correct

Middle-age adults frequently find themselves trying to balance responsibilities related to employment, family life, care of children, and care of aging parents. People finding themselves in this situation are frequently referred to as being a part of: The sandwich generation. The millennial generation. Generation X. Generation Y.

The sandwich generation

The patient tells the nurse that she is enrolled in a preferred provider organization (PPO) but does not understand what this is. What is the nurse's best explanation of a PPO? A). This health plan is for people who cannot afford their own health insurance B). This health plan is operated by the government to provide health care to older adults C). This health plan provides you with a preferred list of physicians, hospitals, and providers from which you can choose D). This is a fee-for-service plan in which you can choose any physician or hospital

This health plan gives you with a list of physicians and hospitals from which you can choose - PPO plans limit the enrollee's choice to a list of preferred providers such as hospitals and physicians. A participant pays more to use a provider not on the preferred list. PPO plans focus on health maintenance.

Resolution of an ethical dilemma involves discussion with the patient, the patient's family, and participants from all health care disciplines. Which of the following best describes the role of the nurse in the resolution of ethical dilemma? 1. To articulate the nurses' unique point of view, including knowledge based on clinical and psychosoical observation 2. To study the literature on current research about the possible clinical interventions available for the patient in question 3. To hold a point of view but realize that respect for the authority of administrators and physicians takes precedence over personal opinion 4. To allow the patient and the physician to resolve the dilemma on the basis of ethical principles without regard to personal held values or opinions

To articulate the nurses' unique point of view, including knowledge based on clinical and psychosoical observation

The healthcare provider prescribes an IV solution of 5% dextrose in water with magnesium sulfate 4 gram/50 ml to be infused over 30 minutes for a client with preeclampsia. The nurse should program the infusion pump to deliver how many ml/hour? (Enter numeric value only. If rounding is required, round to the nearest whole number.)

To calculate the rate of infusion for ml/hour: 50 ml: 30 minutes :: X ml : 60 minutes 50 ml / X ml :: 30 min / 60 min 3000 = 30X X = 100 ml/hour

The healthcare provider prescribes an IV infusion of 0.9% sodium chloride with 40 mEq KCl/500 ml to infuse over 3 hours for a client with hypokalemia. The nurse should program the infusion pump to deliver how many ml/hour? (Enter numeric value only. If rounding is required, round to the nearest whole number.)

To determine ml/hour: 500 ml : 3 hours :: as X ml : 1 hour 500/X :: 3/1 500 = 3X X = 166.66 (rounds to)= 167 ml/hour.

The healthcare provider prescribes oxytocin (Pitocin) 0.5 milliunits/minute for a client in labor. One liter Ringer's Lactate with 10 units of oxytocin (Pitocin) is infusing. The nurse should program the infusion pump at how many ml/hour? (Enter numeric value only.)

Using the formula, D/H x Q, 0.5 milliunits / 10,000 milliunits x 1000 ml = 0.05 ml/minute. Multiply by 60 minutes = 3 ml/hour.

Evaluation of spiritual care is necessary to determine if a patient's level of spiritual health has changed following intervention. If the use of rituals was part of a nurse's care plan, which of the following questions is most appropriate to evaluate its efficacy?

Were prayer or meditation helpful to you?

A nurse is caring for a 78-year-old patient with chronic multiple sclerosis. The patient has severe fatigue, muscle weakness, severe muscle spasms, and difficulties with coordination and balance. Her disease will likely worsen. The nurse has gained the patient's trust and wants to assess her life satisfaction. Which of the following questions should the nurse ask? (Select all that apply.)

What about your family makes you proudest? Looking back, what is your greatest accomplishment?

Felony

a crime, typically one involving violence, regarded as more serious than a misdemeanor, and usually punishable by imprisonment for more than one year or by death.

Nurse Midwife

a medically trained person who helps deliver babies

The Joint Commission (TJC)

a not-for-profit organization that evaluates and accredits different types of healthcare facilities

Laboratory

a person who performs the practical hands-on work in laboratories, work in diverse settings which include health care, industry, research, and educational institutions. May work in a wide variety of fields such as medicine, biology, chemistry, electronics, geology and the environment.

Data Cluster

a set of signs or symptoms gathered during assessment that you group together in a logical way. ex: nurse reports symptoms of: "patient wincing when incision palpated," "patient acknowledges discomfort over incision," "patient rates discomfort at 7 on scale of 0 to 10," "pain increases with movement." Analyzing data: pattern of a comfort problem.

Crimminal Law

a system of law concerned with the punishment of those who commit crimes. Concerned with acts that threaten society but may involve only an individual.

Which situation best indicates that the nurse has a good understanding regarding auditing and monitoring of patients' health records? a. The nurse determines the degree to which standards of care are met by reviewing patients' health records. b. The nurse realizes that care not documented in patients' health records still qualifies as care provided. c. The nurse knows that reimbursement is based on the diagnosis-related groups documented in patients' records. d. The nurse compares data in patients' records to determine whether a new treatment had better outcomes than the standard treatment.

a. The nurse determines the degree to which standards of care are met by reviewing patients' health records.

A nurse needs to begin discharge planning for a patient admitted with pneumonia and a congested cough. When is the best time the nurse should start discharge planning for this patient? a. Upon admission b. Right before discharge c. After the congestion is treated d. When the primary care provider writes the order

a. Upon admission

A patient has a diagnosis of pneumonia. Which entry should the nurse chart to help with financial reimbursement? a. Used incentive spirometer to encourage coughing and deep breathing. Lung congested upon auscultation in lower lobes bilaterally. Pulse ox 86%. Oxygen per nasal cannula applied at 2 L/min per standing order. b. Cooperative, patient coughed and deep breathed using a pillow as a splint. Stated, "felt better." Finally, patient had no complaints. c. Breathing without difficulty. Sitting up in bed watching TV. Had a good day. d. Status unchanged. Remains stable with no abnormal findings. Checked every 2 hours.

a. Used incentive spirometer to encourage coughing and deep breathing. Lung congested upon auscultation in lower lobes bilaterally. Pulse ox 86%. Oxygen per nasal cannula applied at 2 L/min per standing order.

A patient is being discharged home. Which information should the nurse include? a. Acuity level b. Community resources c. Standardized care plan d. Kardex

b. Community resources

A nurse wants to integrate all pertinent patient information into one record, regardless of the number of times a patient enters the health care system. Which term should the nurse use to describe this system? a. Electronic medical record b. Electronic health record c. Electronic charting record d. Electronic problem record

b. Electronic health record

A nurse is teaching the staff about health care reimbursement. Which information should the nurse include? a. Sentinel events help determine reimbursement issues for health care. b. Home health, long-term care, and hospital nurses' documentation can affect reimbursement for health care. c. A clinical information system must be installed by 2014 to obtain health care reimbursement. d. HIPAA is the basis for establishing reimbursement for health care.

b. Home health, long-term care, and hospital nurses' documentation can affect reimbursement for health care.

A nurse is discussing the advantages of standardized documentation forms in the nursing information system. Which advantage should the nurse describe? a. Varied clinical databases b. Reduced errors of omission c. Increased hospital costs d. More time to read charts

b. Reduced errors of omission

A nurse in a long-term care setting that is funded by Medicare and Medicaid is completing standardized protocols for assessment and care planning and for meeting quality improvement within and across facilities. Which task did the nurse just complete? a. A focused assessment/specific body system b. The Resident Assessment Instrument/Minimum Data Set c. An admission assessment and acuity level d. An intake assessment form and auditing phase

b. The Resident Assessment Instrument/Minimum Data Set

A preceptor is working with a new nurse on documentation. Which situation will cause the preceptor to intervene? a. The new nurse uses a black ink pen to chart. b. The new nurse charts consecutively on every other line. c. The new nurse ends each entry with signature and title. d. The new nurse keeps the password secure.

b. The new nurse charts consecutively on every other line.

A nurse obtained a telephone order from a primary care provider for a patient in pain. Which chart entry should the nurse document? a. 12/16/20XX 0915 Tylenol 3, 2 tablets, every 6 hours for incisional pain. VO Dr. Day/J. Winds, RN, read back. b. 12/16/20XX 0915 Tylenol 3, 2 tablets, every 6 hours for incisional pain. TO J. Winds, RN, read back. c. 12/16/20XX 0915 Tylenol 3, 2 tablets, every 6 hours for incisional pain. TO Dr. Day/J. Winds, RN, read back. d. 12/16/20XX 0915 Tylenol 3, 2 tablets, every 6 hours for incisional pain. TO J. Winds, RN.

c. 12/16/20XX 0915 Tylenol 3, 2 tablets, every 6 hours for incisional pain. TO Dr. Day/J. Winds, RN, read back.

A hospital is using a computer system that allows all health care providers to use a protocol system to document the care they provide. Which type of system/design will the nurse be using? a. Clinical decision support system b. Nursing process design c. Critical pathway design d. Computerized provider order entry system

c. Critical pathway design

A nurse is giving a hand-off report to the oncoming nurse. Which information is critical for the nurse to report? a. The patient had a good day with no complaints. b. The family is demanding and argumentative. c. The patient has a new pain medication, Lortab. d. The family is poor and had to go on welfare.

c. The patient has a new pain medication, Lortab.

A nurse preceptor is working with a student nurse. Which behavior by the student nurse will require the nurse preceptor to intervene? a. The student nurse reviews the patient's medical record. b. The student nurse reads the patient's plan of care. c. The student nurse shares patient information with a friend. d. The student nurse documents medication administered to the patient.

c. The student nurse shares patient information with a friend.

A nurse is using the source record and wants to find the daily weights. Where should the nurse look? a. Database b. Medical history and examination c. Progress notes d. Graphic sheet and flow sheet

d. Graphic sheet and flow sheet

A home health nurse is preparing for an initial home visit. Which information should be included in the patient's home care medical record? a. Nursing process form b. Step-by-step skills manual c. A list of possible procedures d. Reports to third party payers

d. Reports to third party payers

A nurse is preparing a change-of-shift report for a patient who had chest pain. Which information is critical for the nurse to include? a. Pupils equal and reactive to light b. The family is a "pain" c. Had poor results from the pain medication d. Sharp pain of 8 on a scale of 1 to 10

d. Sharp pain of 8 on a scale of 1 to 10

After providing care, a nurse charts in the patient's record. Which entry should the nurse document? a. Appears restless when sitting in the chair b. Drank adequate amounts of water c. Apparently is asleep with eyes closed d. Skin pale and cool

d. Skin pale and cool

Durable Power of Attorneys For Health Care (DPAHC)

is a legal document that designates a person or people of one's choosing to make health care decisions when the patient is no longer able to make decisions on his or her own behalf.

unintentional tort

negligence and malpractice

Which of the following properly applies an ethical principle to justify access to health care? (select all that apply) 1. Access to health care reflects the commitment of society to principles of beneficence and justice 2. If low income compromises access to care, respect for autonomy is compromised 3. Access to health care is a privilege in the United Sates, not a right 4. Poor access to affordable health care causes her, that is ethically troubling because nonmalficence is a basic principle of health care ethics. 5. Providers are exempt from fidelity to peo;e with drug addiction because addiction reflect a lack of personal accountability 6. If a new drug is discovered that cures a disease but at great cost per patient, the principle of justice suggests that the drug should be made available to those who can afford it

1, 2, 4

The ethics of care suggests that ethical dilemmas can best be solved by attention to relationships. How does this differ from other ethical practices? (select all that apply) 1. Ethics of care pays attention to the environment in which caring occurs 2. Ethics of care us used only in nursing practice 3. Ethics of care focuses only on the code of ethics for nursing 4. Ethics of care focuses only on only understanding relationships

1, 2, 5

It can be difficult to agree on a common definition of the word quantity when it comes to quality of life. Why? (select all that apply) 1. Average income varies in different regions of the country 2. Community vales influence definitions of quality, and are subject to change over time 3. Individual experiences influence perceptions of quality in different ways, making consensus difficult. 4. The value of elements such as cognitive skills, ability to perform meaningful work, and relationship to family is difficult to quantify using objective measures 5. Statistical analysis is difficult to apply when the outcome cannot be quantified 6. Whether or not a person has a job is an objective measure, but it does not play a role in understanding quality of life

2 3 4 5

What are the correct steps to resolve an ethical dilemma on a clinical unit? Place the steps in correct order. 1. Clarify values 2. Ask the questions, Is this an ethical dilemma? 3. Verbalize the problem 4. Gather information 5. Identify course of action 5. Evaluate the plan 6. Negotiate the plan

2, 4, 1, 3, 5, 7, 6

Ethical dilemmas often arise over a conflict of opinion. Reliance on a predictable series of steps can help people in a conflict find common ground. What is the best order of these actions in order to promote the resolution of an ethical dilemma? 1. List the actions that could be taken to resolve the action over time 2. Agree on a statement of the problem or dilemma that you are trying to resolve 3. Agree on a plan to evaluate action overtime 4. Gather all relevant information regarding the clinical, social and spiritual aspects of the dilemma. 5. Take time to clarify values and distinguish between two facts and opinions--your own and those of others involved. 6. Negotiate a plan

4, 5, 2, 1, 6, 3

The nurse notes that an advance directive is on a patient's medical record. Which statement represents the best description of an advance directive guideline the nurse will follow? A. A living will allows an appointed person to make health care decisions when the patient is in an incapacitated state. B. A living will is invoked only when the patient has a terminal condition or is in a persistent vegetative state. C. The patient cannot make changes in the advance directive once admitted to the hospital. D. A durable power of attorney for health care is invoked only when the patient has a terminal condition or is in a persistent vegetative state. B. A living will is invoked only when the patient has a terminal condition or is in a persistent vegetative state. A living will does not assign another individual to make decisions for the patient. A durable power of attorney for health care is active when the patient is incapacitated or cognitively impaired. A cognitively intact patient may

A living will is invoked only when the patient has a terminal condition or is in a persistent vegetative state. A living will does not assign another individual to make decisions for the patient. A durable power of attorney for health care is active when the patient is incapacitated or cognitively impaired. A cognitively intact patient may change an advance directive at any time.

A new graduate nurse is being mentored by a more experienced nurse. They are discussing the ways nurses need to remain active professionally. Which of the statements below indicates the new graduate understands ways to remain involved professionally? (Select all that apply.) A. "I am thinking about joining the health committee at my church." B. "I need to read newspapers, watch news broadcasts, and search the Internet for information related to health." C. "I will join nursing committees at the hospital after I have several years of experience and better understand the issues affecting nursing." D. "Nurses do not have very much voice in legislation in Washington, DC, because of the shortage of nurses.

A. "I am thinking about joining the health committee at my church." B. "I need to read newspapers, watch news broadcasts, and search the Internet for information related to health." Nurses need to be actively involved in their community and be aware of current issues in health care. Staying abreast of current news and public opinion through the media is essential. Nurses need to join nursing committees at all levels of their career, not just when they have experience. Nurses have a powerful voice in the legislature.

You are floated to work on a nursing unit where you are given an assignment that is beyond your capability. What is the best nursing action to take first? A. Call the nursing supervisor to discuss the situation B. Discuss the problem with a colleague C. Leave the nursing unit and go home D. Say nothing and begin your work

A. Call the nursing supervisor to discuss the situation Alerting the nursing supervisor as a representative of the hospital administration is the first step in providing notice that a problem may exist related to insufficient staffing. This notice serves to share the burden of knowledge of the staffing inequity issues that may create an unsafe patient situation for the hospital and nursing staff.

The patient has a fractured femur that is placed in skeletal traction with a fresh plaster cast applied. The patient experiences decreased sensation and a cold feeling in the toes of the affected leg. The nurse observes that the patient's toes have become pale and cold but forgets to document this because one of the nurse's other patients experienced cardiac arrest at the same time. Two days later the patient in skeletal traction has an elevated temperature, and he is prepared for surgery to amputate the leg below the knee. Which of the following statements regarding a breach of duty apply to this situation? (Select all that apply.) A. Failure to document a change in assessment data B. Failure to provide discharge instructions C. Failure to follow the six rights of medication administration D. Failure to use proper medical equipment ordered for patient monitoring E. Failure to notify a health care provider about a change in the patient's condition

A. Failure to document a change in assessment data E. Failure to notify a health care provider about a change in the patient's condition The failure to document a change in assessment data and the failure to notify a health care provider about a change in patient status reflect a breach of duty to the patient.

Which of the following actions, if performed by a registered nurse, would result in both criminal and administrative law sanctions against the nurse? (Select all that apply.) A. Taking or selling controlled substances B. Refusing to provide health care information to a patient's child C. Reporting suspected abuse and neglect of children D. Applying physical restraints without a written physician's order

A. Taking or selling controlled substances D. Applying physical restraints without a written physician's order The inappropriate use of controlled substances is prohibited by every Nurse Practice Act. A physical restraint can be applied only on the written order of a health care provider based on Joint Commission and Medicare guidelines.

National League for Nursing Accrediting Commission

Affiliated with the National League for Nursing, this commission is an accrediting body for all types of nursing education programs.

Nurse Practitioner (NP)

An APRN who provides health care to a group of patients, usually in an outpatient, ambulatory care or community-based setting

Clinical Nurse Specialist (CNS)

An advanced practice nurse who provides direct care to clients and participates in health education and research.

an 18 year old woman is the ER with fever and cough. the nurse obtains her vital signs, listens to lung and heart sounds determines level of comfort, collects blood and sputum. which standard of practice is performed? - Diagnosis - Evaluation - Assessment - Implementation

Assessment

A nurse is caring for a patient who recently had coronary bypass surgery. Which are legal sources of standards of care the nurse uses to deliver safe health care? (Select all that apply.) A. Information provided by the head nurse B. Policies and procedures of the employing hospital C. State Nurse Practice Act D. Regulations identified in The Joint Commission's manual E. The American Nurses Association standards of nursing practice

B, C, D, E Policies and procedures of employing agencies and standards set by statutes, accrediting agencies, and professional organizations describe the minimum requirements for safe care.

A home health nurse notices significant bruising on a 2-yearold patient's head, arms, abdomen, and legs. The patient's mother describes the patient's frequent falls. What is the best nursing action for the home health nurse to take? A. Document her findings and treat the patient B. Instruct the mother on safe handling of a 2-year-old child C. Contact a child abuse hotline D. Discuss this story with a colleague

C. Contact a child abuse hotline Nurses are mandated reporters of suspected child abuse. These assessment findings possibly indicate child abuse.

You are the night shift nurse and are caring for a newly admitted patient who appears to be confused. The family asks to see the patient's medical record. What is the first nursing action to take? A. Give the family the record B. Give the patient the record C. Discuss the issues that concern the family with them D. Call the nursing supervisor

C. Discuss the issues that concern the family with them Family members do not have the right to private personal health information without the consent of the patient. Establishing a therapeutic relationship with the family and exploring their concerns gives you the information you need to determine how to best address their concerns.

The point of the ethical practice is an agreement to reassure the public that in all ways the health care team not only works to heal patients but agrees to do this in the least painful and harmful way possible. This principle is commonly called the principle of _____________?

nonmaleficence

The nurse received a hand-off report at the change of shift in the conference room from the night shift nurse. The nursing student assigned to the nurse asks to review the medical records of the patients assigned to them. The nurse begins assessing the assigned patients and lists the nursing care information for each patient on each individual patient's message board in the patient rooms. The nurse also lists the patients' medical diagnoses on the message board. Later in the day the nurse discusses the plan of care for a patient who is dying with the patient's family. Which of these actions describes a violation of the Health Insurance Portability and Accountability Act (HIPAA)? A. Discussing patient conditions in the nursing report room at the change of shift B. Allowing nursing students to review patient charts before caring for patients to whom they are assigned C. Posting medical information about the patient on a message board in the patient's room D. Releasing patient information regarding terminal illness to family when the patient has given permission for information to be shared

C. Posting medical information about the patient on a message board in the patient's room Posting the medical condition of a patient on a message board in the patient's room is not necessary for the patient's treatment. Doing so can result in this information being accessed by persons who are not involved in the patient's treatment.

A nurse stops to help in an emergency at the scene of an accident. The injured party files a suit, and the nurse's employing institution insurance does not cover the nurse. What would probably cover the nurse in this situation? A. The nurse's automobile insurance B. The nurse's homeowner's insurance C. The Good Samaritan laws, which grant immunity from suit if there is no gross negligence D. The Patient Care Partnership, which may grant immunity from suit if the injured party consents

C. The Good Samaritan laws, which grant immunity from suit if there is no gross negligence The Good Samaritan law holds the health care provider immune from liability as long as he or she functions within the scope of his or her expertise.

A nurse is sued for failure to monitor a patient appropriately after a procedure. Which of the following statements are correct about this lawsuit? (Select all that apply.) A. The nurse represents the plaintiff. B. The defendant must prove injury, damage, or loss. C. The person filing the lawsuit has the burden of proof. D. The plaintiff must prove that a breach in the prevailing standard of care caused an injury.

C. The person filing the lawsuit has the burden of proof. D. The plaintiff must prove that a breach in the prevailing standard of care caused an injury. The patient as plaintiff must prove that the defendant nurse had a duty, breached the duty, and because of this breach caused the patient injury or damage.

Health Information Technology Act (HITECH Act, 2009)

Civil and criminal sanctions may be brought against both the nurse and the organization should either or both violate HIPPA or this act by willfully or inadvertently posting patient health information (phi) on social media.

A woman who is a Jehovah's Witness has severe life-threatening injuries and is hemorrhaging following a car accident. The health care provider ordered 2 units of packed red blood cells to treat the woman's anemia. The woman's husband refuses to allow the nurse to give his wife the blood. What is the nurse's responsibility? A. Obtain a court order to give the blood B. Coerce the husband into giving the blood C. Call security and have the husband removed from the hospital D. Abide by the husband's wishes and inform the health care provider D. Abide by the husband's wishes and inform the health care provider Adult patients such as those who are Jehovah's Witnesses are able

D. Abide by the husband's wishes and inform the health care provider Adult patients such as those who are Jehovah's Witnesses are able to refuse treatment for personal religious reasons.

A homeless man enters the emergency department seeking health care. The health care provider indicates that the patient needs to be transferred to the City Hospital for care. This action is most likely a violation of which of the following laws? A. Health Insurance Portability and Accountability Act (HIPAA) B. Americans with Disabilities Act (ADA) C. Patient Self-Determination Act (PSDA) D. Emergency Medical Treatment and Active Labor Act (EMTALA)

D. Emergency Medical Treatment and Active Labor Act (EMTALA) The EMTALA requires that an emergency situation needs to be established and that the patient needs to be stabilized before a transfer is appropriate.

you are preparing a presentation for your classmates regarding the clinical care coordination conference for a patient with terminal cancer. as part of the preparation you have your classmates read the nursing code of ethics for professional registered nurses. your instructor asks the class why this document is important. what best describes this code? - Improves self-health care - Protects the patient's confidentiality - Ensures identical care to all patients - Defines the principals of right and wrong to provide patient care

Defines the principals of right and wrong to provide patient care

A nurse notes that the health care unit keeps a listing of the patient names at the front desk in easy view for health care providers to more efficiently locate the patient. The nurse talks with the nursing manager because this action is a violation of which act? A. Mental Health Parity Act B. Patient Self-Determination Act (PSDA) C. Health Insurance Portability and Accountability Act (HIPAA) D. Emergency Medical Treatment and Active Labor Act C. Health Insurance Portability and Accountability Act (HIPAA) The Privacy Rule of HIPAA requires that patient

Health Insurance Portability and Accountability Act (HIPAA) The Privacy Rule of HIPAA requires that patient information be protected from unnecessary publication.

The ANA code of nursing ethics articulates that the nurse "promotes, advocates for, and strives to protect the health, safety, and rights of the patient". This includes the protection of patient privacy. On the basis of this principle, if you participate in a public online social network such as Facebook, could you post images of a patient's x-ray film if you obscured or deleted all patient identifiers? 1. Yes, because the patient privacy would not be violated since patient identifiers were removed 2. Yes, because respect for autonomy implies that you have the autonomy to decide what constitutes as privacy 3. No, because even though the patient identifiers are removed, someone could identify the patient on the basis of other comments that you could make online about his or her conditions and your place of work 4. No, because the principle of justice requires you to allocate resources fairly

No, because even though the patient identifiers are removed, someone could identify the patient on the basis of other comments that you could make online about his or her conditions and your place of work

the nurses on an acute care medical floor notice an increase in pressure ulcer formation in their patients. a nurse consultant decides to compare two types of treatment. the first is the procedure currently used to assess the risk for pressure ulcers. the second uses a new assessment instrument to identify at risk patients. given this info, the nurse consultant exemplifies what career - Clinical nurse specialist - Nurse administrator - Nurse Educator - Nurse researcher

Nurse researcher

In most ethical dilemmas in healthcare, the solution to the dilemma requires negotiation among members of the health care team. Why is the nurse's point of view valuable? 1. Nurses understand the principle of autonomy to guide respect for a patient's self-worth 2. Nurses have a scope of practice that encourages their presence during ethical decisions 3. Nurses develop a relationship with the patient that is unique among all professional health care providers. 4. The nurse's code of ethics recommends that a nurse be present at any ethical discussion about patient care

Nurses develop a relationship with the patient that is unique among all professional health care providers

Autonmy

Patient Involve patients in the decision making process of card Nurse Involves the initiation of independent nursing interventions without medical orders and taking responsibility for those actions.

informed consent

Patient gives consent for a procedure to be performed in full knowledge of the procedure and the risk it entails (signed)

Patient Advocacy

The act of speaking and acting on behalf of the patient's needs and well-being.

Statutory Law

The body of law enacted by legislative bodies (as opposed to constitutional law, administrative law, or case law).

American Nurses Association (ANA)

The national professional membership association of nurses that works for the improvement of health standards and the availability of healthcare services, fosters high professional standards for the nursing profession, and advances the economic and general welfare of nurses

Patient Bill of Rights

a development by the American Medical Association that guarantees a patient the right to receive courteous, considerate,respectful treatment in a clean/safe environment; appropriate healthcare; information about his/her health treatment plan in a way that he or she understands; a continuity of care; confidentiality privacy;participation in planning care and treatment; refusal of care; use of grievance mechanisms; treatment without discrimination;an itemized bill and explanation of all charges; and review of the medical record and/or copy at reasonable fee.

health care reform will bring changes in the emphasis of care. which of the following models is expected form the health care reform - Moving from an acute illness to a health promotion, illness prevention model - Moving from an illness prevention to a health promotion model - Moving from an acute illness to a disease management model - Moving from a chronic care to an illness prevention model

moving from an acute illness to a health promotion illness model

end-of-life issues

discussing and formalizing management of life's final phases, after-death disposition of one's body, and lawful distribution of assets Discuss which treatments and procedures are futile or not

how does knowledge of genomes affect patient treatment decisions

genomics is the study of all the genes in a person as well as the interactions of those genes with each other and with that persons environment. genomic info allows health care providers to determine how genomic changes contribute to patient conditions and influence treatment decisions

Nurse Practice Act

law established to regulate nursing practice Statues enacted by the legislature of any of the states or the appropriate officers and define the scope of practice.

advance directives

legal documents that allow individuals to state what medical treatment they want or do not want in the event that they become incapacitated and are unable to express their wishes regarding medical care Living Will Health Care Proxies or Durable Power of Attorneys For Health Care

the exam for RN licensure is exactly the same in every state. this exam: - Guarantees safe nursing care for all patients - Ensures standard nursing care for all patients - Ensures that honest and ethical care is provided - Provides a minimal standard of knowledge for an RN in practice

provides a minimal standard of knowledge for an RN in practice

HIPPA Technology

regulations governing both electronic medical records and electronic communications


Set pelajaran terkait

Health Research Quiz 1 Practice Questions

View Set

Plant Anatomy and Nutrient Acquisition

View Set

Module 2 Section I: Supplier Relationship Management (SRM)

View Set

The Reconstruction Era: 1865-1877

View Set

Lesson 14 Managing Media Literacy

View Set

4.04 quiz understanding the law of supply

View Set

Chapter 59: Assessment and Management of Patients with Male Reproductive Disorders

View Set

Brazilian Portuguese Vocabulary: Your home = Sua casa by Street Smart Brazil

View Set

Chapter 1: Creating Customer Relationships and Value through Marketing

View Set